SlideShare ist ein Scribd-Unternehmen logo
1 von 18
Downloaden Sie, um offline zu lesen
www.VNMATH.com




                    MỘT SỐ BÀI TOÁN SỐ HỌC
                    LIÊN QUAN ĐẾN LŨY THỪA
                                 Phạm Văn Quốc
                   (Trường THPT chuyên Khoa Học Tự Nhiên)


    Trong các kỳ thi học sinh giỏi chúng ta hay gặp các bài toán số học liên quan
đến lũy thừa như chứng minh sự chia hết, chứng minh sự tồn tại hoặc tìm các
số nguyên thỏa mãn điều kiện,... Trong những năm gần đây, dạng toán này cũng
xuất hiện nhiều trong các đề thi quốc gia, đề thi chọn đội tuyển thi quốc tế
(CĐT) của các nước, các đề dự tuyển và các đề thi Toán quốc tế (IMO). Đây là
những bài toán hay và tất nhiên không dễ nếu không nắm được một số kỹ thuật
cũng như nhận dạng được kiểu bài toán. Các lời giải thường sử dụng công cụ
không khó nhưng chứa đựng nhiều sự tinh tế và sự linh hoạt trong vận dụng kiến
thức. Bài viết dưới đây đề cập đến một số kiến thức cơ bản và kỹ năng liên quan
đến các bài toán dạng này.

I. Kiến thức cơ bản
   Trong phần này là một số kiến thức cơ bản nhưng chúng hay được dùng trong
các dạng toán mà ta đang xét: công thức lũy thừa, số mũ "đúng", định lý Fermat,
Định lý Euler, cấp của số nguyên và một số tính chất liên quan hay dùng .
1. Một số khai triển liên quan đến lũy thừa
Định lý 1. Cho 𝑛 là số nguyên dương, khi đó với 𝑥, 𝑦 bất kỳ ta có
   ∘ 𝑥 𝑛 − 𝑦 𝑛 = (𝑥 − 𝑦) (𝑥 𝑛−1 + 𝑥 𝑛−2 𝑦 + 𝑥 𝑛−3 𝑦 2 + · · · + 𝑥𝑦 𝑛−2 + 𝑦 𝑛−1 ) ,
   ∘ 𝑥 𝑛 + 𝑦 𝑛 = (𝑥 + 𝑦) (𝑥 𝑛−1 − 𝑥 𝑛−2 𝑦 + 𝑥 𝑛−3 𝑦 2 − · · · − 𝑥𝑦 𝑛−2 + 𝑦 𝑛−1 ) nếu 𝑛 lẻ,
   ∘ (𝑥 + 𝑦) 𝑛 = 𝑥 𝑛 + 𝐶 1 𝑥 𝑛−1 𝑦 + 𝐶 2 𝑥 𝑛−2 𝑦 2 + · · · + 𝐶 𝑛𝑛−1 𝑥𝑦 𝑛−1 + 𝑦 𝑛 .
                         𝑛             𝑛

   Ta hãy bắt đầu bằng ví dụ sau.
Ví dụ 1. (Romania 2002) Cho 𝑘, 𝑛 là các số nguyên dương với 𝑛 > 2. Chứng
minh rằng phương trình
                                𝑥𝑛 − 𝑦 𝑛 = 2𝑘
không có nghiệm nguyên dương.

                                        Lời giải.

   Giả sử phương trình có nghiệm nguyên dương (𝑥, 𝑦). Nếu gcd (𝑥, 𝑦) = 𝑑 >
1 ⇒ 𝑑 | 2 𝑘 nên 𝑑 là lũy thừa của 2. Bằng cách chia hai vế cho 𝑑 𝑛 , ta có thể giả sử
gcd (𝑥, 𝑦) = 1 và suy ra 𝑥, 𝑦 lẻ.

                                            1
www.VNMATH.com



     Nếu 𝑛 chẵn, 𝑛 = 2𝑚 ta có 𝑥 𝑛 − 𝑦 𝑛 = (𝑥 𝑚 − 𝑦 𝑚 ) (𝑥 𝑚 + 𝑦 𝑚 ) nên 𝑥 𝑚 − )︀ 𝑚 =
                                                                        (︀          𝑦
  𝑎    𝑚    𝑚     𝑘−𝑎                                        𝑚      𝑎−1        𝑘−2𝑎
2 , 𝑥 + 𝑦 =2          với 𝑎 là số nguyên dương. Khi đó 𝑥 = 2               1+2        mà
 𝑥 lẻ nên 𝑎 = 1. Hơn nữa vì 𝑚 ≥ 2 nên

               𝑥 𝑚 − 𝑦 𝑚 = (𝑥 − 𝑦) 𝑥 𝑚−1 + 𝑥 𝑚−2 𝑦 + · · · + 𝑦 𝑚−1 > 2
                                   (︀                             )︀


mâu thuẫn.
  Do đó 𝑛 là số lẻ. Ta có

                  𝑥 𝑛 − 𝑦 𝑛 = (𝑥 − 𝑦) 𝑥 𝑛−1 + 𝑥 𝑛−2 𝑦 + · · · + 𝑦 𝑛−1 .
                                     (︀                              )︀


Nhưng do 𝑥, 𝑦 lẻ nên 𝑥 𝑛−1 + 𝑥 𝑛−2 𝑦 + · · · + 𝑦 𝑛−1 ≡ 𝑛 ≡ 1 (mod 2). Suy ra 𝑥 𝑛−1 +
 𝑥 𝑛−2 𝑦 + · · · + 𝑦 𝑛−1 = 1, điều này là không thể vì 𝑥, 𝑦 nguyên dương và 𝑛 > 2. Vậy
phương trình đã cho vô nghiệm.
     Lời giải của ví dụ này chủ yếu dùng công thức của hiệu hai lũy thừa và tính
chất tích của hai số nguyên dương là lũy thừa của 2 thì mỗi số là lũy thừa của 2.
Ví dụ tiếp theo cũng có ý giải cũng gần giống nhưng cần một chút khéo léo hơn.
Ví dụ 2. (Dự tuyển IMO 2008) Cho 𝑛 là số nguyên dương và 𝑝 là số nguyên tố.
Chứng minh rằng nếu 𝑎, 𝑏, 𝑐 là các số nguyên (không nhất thiết dương) thỏa mãn
đẳng thức
                                  𝑎 𝑛 + 𝑝𝑏 = 𝑏 𝑛 + 𝑝𝑐 = 𝑐 𝑛 + 𝑝𝑎
thì 𝑎 = 𝑏 = 𝑐.

                                       Lời giải.

   Ta chứng minh bài toán bằng phản chứng. Rõ ràng nếu hai trong ba số 𝑎, 𝑏, 𝑐
bằng nhau thì tất cả chúng bằng nhau. Giả sử cả ba số phân biệt đôi một, khi
đó theo giả thiết ta có
                           𝑎𝑛− 𝑏𝑛 𝑏𝑛− 𝑐𝑛 𝑐𝑛− 𝑎𝑛
                                 .      .       = −𝑝3 .
                            𝑎− 𝑏   𝑏− 𝑐   𝑐− 𝑎

Vì vế phải âm nên có ít nhất một trong ba số 𝑎, 𝑏, 𝑐 là số âm. Hơn nữa 𝑛 phải là
số chẵn (nếu trái lại suy ra mỗi thừa số ở vế trái là số dương).
    +, Nếu 𝑝 là số lẻ, mà
                          𝑎𝑛− 𝑏𝑛
                      2          = 𝑎 𝑛−1 + 𝑎 𝑛−2 𝑏 + · · · + 𝑏 𝑛−1 .
                           𝑎− 𝑏

Suy ra 𝑎, 𝑏 khác tính chẵn lẻ, tức là 2 𝑎 − 𝑏. Tương tự 2 𝑏 − 𝑐, 2 𝑐 − 𝑎 đây là
điều mâu thuẫn.
   +, Nếu 𝑝 = 2. Rõ ràng nếu một trong ba số 𝑎, 𝑏, 𝑐 bằng 0 thì cả ba số bằng 0.
Xét |𝑎| , |𝑏| , |𝑐| ≥ 1. Đặt 𝑛 = 2𝑚 ta có

                          𝑎𝑛− 𝑏𝑛          𝑎2𝑚 − 𝑏2𝑚
                  𝐴 =            = (𝑎 + 𝑏) 2
                           𝑎− 𝑏             𝑎 − 𝑏2

                                            2
www.VNMATH.com



                    = (𝑎 + 𝑏) 𝑎2𝑚−2 + 𝑎2𝑚−4 𝑏2 + · · · + 𝑏2𝑚−2 .
                             (︀                               )︀


Dễ thấy nếu 𝑚 > 1 ta có ngay |𝐴| ≥ 4 (do 𝐴 ̸= 0) và tương tự suy ra mâu thuẫn
vì tích của chúng là −8. Do đó 𝑚 = 1 ⇒ 𝑛 = 2 và ta thu được

                           (𝑎 + 𝑏) (𝑏 + 𝑐) (𝑐 + 𝑎) = −8.

Chú ý là do 𝑝 = 2 nên từ giả thiết ta có ngay 𝑎, 𝑏, 𝑐 cùng tính chẵn lẻ, suy ra
𝑎 + 𝑏, 𝑏 + 𝑐, 𝑐 + 𝑎 chẵn. Mà −8 = 2.2. (−2) nên dễ thấy hai trong ba số 𝑎, 𝑏, 𝑐 phải
bằng nhau, suy ra 𝑎 = 𝑏 = 𝑐 (mâu thuẫn). Từ đó ta có điều phải chứng minh.
2. Số mũ "đúng" (exact exponent)
Định nghĩa 2. Cho 𝑝 là số nguyên tố, 𝑎 là số nguyên và 𝛼 là số tự nhiên. Ta nói
 𝑝 𝛼 là ước đúng (exact power) của 𝑎, và 𝛼 được gọi là số mũ đúng của 𝑝 trong khai
triển của 𝑎, nếu 𝑝 𝛼 | 𝑎 và 𝑝 𝛼+1 𝑎. Khi đó ta viết 𝑝 𝛼 ‖ 𝑎 và ký hiệu 𝛼 = 𝑣 𝑝 (𝑎).
Tính chất 3. Cho 𝑎, 𝑏 là các số nguyên, khi đó ta có
      ∘ Nếu 𝑝 𝛼 ‖ 𝑎 và 𝑝 𝛽 ‖ 𝑏 thì 𝑝 𝛼+𝛽 ‖ 𝑎𝑏,
      ∘ Nếu 𝑝 𝛼 ‖ 𝑎 thì 𝑝 𝑘𝛼 ‖ 𝑎 𝑘 ,
      ∘ Nếu 𝑝 𝛼 ‖ 𝑎 và 𝑝 𝛽 ‖ 𝑏 với 𝑎 ̸= 𝑏 thì 𝑝min(𝛼,𝛽) ‖ 𝑎 + 𝑏.
Ví dụ 3. Cho 𝑎, 𝑘 là các số nguyên dương và 𝑝 là số nguyên tố lẻ, 𝛼 ∈ N+ sao
cho 𝑝 𝛼 ‖ 𝑎 − 1, khi đó với mọi số nguyên 𝛽 ≥ 0 thì 𝑝 𝛼+𝛽 ‖ 𝑎 𝑘 − 1 ⇔ 𝑝 𝛽 ‖ 𝑘.

                                     Lời giải.

   Ta chứng minh bằng quy nạp theo 𝛽. Nếu 𝛽 = 0, thì

           𝑎𝑘 −1
                 = 𝑎 𝑘−1 + · · · + 𝑎 + 1 ≡ 𝑘 (mod 𝑝) do 𝑎 ≡ 1 (mod 𝑝)
           𝑎−1

và suy ra nó không chia hết cho 𝑝. Bài toán đúng.
   Giả sử bài toán đúng với 𝛽 ≥ 0 nào đó và 𝑘 = 𝑝 𝛽+1 𝑡 với 𝑝 𝑡. Theo giả thiết
                         𝛽
quy nạp ta có 𝑎 𝑘/𝑝 = 𝑎 𝑝 𝑡 = 𝑚𝑝 𝛼+𝛽 + 1 với 𝑚 không chia hết cho 𝑝. Hơn nữa ta
có
                                        )︀ 𝑝
             𝑎𝑘 −1 =         𝑚𝑝 𝛼+𝛽 + 1 − 1
                          (︀
                                   )︀ 𝑝                 )︀2
                             𝑚𝑝 𝛼+𝛽 + · · · + 𝐶 2 𝑚𝑝 𝛼+𝛽 + 𝑚𝑝 𝛼+𝛽+1 .
                          (︀                      (︀
                     =                          𝑝

              𝑝 (𝑝 − 1)
Vì 𝑝 | 𝐶 2 =
         𝑝              , nên tất cả các số hạng trong khai triển trên, ngoại trừ số
                  2
hạng cuối cùng, đều chia hết cho 𝑝 𝛼+𝛽+2 . Từ đó ta có ngay 𝑝 𝛼+𝛽+1 ‖ 𝑎 𝑘 − 1. Theo
nguyên lý quy nạp ta có điều phải chứng minh.
     Tương tự như ví dụ trên, ta có bài toán với 𝑝 = 2. Trong trường hợp này do
  2
 𝐶 𝑝 = 1 nên bài toán có sự thay đổi một chút.
Ví dụ 4. Cho 𝑎, 𝑘 là các số nguyên dương, 𝛼 ∈ N+ sao cho 2 𝛼 ‖ 𝑎2 − 1, khi đó
với mọi số nguyên 𝛽 ≥ 0 thì 2 𝛼+𝛽 ‖ 𝑎 𝑘 − 1 ⇔ 2 𝛽+1 ‖ 𝑘.

                                         3
www.VNMATH.com



                                          Lời giải.

   Rõ ràng ta chỉ cần xét với 𝑘 là số nguyên dương chẵn. Ta chứng minh bài
toán bằng quy nạp theo 𝛽 ≥ 0. Với 𝛽 = 0 ⇔ 𝑘 = 2𝑛 (𝑛 là số lẻ). Khi đó
                                  𝑛−1
                       𝑎 𝑘 − 1 ∑︁ 2(𝑛−1−𝑖)
                              =     𝑎      ≡ 𝑛 ̸≡ 0 (mod 2 𝛼 ) .
                       𝑎2 − 1   𝑖=0


Bài toán đúng với 𝛽 = 0. Giả sử bài toán đúng với 𝛽 ≥ 0, đặt 𝑘 = 2𝑛 với 2 𝛽+1 ‖ 𝑛,
khi đó
                          𝑎𝑘 −1
                                 = 𝑎 𝑛 + 1 ≡ 2 (mod 4)
                          𝑎𝑛−1
(do 𝑎 lẻ và 𝑛 chẵn). Mà 2 𝛼+𝛽 ‖ 𝑎 𝑛 − 1. Suy ra bài toán đúng đến 𝛽 + 1. Ta có
điều phải chứng minh.
Chú ý: Hai ví dụ trên có thể tổng quát hơn, đó là định lý về số mũ đúng như
dưới đây, cách chứng minh hoàn toàn tương tự bằng quy nạp.
Định lý 4. (Lifting the Exponent Lemma) i. Với 𝑥, 𝑦 là các số nguyên (không
nhất thiết dương), 𝑛 là số nguyên dương và 𝑝 là số nguyên tố lẻ sao cho 𝑝 | 𝑥 − 𝑦
và 𝑥, 𝑦 không chia hết cho 𝑝. Khi đó

                           𝑣 𝑝 (𝑥 𝑛 − 𝑦 𝑛 ) = 𝑣 𝑝 (𝑥 − 𝑦) + 𝑣 𝑝 (𝑛) .

   ii. Với 𝑥, 𝑦 là hai số nguyên lẻ và 𝑛 là số nguyên dương chẵn. Khi đó

                 𝑣2 (𝑥 𝑛 − 𝑦 𝑛 ) = 𝑣2 (𝑥 − 𝑦) + 𝑣2 (𝑥 + 𝑦) + 𝑣2 (𝑛) − 1.

Nhận xét: +, Nếu 𝑛 lẻ bằng cách thay 𝑦 bởi −𝑦 thì ta cũng có đẳng thức tương
tự trong phần i, 𝑣 𝑝 (𝑥 𝑛 + 𝑦 𝑛 ) = 𝑣 𝑝 (𝑥 + 𝑦) + 𝑣 𝑝 (𝑛) ;
    +, Trong phần ii, do 𝑥, 𝑦 lẻ nên một trong hai số 𝑣2 (𝑥 − 𝑦) , 𝑣2 (𝑥 + 𝑦) bằng
1.
Ví dụ 5. (Nga 1996) Các số nguyên dương 𝑎, 𝑏, 𝑝, 𝑛, 𝑘 thỏa mãn 𝑎 𝑛 + 𝑏 𝑛 = 𝑝 𝑘 .
Chứng minh rằng nếu 𝑛 > 1 là số lẻ và 𝑝 là số nguyên tố lẻ thì 𝑛 là lũy thừa của
 𝑝.

                                          Lời giải.

     Ta có 𝑝 𝑘 = (𝑎 + 𝑏) (𝑎 𝑛−1 − 𝑎 𝑛−2 𝑏 + · · · + 𝑏 𝑛−1 ) suy ra 𝑎 + 𝑏 = 𝑝 𝑗 , 𝑗 ≥ 1. Giả sử
𝑝 𝑙 ‖ 𝑛 mà 𝑎 ≡ −𝑏 (mod 𝑝). Nên theo định lý 4 ta có

                       𝑣 𝑝 (𝑎 𝑛 + 𝑏 𝑛 ) = 𝑣 𝑝 (𝑎 + 𝑏) + 𝑣 𝑝 (𝑛) = 𝑗 + 𝑙,

tức là 𝑝 𝑙+𝑗 ‖ (𝑎 𝑛 + 𝑏 𝑛 ) = 𝑝 𝑘 ⇒ 𝑙 = 𝑘 − 𝑗. Hơn nữa theo định lý 4 ta lại có
               (︁ 𝑘−𝑗              )︁
                    𝑝        𝑝 𝑘−𝑗
                                      = 𝑣 𝑝 (𝑎 + 𝑏) + 𝑣 𝑝 𝑝 𝑘−𝑗 = 𝑗 + 𝑘 − 𝑗 = 𝑘,
                                                         (︀    )︀
             𝑣𝑝 𝑎       + 𝑏


                                              4
www.VNMATH.com



               𝑘−𝑗     𝑘−𝑗       𝑘−𝑗      𝑘−𝑗
suy ra 𝑝 𝑘 ‖ 𝑎 𝑝 + 𝑏 𝑝         và 𝑎 𝑝 + 𝑏 𝑝   ‖ 𝑎 𝑛 + 𝑏 𝑛 (vì 𝑛 lẻ và 𝑝 𝑘−𝑗 ‖ 𝑛). Mà
                                    𝑘−𝑗   𝑘−𝑗
 𝑎 𝑛 + 𝑏 𝑛 = 𝑝 𝑘 nên ta có 𝑝 𝑘 = 𝑎 𝑝 + 𝑏 𝑝 = 𝑎 𝑛 + 𝑏 𝑛 và từ đó 𝑛 = 𝑝 𝑘−𝑗 .
3. Định lý Fermat (nhỏ)
Định lý 5. (Định lý Fermat) Cho 𝑝 là một số nguyên tố thỏa mãn 𝑝          𝑎. Khi đó

                                 𝑎 𝑝−1 ≡ 1 (mod 𝑝) .

Đối với số nguyên 𝑎 bất kỳ, ta có 𝑎 𝑝 ≡ 𝑎 (mod 𝑝) .
Ví dụ 6. Giả sử số nguyên tố 𝑝 có dạng 4𝑘 + 3 và là ước của (𝑥2 + 𝑦 2 ) thì
𝑝 | 𝑥, 𝑝 | 𝑦.

                                       Lời giải.

    Giả sử trái lại 𝑝 𝑥, 𝑝 𝑦 ⇒ gcd (𝑥, 𝑝) = gcd (𝑦, 𝑝) = 1. Do đó theo định
lý Fermat ta có 𝑥 𝑝−1 ≡ 𝑦 𝑝−1 ≡ 1 (mod 𝑝). Trong khi đó từ giả thiết ta có
                              𝑝−1      𝑝−1
 𝑥2 ≡ −𝑦 2 (mod 𝑝) ⇒ (𝑥2 ) 2 ≡ (−𝑦 2 ) 2 (mod 𝑝). Hay 𝑥 𝑝−1 ≡ −𝑦 𝑝−1 (mod 𝑝) ⇔
1 ≡ −1 (mod 𝑝). Ta có mâu thuẫn vì 𝑝 là số nguyên tố lẻ.
Nhận xét: Từ ví dụ này ta suy ra ngay với 𝑛 là số nguyên dương mọi ước nguyên
tố lẻ của 𝑛2 + 1 đều có dạng 4𝑘 + 1. Và từ đó mọi ước dương của 𝑛2 + 1 đều có
dạng 2 𝑖 (4𝑘 + 1) , 𝑖 = 0, 1.
Ví dụ 7. (CĐT Mỹ 2008) Tìm tất cả các số nguyên dương 𝑛 sao cho 𝑛7 + 7 là
số chính phương.

                                       Lời giải.

   Giả sử tồn tại 𝑚 ∈ N sao cho 𝑛7 + 7 = 𝑚2 . Khi đó ta có

           𝑚2 + 112 = 𝑛7 + 27 = (𝑛 + 2) 𝑛6 − 2𝑛5 + · · · − 25 𝑛 + 26 .
                                       (︀                           )︀


Rõ ràng 𝑚2 + 112 ≡ 1, 2 (mod 4) ⇒ 𝑛7 + 27 ≡ 1, 2 (mod 4). Từ đó dễ dàng thấy
 𝑛 ≡ 1 (mod 4). Mà theo đẳng thức trên ta có (𝑛 + 2) | 𝑚2 + 112 suy ra 𝑚2 + 112
phải có một ước nguyên tố 𝑝 có dạng 4𝑖 + 3. Từ đó 112 ≡ −𝑚2 (mod 𝑝). Nếu
𝑝 ̸= 11 thì theo định lý Fermat ta có 11 𝑝−1 ≡ 𝑚 𝑝−1 ≡ 1 (mod 𝑝) nhưng trong khi
đó                                   )︀ 𝑝−1
                       11 𝑝−1 ≡ −𝑚2 2 = −𝑚 𝑝−1 (mod 𝑝)
                               (︀

mâu thuẫn. Suy ra 𝑝 = 11. Do đó 𝑚 = 11𝑘, 𝑘 ∈ N và 11 | (𝑛 + 2) .
  Hơn nữa từ đó ta lại có

                 𝑛6 − 2𝑛5 + · · · − 25 𝑛 + 26 ≡ 7.26 ≡ 8 (mod 11) .

Suy ra 112 | (𝑛 + 2) ⇒ 𝑛 = 112 ℎ − 2 và ℎ là ước dương của 𝑘 2 + 1. Theo ví
dụ 6 các ước nguyên tố lẻ của 𝑘 2 + 1 chỉ có dạng 4𝑖 + 1, tức là ℎ chỉ có dạng
2 𝑗 (4𝑖 + 1) , 𝑗 = 0, 1. Khi đó 𝑛 = 112 ℎ − 2 ≡ 0; 3 (mod 4). Ta có mâu thuẫn vì
 𝑛 ≡ 1 (mod 4). Do đó không tồn tại 𝑛 thỏa mãn đầu bài.

                                          5
www.VNMATH.com



4. Hàm Euler
Định nghĩa 6. Cho 𝑛 là số nguyên dương. Hàm Euler 𝜙 xác định trên tập các
số nguyên dương như sau: 𝜙 (𝑛) là số các số nguyên dương nhỏ hơn 𝑛 và nguyên
tố cùng nhau với 𝑛. Quy ước 𝜙 (1) = 1.
Tính chất 7. Hàm 𝜙 (𝑛) có tính chất nhân theo nghĩa: Nếu 𝑎, 𝑏 là hai số nguyên
tố cùng nhau thì 𝜙 (𝑎𝑏) = 𝜙 (𝑎) 𝜙 (𝑏) .
Tính chất 8. Giả sử 𝑛 = 𝑝1𝛼1 𝑝2𝛼2 . . . 𝑝 𝑘𝛼 𝑘 là phân tích tiêu chuẩn của 𝑛 > 1. Khi
đó                         (︂         )︂ (︂           )︂    (︂      )︂
                                  1                1             1
                 𝜙 (𝑛) = 𝑛 1 −                1−         ··· 1 −       .
                                  𝑝1               𝑝2            𝑝𝑘
Định lý 9. (Định lý Euler ) Cho 𝑎, 𝑛 là các số nguyên, 𝑛 > 1, (𝑎, 𝑛) = 1. Khi đó

                                   𝑎 𝜙(𝑛) ≡ 1 (mod 𝑛) .

Với 𝑎, 𝑛 là hai số nguyên dương bất kỳ ta có

                                 𝑎 𝑛 ≡ 𝑎 𝑛−𝜙(𝑛) (mod 𝑛) .

Nhận xét: Định lý Fermat là trường hợp riêng của định lý Euler trong trường
hợp 𝑛 là số nguyên tố.
Bổ đề 10. Với 𝑎, 𝑏 là các số nguyên dương 𝑎, 𝑏 thì với 𝑛 đủ lớn ta có

                                  𝑏 𝑛+𝜙(𝑎) ≡ 𝑏 𝑛 (mod 𝑎)
                   {︀                                        }︀
(cụ thể hơn 𝑛 ≥ max 𝑣 𝑝 𝑖 (𝑎) + 1, 𝑝 𝑖 là ước nguyên tố của 𝑎 ).
   Chứng minh của bổ đề này có thể xem trong bài 7 dưới đây.
Ví dụ 8. (IMO 1971) Chứng minh rằng dãy số 2 𝑛 − 3 (𝑛 > 1) chứa một dãy con
vô hạn gồm các số đôi một nguyên tố cùng nhau.

                                           Lời giải.

    Ta xây dựng dãy con bằng quy nạp. Giả sử ta đã xây dựng được dãy các số
 𝑎1 = 2 𝑛1 − 1, 𝑎2 = 2 𝑛2 − 1, ..., 𝑎 𝑘 = 2 𝑛 𝑘 − 1, ở đó 2 = 𝑛1 < 𝑛2 < · · · < 𝑛 𝑘 , mà các
phần tử đôi một nguyên tố cùng nhau. Ta sẽ xây dựng 𝑎 𝑘+1 = 2 𝑛 𝑘+1 − 3 như sau:
    Đặt 𝑠 = 𝑎1 𝑎2 ...𝑎 𝑘 ta có 𝑠 là số lẻ nên theo định lý Euler ta có 𝑠 | 2 𝜙(𝑠) − 1 ⇒
2 𝜙(𝑠) − 1 = 𝑞𝑠, 𝑞 ∈ N. Khi đó ta có 2 𝜙(𝑠)+2 − 3 = 4𝑞𝑠 − 1 là nguyên tố cùng
nhau với 𝑠, nên ta có thể chọn 𝑛 𝑘+1 = 𝜙 (𝑠) + 2. Rõ ràng 𝑛 𝑘+1 > 𝑛 𝑘 và dãy
số 𝑎1 , 𝑎2 , ..., 𝑎 𝑘 , 𝑎 𝑘+1 đôi một nguyên tố cùng nhau, từ đó ta có điều phải chứng
minh.
Ví dụ 9. (Mỹ 1991) Chứng minh rằng với mọi số nguyên 𝑛 ≥ 1, dãy số sau
                                       2     22
                               2, 22 , 22 , 22 , . . . (mod 𝑛)

là dãy hằng số kể từ một lúc nào đó.

                                              6
www.VNMATH.com



                                     Lời giải.
    Ta chứng minh bằng quy nạp theo 𝑛. Với 𝑛 = 1 bài toán hiển nhiên đúng.
Giả sử bài toán đúng đến 𝑘 − 1, (𝑘 ≥ 2).
    +, Nếu 𝑘 chẵn, 𝑘 = 2 𝑎 𝑏 với 𝑏 lẻ. Khi đó theo giả thiết quy nạp thì dãy đã cho
là hằng số từ lúc nào đó mô-đun 𝑏. Mà rõ ràng dãy này là dãy 0 mô-đun 2 𝑎 kể từ
lúc nào đó. Vì thế theo mô-đun 𝑘 nó cũng là dãy hằng số kể từ một lúc nào đó.
    +, Nếu 𝑘 lẻ, theo định lý Euler ta có 2 𝜙(𝑘) ≡ 1 (mod 𝑘). Mà theo giả thiết quy
                      2
nạp dãy 1, 2, 22 , 22 , . . . (dãy các số mũ) là hằng số kể từ một lúc nào đó mô-đun
                         2     22
 𝜙 (𝑘). Vì thế 2, 22 , 22 , 22 , . . . (mod 𝑘) là dãy hằng số kể từ một lúc nào đó.
    Theo nguyên lý quy nạp ta có điều phải chứng minh.
5. Cấp (order) của một số nguyên
Định nghĩa 11. Cho 𝑎, 𝑛 là hai số nguyên dương và 𝑎 là số nguyên bất kỳ thỏa
mãn (𝑎, 𝑛) = 1. Số nguyên dương ℎ nhỏ nhất sao cho 𝑎ℎ ≡ 1 (mod 𝑛) được gọi
là cấp của 𝑎 mô-đun 𝑛, ký hiệu ord 𝑛 (𝑎) .
Bổ đề 12. Cho 𝑚, 𝑛 là các số nguyên dương và 𝑎 là số nguyên khác 1. Khi đó
                                             ⃒              ⃒
                    gcd (𝑎 𝑚 − 1, 𝑎 𝑛 − 1) = ⃒ 𝑎gcd(𝑚,𝑛) − 1⃒ .

Định lý 13. Cho 𝑎, 𝑛 là các số nguyên nguyên tố cùng nhau với 𝑛 > 0 khi đó
ord 𝑛 (𝑎) | 𝜙 (𝑛).
Định nghĩa 14. Nếu 𝑟, 𝑛 là các số nguyên tố cùng nhau với 𝑛 > 0 và nếu
ord 𝑛 𝑟 = 𝜙 (𝑛) thì 𝑟 được gọi là căn nguyên thủy mô-đun 𝑛.
Định lý 15. Nếu 𝑟, 𝑛 là các số nguyên dương nguyên tố cùng nhau và nếu 𝑟 là
                                                     {︀           }︀
căn nguyên thủy mô-đun 𝑛. Khi đó các số nguyên 𝑟, 𝑟2 , ..., 𝑟 𝜙(𝑛) tạo thành một
hệ thặng dư thu gọn mô-đun 𝑛.
Định lý 16. (Sự tồn tại của căn nguyên thủy) Mọi số nguyên tố đều có căn
nguyên thủy. Tổng quát hơn: số nguyên dương 𝑛 > 1 có căn nguyên thủy khi
và chỉ khi 𝑛 = 2, 4, 𝑝 𝑘 hoặc 2𝑝 𝑘 trong đó 𝑝 là số nguyên tố lẻ và 𝑘 là số nguyên
dương.
Ví dụ 10. Cho 𝑛 là hai số nguyên dương và 𝑝 là số nguyên tố thỏa. Chứng minh
rằng nếu 𝑚 là ước nguyên tố lẻ của 𝑛 𝑝 + 1 thì 2𝑝 | 𝑚 − 1 hoặc 𝑚 | 𝑛2 − 1.
                                     Lời giải.
    Từ giả thiết 𝑚 | 𝑛 𝑝 + 1 ⇒ 𝑛 𝑝 ≡ −1 ̸≡ 1 (mod 𝑚) vì 𝑚 > 2. Suy ra 𝑛2𝑝 ≡
1 (mod 𝑚).
    Đặt 𝑑 = ord 𝑚 𝑛 khi đó 𝑑 là ước của 2𝑝 nhưng không là ước của 𝑝. Mà 𝑝 là số
nguyên tố suy ra 𝑑 = 2 hoặc 𝑑 = 2𝑝.
    +, Nếu 𝑑 = 2 suy ra 𝑛2 ≡ 1 (mod 𝑚) hay 𝑚 | 𝑛2 − 1.
    +, Nếu 𝑑 = 2𝑝 ⇒ 2𝑝 | 𝑚 − 1 vì theo định lý Fermat ta có 𝑑 | 𝑚 − 1 (đpcm).
Ví dụ 11. (Dự tuyển IMO 2006) Tìm tất cả các nghiệm nguyên của phương
trình
                                 𝑥7 − 1
                                        = 𝑦 5 − 1.
                                  𝑥−1

                                         7
www.VNMATH.com



                                      Lời giải.

                                               𝑥7 − 1
   Gọi 𝑝 là một ước nguyên tố bất kỳ của              = 𝑥6 + 𝑥5 + · · · + 𝑥 + 1. Ta có
                                                𝑥−1
hai trường hợp:
                                   𝑥7 − 1
   +, Nếu 𝑝 | (𝑥 − 1), suy ra             ≡ 1 + 1 + · · · + 1 + 1 ≡ 7 (mod 𝑝) ⇒ 𝑝 = 7;
                                    𝑥−1
   +, Nếu 𝑝 (𝑥 − 1), ta có ngay ord 𝑝 𝑥 = 7. Từ đó suy ra 𝑝 ≡ 1 (mod 7) .
                                                  𝑥7 − 1
   Vậy ta có mọi ước nguyên dương 𝑑 của                    thỏa mãn 𝑑 ≡ 0; 1 (mod 7).
                                                   𝑥−1
Bây giờ giả sử (𝑥, 𝑦) là một nghiệm của phương trình đã cho. Mà 𝑦 5 − 1 =
(𝑦 − 1) (𝑦 4 + 𝑦 3 + 𝑦 2 + 𝑦 + 1) suy ra

                                  𝑦 − 1 ≡ 0; 1 (mod 7)
                       4    3     2
                      𝑦 + 𝑦 + 𝑦 + 𝑦 + 1 ≡ 0; 1 (mod 7) .

Ta có mâu thuẫn vì khi đó 𝑦 ≡ 1; 2 (mod 7) ⇒ 𝑦 4 + 𝑦 3 + 𝑦 2 + 𝑦 + 1 ≡ 5; 3 (mod 7).
Vậy bài toán vô nghiệm.
Ví dụ 12. (CĐT Trung Quốc 2006) Tìm tất cả các cặp số nguyên (𝑎, 𝑛) sao cho
(𝑎 + 1) 𝑛 − 𝑎 𝑛
                là số nguyên.
       𝑛
                                      Lời giải.

     Xét 𝑛 ≥ 2, giả sử 𝑝 là ước nguyên tố nhỏ nhất của 𝑛. Khi đó 𝑝 | (𝑎 + 1) 𝑛 −𝑎 𝑛 ⇒
(𝑝, 𝑛) = 1 nên tồn tại 𝑏 sao cho 𝑎𝑏 ≡ 1 (mod 𝑝). Ta có (𝑎 + 1) 𝑛 ≡ 𝑎 𝑛 (mod 𝑝) ⇒
((𝑎 + 1) 𝑏) 𝑛 ≡ 1 (mod 𝑝). Do đó 𝑑 | 𝑛 với 𝑑 = ord 𝑝 (𝑎𝑏 + 𝑏). Hơn nữa theo định lý
Fermat ta có ((𝑎 + 1) 𝑏) 𝑝−1 ≡ 1 (mod 𝑝) ⇒ 𝑑 | 𝑝 − 1 ⇒ gcd (𝑑, 𝑛) = 1 (theo định
nghĩa của 𝑝). Tức là 𝑑 = 1, suy ra 𝑎 + 1 ≡ 𝑎 (mod 𝑝). Mâu thuẫn, do đó 𝑛 = 1.
     Chú ý : Ta có thể trình bày lời giải bằng căn nguyên thủy như sau: Gọi 𝑔 là
căn nguyên thủy mô-đun 𝑝. Theo giả thiết ta dễ thấy rằng 𝑎, 𝑎 + 1 không chia
hết cho 𝑝, suy ra (𝑎 + 1, 𝑝) = (𝑎, 𝑝) = 1 ⇒ 𝑎 + 1 ≡ 𝑔 𝑘 , 𝑎 ≡ 𝑔 ℎ (mod 𝑝) với 𝑘 ̸= ℎ.
Thay vào phương trình đã cho suy ra 𝑔 𝑘𝑛 ≡ 𝑔 ℎ𝑛 (mod 𝑝) ⇒ 𝑝 − 1 | 𝑛 (𝑘 − ℎ). Mà
 𝑝 là ước nguyên tố nhỏ nhất của 𝑛 nên (𝑝 − 1, 𝑛) = 1 ⇒ 𝑝 − 1 | 𝑘 − ℎ. Theo định
lý Fermat suy ra 𝑝 | 𝑔 𝑝−1 − 1 | 𝑔 𝑘 − 𝑔 ℎ ⇒ 𝑝 = 1 mâu thuẫn.
6. Một số hệ quả hay dùng khác
i. Nếu 𝑝 | 𝑥 thì (𝑥 + 𝑦) 𝑛 ≡ 𝑛𝑥𝑦 𝑛−1 + 𝑦 𝑛 (mod 𝑝2 ) và nếu 𝑎 ≡ 𝑏 (mod 𝑝) thì
 𝑎 = 𝑘𝑝 + 𝑏 ta có 𝑎 𝑛 ≡ 𝑛𝑘𝑝𝑏 𝑛−1 + 𝑏 𝑛 (mod 𝑝2 ) .
ii . Cho 𝑚 là số nguyên dương và 𝑎, 𝑏 là hai số nguyên tố cùng nhau với 𝑚. Nếu
 𝑥, 𝑦 là hai số nguyên thỏa mãn 𝑎 𝑥 ≡ 𝑏 𝑥 (mod 𝑚) và 𝑎 𝑦 ≡ 𝑏 𝑦 (mod 𝑚). Khi đó
 𝑎gcd(𝑥,𝑦) ≡ 𝑏gcd(𝑥,𝑦) (mod 𝑚).
iii . Cho 𝑝 là một số nguyên tố lẻ. Khi đó
     a) nếu 𝑎 ≥ 2 thì 𝑎 𝑝 − 1 có một ước nguyên tố mà không là ước của 𝑎 − 1;
     b) nếu 𝑎 ≥ 2, 𝑝 ̸= 3 hoặc 𝑎 > 2 thì 𝑎 𝑝 + 1 có một ước nguyên tố mà không là
ước của 𝑎 + 1.

                                          8
www.VNMATH.com



     Hướng dẫn. Giả sử trái lại suy ra mọi ước của 𝐴 = 𝑎 𝑝−1 + 𝑎 𝑝−2 + · · · + 𝑎 + 1 đều
là ước của 𝑎 − 1 mà 𝐴 = (𝑎 − 1) 𝐵 + 𝑝 ⇒ gcd (𝑎 − 1, 𝑎 𝑝−1 + 𝑎 𝑝−2 + · · · + 𝑎 + 1) |
 𝑝 ⇒ 𝐴 là lũy thừa của 𝑝. Do đó 𝑝2 ‖ 𝑎 𝑝 − 1 ⇒ 𝐴 = 𝑝 − 1 mâu thuẫn.
iv. Nếu 𝑎 là số nguyên không chia hết cho số nguyên tố 𝑝 và có một số nguyên
dương 𝑘 thỏa mãn 𝑎 𝑘 ≡ −1 (mod 𝑝) khi đó nếu 𝑑 = ord 𝑝 𝑎 thì ℎ = 𝑑/2 là số
nguyên dương nhỏ nhất thỏa mãn 𝑎ℎ ≡ −1 (mod 𝑝) .
                   𝑎2𝑘                            𝑘
(︀ ℎ Hướng dẫn. )︀ ≡ 1 (mod 𝑝) ⇒ 𝑑 | 2𝑘 mà 𝑎 ≡ −1 (mod 𝑝) nên 𝑑 = 2ℎ ⇒
          )︀ (︀ ℎ
   𝑎 − 1 𝑎 + 1 ≡ 0 (mod 𝑝) .
                                                                              𝑥𝑝 −1
v. Nếu 𝑥 là số nguyên dương và 𝑝, 𝑞 là hai số nguyên tố sao cho 𝑞 |                   thì
                                                                              𝑥−1
 𝑞 = 𝑝 hoặc 𝑞 ≡ 1 (mod 𝑝) .
                                                       (︀        )︀              (︀ )︀
vi. Số nguyên dương 𝑑 nhỏ nhất thỏa mãn 2 𝑑 ≡ 1 mod 3 𝑘 là 𝑑 = 𝜙 3 𝑘 =
2.3 𝑘−1 . Nói cách khác 2 là căn nguyên thủy mô-đun 3 𝑛 .
     Hướng dẫn. Nếu 𝑝 là số nguyên tố lẻ và 𝑟 là căn nguyên thủy mô-đun 𝑝2 thì
 𝑟 là căn nguyên thủy mô-đun 𝑝 𝑘 với mọi số nguyên dương 𝑘.

II. Bài tập áp dụng
Bài 1. (IMO 2005) Xét dãy số 𝑎1 , 𝑎2 , ... xác định bởi công thức
                        𝑎 𝑛 = 2 𝑛 + 3 𝑛 + 6 𝑛 − 1 (𝑛 = 1, 2, ...) .
Xác định tất cả các số nguyên dương mà chúng nguyên tố cùng nhau với mọi số
hạng của dãy trên.
                                        Lời giải.
   Ta sẽ chứng minh với mọi số nguyên tố 𝑝 luôn tồn tại một số hạng 𝑎              𝑚   của
dãy sao cho 𝑝 | 𝑎 𝑚 , từ đó suy ra tất cả các số cần tìm là 1.
   +, Với 𝑝 = 2, 3 rõ ràng 𝑎2 = 48 từ đó 𝑝 | 𝑎2 .
   +, Xét 𝑝 > 3. Áp dụng định lý Fermat ta có
        6𝑎 𝑝−2 = 3.2 𝑝−1 + 2.3 𝑝−1 + 6 𝑝−1 − 6 ≡ 3 + 2 + 1 − 6 ≡ 0 (mod 𝑝) .
Từ đó 𝑝 | 𝑎 𝑝−2 ⇒ gcd (𝑝, 𝑎 𝑝−2 ) = 𝑝 > 1. Vậy chỉ có số 1 thỏa mãn bài toán.
Bài 2. (Dự tuyển IMO 2005) Giả sử 𝑎, 𝑏 là hai số nguyên dương sao cho 𝑎 𝑛 + 𝑛
là ước của 𝑏 𝑛 + 𝑛 với mọi số nguyên dương 𝑛. Chứng minh rằng 𝑎 = 𝑏.
                                        Lời giải.
   Giả sử 𝑎 ̸= 𝑏, khi đó từ giả thiết dễ thấy 𝑏 > 𝑎. Chọn 𝑝 là số nguyên tố lớn
hơn 𝑏 và lấy 𝑛 = (𝑎 + 1) (𝑝 − 1) + 1. Theo cách chọn này ta có 𝑛 ≡ 1 (mod 𝑝 − 1)
và 𝑛 ≡ −𝑎 (mod 𝑝). Khi đó theo định lý Fermat ta có
                                    )︀ 𝑎+1
                      𝑟 𝑛 ≡ 𝑟 𝑟 𝑝−1
                             (︀
                                           ≡ 𝑟 (mod 𝑝) ∀𝑟 ∈ Z.
     Ta lại có 𝑎 𝑛 + 𝑛 ≡ 𝑎 − 𝑎 ≡ 0 (mod 𝑝) ⇒ 𝑝 | 𝑎 𝑛 + 𝑛. Hơn nữa 𝑏 𝑛 + 𝑛 ≡ 𝑏 − 𝑎 ≡
0 (mod 𝑝) ⇒ 𝑝 | 𝑏 − 𝑎. Điều này mâu thuẫn vì 𝑝 > 𝑏. Do đó 𝑎 = 𝑏 thỏa mãn.
Bài 3. (Bulgaria 1995) Tìm tất cả các số nguyên tố 𝑝, 𝑞 sao cho 𝑝𝑞 là ước của
(5 𝑝 − 2 𝑝 ) (5 𝑞 − 2 𝑞 ).

                                            9
www.VNMATH.com



                                       Lời giải.

    Do tính đối xứng nên ta có thể giả sử 𝑝 ≤ 𝑞 mà (5 𝑝 − 2 𝑝 ) (5 𝑞 − 2 𝑞 ) là số lẻ nên
ta có 5 ≤ 𝑝 ≤ 𝑞. Để ý rằng nếu số nguyên tố 𝑘 là ước của 5 𝑘 − 2 𝑘 thì theo định
lý Fermat ta có ngay 3 ≡ 5 − 2 ≡ 5 𝑘 − 2 𝑘 (mod 𝑘) ⇒ 𝑘 = 3.
    Giả sử 𝑝 > 3, theo nhận xét trên ta có 𝑝 là ước của 5 𝑞 −2 𝑞 hay 5 𝑞 ≡ 2 𝑞 (mod 𝑝).
Lại theo định lý Fermat thì 5 𝑝−1 ≡ 2 𝑝−1 (mod 𝑝). Do đó

                           5gcd(𝑝−1,𝑞) ≡ 2gcd(𝑝−1,𝑞) (mod 𝑝) .

Mà 𝑞 ≥ 𝑝 ⇒ gcd (𝑝 − 1, 𝑞) = 1 do đó ta có 5 ≡ 2 (mod 𝑝) ⇒ 𝑝 = 3 mâu thuẫn.
    Suy ra 𝑝 = 3. Nếu 𝑞 > 3 suy ra 𝑞 là ước của 5 𝑝 − 2 𝑝 = 53 − 23 = 9.13 ⇒ 𝑞 = 13
thỏa mãn.
    Vậy tất cả các cặp (𝑝, 𝑞) cần tìm là: (3, 3) ; (3, 13) ; (13, 3).
Bài 4. (Dự tuyển IMO 2003) Cho 𝑝 là một số nguyên tố. Chứng minh rằng tồn
tại một số nguyên tố 𝑞 sao cho với mọi số nguyên 𝑛, số 𝑛 𝑝 − 𝑝 không chia hết cho
 𝑞.

                                       Lời giải.

   Ta có
                  𝑝𝑝 −1
                        = 1 + 𝑝 + 𝑝2 + · · · 𝑝 𝑝−1 ≡ 𝑝 + 1 mod 𝑝2 ,
                                                           (︀    )︀
                  𝑝−1
                                             𝑝𝑝 −1
suy ra có ít nhất một ước nguyên tố của             không đồng dư 1 mô-đun 𝑝2 . Gọi
                                             𝑝−1
số nguyên tố này là 𝑞 và ta sẽ chỉ ra đây là số 𝑞 cần tìm.
     Thật vậy, giả sử tồn tại số nguyên 𝑛 sao cho 𝑛 𝑝 ≡ 𝑝 (mod 𝑞). Khi đó, theo
                           2
cách chọn số 𝑞 ta có 𝑛 𝑝 ≡ 𝑝 𝑝 ≡ 1 (mod 𝑞). Mặt khác, theo định lý Fermat,
 𝑛 𝑞−1 ≡ 1 (mod 𝑞), vì 𝑞 là số nguyên tố. Hơn nữa ta có 𝑝2 𝑞 − 1 nên (𝑝2 , 𝑞 − 1) |
 𝑝 ⇒ 𝑛 𝑝 ≡ 1 (mod 𝑞). Suy ra 𝑝 ≡ 1 (mod 𝑞). Khi đó ta thu được

                          1 + 𝑝 + · · · + 𝑝 𝑝−1 ≡ 𝑝 (mod 𝑞) .

Cùng với định nghĩa của 𝑞 ta có ngay 𝑝 ≡ 0 (mod 𝑞), đây là điều mâu thuẫn. Ta
có điều phải chứng minh.
Nhận xét: Để dùng định lý Fermat, ta sẽ tìm cách chọn số 𝑞 có dạng 𝑝𝑘 + 1. Khi
đó

                        ∃𝑛 𝑛 𝑝 ≡      𝑝 (mod 𝑞) ⇔ 𝑝 𝑘 ≡ 1 (mod 𝑞) ,
                 tức là ∀𝑛 𝑛 𝑝 ̸≡     𝑝 (mod 𝑞) ⇔ 𝑝 𝑘 ̸≡ 1 (mod 𝑞) .

Do đó để tìm 𝑞 ta sẽ chọn 𝑞 là một ước nguyên tố của 𝑝 𝑝 − 1 và ta có cách chọn
như trên.
Bài 5. (Dự tuyển IMO 2005) Tìm tất cả các số nguyên dương 𝑛 > 1 sao cho tồn
tại duy nhất số nguyên 𝑎 với 0 < 𝑎 ≤ 𝑛! thỏa mãn 𝑎 𝑛 + 1 chia hết cho 𝑛!.

                                           10
www.VNMATH.com



                                      Lời giải.
   Ta sẽ chỉ ra rằng tất cả các số 𝑛 cần tìm là các số nguyên tố. Thật vậy:
   +, Với 𝑛 = 2 khi đó rõ ràng chỉ có duy nhất 𝑎 = 1 thỏa mãn yêu cầu.
   +, Với 𝑛 > 2 và 𝑛 chẵn ta có 4 | 𝑛! nhưng 𝑎 𝑛 + 1 ≡ 1, 2 (mod 4) , khi đó không
tồn tại 𝑎 thỏa mãn.
   +, Xét 𝑛 lẻ. Ta có
                     (𝑛! − 1) 𝑛 + 1 ≡ (−1) 𝑛 + 1 ≡ 0 (mod 𝑛!) .                   (*)
Suy ra nếu 𝑛 là hợp số và 𝑑 là một ước nguyên tố của 𝑛 thì ta có
                  (︂      )︂ 𝑛       𝑛
                                   ∑︁ (𝑛!) 𝑘
                     𝑛!
                        −1 +1=          𝐶 𝑛𝑘 𝑘 ≡ 0 (mod 𝑛!)
                     𝑑              𝑘=1
                                             𝑑

bởi vì rõ ràng 𝑑2 | 𝑛! nên mỗi số hạng của tổng trên đều chia hết cho 𝑛!. Do vậy
trường hợp này cũng không thỏa mãn.
    +, Xét 𝑛 là số nguyên tố lẻ và giả sử 𝑎 là số nguyên thỏa mãn 0 < 𝑎 ≤ 𝑛!, 𝑎 𝑛 +1
chia hết cho 𝑛!. Do 𝑛 lẻ nên ta có khai triển
                 𝑎 𝑛 + 1 = (𝑎 + 1) 𝑎 𝑛−1 − 𝑎 𝑛−2 + · · · + 𝑎2 − 𝑎 + 1 .
                                  (︀                                 )︀

Ta sẽ chứng minh 𝑎 = 𝑛! − 1 là giá trị duy nhất thỏa mãn.
    Xét 𝑝 là số nguyên tố, 𝑝 ≤ 𝑛. Ta có 𝑝 | 𝑎 𝑛 + 1 ⇒ 𝑝 | (−𝑎) 𝑛 − 1. Theo định lý
Fermat ta lại có 𝑝 | (−𝑎) 𝑝−1 − 1 nên 𝑝 | (−𝑎)(𝑛,𝑝−1) − 1 = −𝑎 − 1. Suy ra 𝑛 | 𝑎 + 1
(𝑝 = 𝑛) và nếu 𝑝 < 𝑛 thì 𝑎 𝑛−1 − 𝑎 𝑛−2 + · · · + 𝑎2 − 𝑎 + 1 ≡ 𝑛 (mod 𝑝) ̸≡ 0 (mod 𝑝)
do (𝑛, 𝑝) = 1. Từ đó ta thu được 𝑎 𝑛−1 − 𝑎 𝑛−2 + · · · + 𝑎2 − 𝑎 + 1 và (𝑛 − 1)! nguyên
tố cùng nhau, suy ra (𝑛 − 1)! | 𝑎 + 1. Mà 𝑛 | 𝑎 + 1 nên 𝑛! | 𝑎 + 1. Do điều kiện
0 < 𝑎 ≤ 𝑛! ⇒ 𝑛! = 𝑎 + 1 ⇒ 𝑎 = 𝑛! − 1. Cùng với (*) ta có 𝑎 = 𝑛! − 1 là giá trị
duy nhất thỏa mãn.
    Vậy tất các số nguyên dương 𝑛 cần tìm là tất cả các số nguyên tố.
Bài 6. (Dự tuyển IMO 2006) Chứng minh rằng với mọi số nguyên dương 𝑛 luôn
tồn tại số nguyên 𝑚 sao cho 2 𝑚 + 𝑚 chia hết cho 𝑛.
                                      Lời giải.
    Ta sẽ chứng minh bằng quy nạp theo 𝑛 rằng luôn tồn tại số nguyên dương 𝑚
đủ lớn để 2 𝑚 ≡ −𝑚 (mod 𝑛) .
    +, Với 𝑛 = 1 hiển nhiên đúng.
    +, Xét 𝑛 > 1. Theo tính chất của hàm Euler ta có dãy các số mũ của 2 theo
mô-đun 𝑛 là tuần hoàn với chu kỳ là bội của 𝜙 (𝑛). Do đó với 𝑥, 𝑦 đủ lớn và
 𝑥 ≡ 𝑦 (mod 𝜙 (𝑛)) thì 2 𝑥 ≡ 2 𝑦 (mod 𝑛).
    Chọn số 𝑚 có dạng 𝑚 ≡ −2 𝑘 (mod 𝑛𝜙 (𝑛)). Khi đó 2 𝑚 ≡ −𝑚 (mod 𝑛) ⇔
2 𝑚 ≡ 2 𝑘 (mod 𝑛). Theo giả thiết quy nạp ta có thể chọn được số 𝑘 đủ lớn
để 2 𝑘 ≡ −𝑘 (mod 𝜙 (𝑛)) ⇒ −2 𝑘 ≡ 𝑚 ≡ 𝑘 (mod 𝜙 (𝑛)). Từ đó ta thu được
2 𝑚 ≡ −𝑚 (mod 𝑛). Theo nguyên lý quy nạp ta có điều phải chứng minh.
Bài 7. (CĐT Mỹ 2007) Hỏi có tồn tại hay không hai số nguyên dương 𝑎, 𝑏 sao
cho 𝑎 không là ước của 𝑏 𝑛 − 𝑛 với mọi số nguyên dương 𝑛.

                                         11
www.VNMATH.com



                                              Lời giải.

    Trước hết ta chứng minh rằng với mọi số nguyên dương 𝑎, 𝑏 thì với 𝑛 đủ lớn
ta có
                            𝑏 𝑛+𝜙(𝑎) ≡ 𝑏 𝑛 (mod 𝑎) .                       (*)
Thật vậy, giả sử 𝑎 = 𝑝1𝛼1 𝑝2𝛼2 . . . 𝑝 𝑘𝛼 𝑘 , trong đó 𝑝1 , 𝑝2 , ..., 𝑝 𝑘 là các số nguyên tố phân
biệt. Vì 𝜙 là hàm nhân tính nên ta có

 𝜙 (𝑎) = 𝜙 (𝑝1𝛼1 ) 𝜙 (𝑝2𝛼2 ) . . . 𝜙 (𝑝 𝑘𝛼 𝑘 ) = 𝑝1𝛼1 − 𝑝1𝛼1 −1 𝑝2𝛼2 − 𝑝2𝛼2 −1 . . . 𝑝 𝑘𝛼 𝑘 − 𝑝 𝑘𝛼 𝑘 −1
                                                  (︀             )︀ (︀             )︀ (︀                )︀
             (︂       )︂ (︂           )︂       (︂        )︂
                   1               1                  1
       = 𝑎 1−               1−            ··· 1 −           .
                   𝑝1              𝑝2                 𝑝𝑘

Khi đó ta có 𝜙 (𝑝 𝑖𝛼 𝑖 ) | 𝜙 (𝑎) và 𝜙 (𝑎) < 𝑎. Với mỗi 𝑝 𝑖 , ta có hai trường hợp:
       +, Nếu 𝑝 𝑖 là ước của 𝑏 ⇒ 𝑏 𝑛 ≡ 0 (mod 𝑝 𝑖𝛼 𝑖 ) với 𝑛 ≥ 𝛼 𝑖 + 1. Suy ra 𝑏 𝑛+𝜙(𝑎) ≡
 𝑏 𝑛 𝑏 𝜙(𝑎) ≡ 𝑏 𝑛 ≡ 0 (mod 𝑝 𝑖𝛼 𝑖 ) với 𝑛 ≥ 𝛼 𝑖 + 1.
       +, Nếu 𝑝 𝑖 không là ước của 𝑏 ⇒ gcd (𝑝 𝑖𝛼 𝑖 , 𝑏) = 1. Theo định lý Euler thì
         𝛼𝑖
 𝑏 𝜙( 𝑝 𝑖 ) ≡ 1 (mod 𝑝 𝑖𝛼 𝑖 ), mà 𝜙 (𝑝 𝑖𝛼 𝑖 ) | 𝜙 (𝑎) ⇒ 𝑏 𝑛+𝜙(𝑎) ≡ 𝑏 𝑛 (mod 𝑝 𝑖𝛼 𝑖 ) .
       Tức là với mỗi 𝑝 𝑖 ta luôn chọn được 𝑛 𝑖 sao cho với mọi 𝑛 > 𝑛 𝑖 thì 𝑏 𝑛+𝜙(𝑎) ≡
 𝑏 𝑛 (mod 𝑝 𝑖𝛼 𝑖 ). Do đó nếu ta chọn 𝑁 = max {𝑛 𝑖 } khi đó với mọi 𝑛 > 𝑁 , ta có
 𝑏 𝑛+𝜙(𝑎) ≡ 𝑏 𝑛 (mod 𝑝 𝑖𝛼 𝑖 ) với mọi 1 ≤ 𝑖 ≤ 𝑘. Bởi vì các 𝑝 𝑖 là các số nguyên tố phân
biệt nên 𝑏 𝑛+𝜙(𝑎) ≡ 𝑏 𝑛 (mod 𝑎) với mọi 𝑛 > 𝑁 (đpcm).
       Trở lại bài toán, ta sẽ chỉ ra bằng quy nạp theo 𝑎 rằng với mọi số nguyên
dương 𝑎, 𝑏 thì luôn tồn tại vô hạn số nguyên dương 𝑛 sao cho 𝑎 là ước của 𝑏 𝑛 − 𝑛
(**).
       +, Với 𝑎 = 1 rõ ràng khẳng định (**) đúng.
       +, Giả sử (*) đúng với mọi số nguyên 1 ≤ 𝑎 < 𝑎0 (𝑎0 ≥ 2). Do 𝜙 (𝑎) < 𝑎 nên
theo giả thiết quy nạp và theo (*) suy ra tồn tại vô số số nguyên dương 𝑛 sao cho

                          𝜙 (𝑎) | (𝑏 𝑛 − 𝑛) và 𝑏 𝑛+𝜙(𝑎) ≡ 𝑏 𝑛 (mod 𝑎) .

Với mỗi 𝑛 như vậy, đặt
                                  𝑏𝑛− 𝑛
                             𝑡=          và 𝑛1 = 𝑏 𝑛 = 𝑛 + 𝑡𝜙 (𝑎) .
                                   𝜙 (𝑎)

Thế thì theo định lý Euler

           𝑏 𝑛1 − 𝑛1 ≡ 𝑏 𝑛+𝑡𝜙(𝑎) − 𝑛 − 𝑡𝜙 (𝑎) ≡ 𝑏 𝑛 − 𝑛 − 𝑡𝜙 (𝑎) ≡ 0 (mod 𝑎) ,

tức là 𝑛1 = 𝑏 𝑛 thỏa mãn yêu cầu. Theo giả thiết quy nạp, có vô hạn số 𝑛1 = 𝑏 𝑛
thỏa mãn điều kiện trong khẳng định (**), từ đó (**) đúng. Và do đó không có
các số nguyên dương 𝑎, 𝑏 nào thỏa mãn đầu bài.
Bài 8. (CĐT Mỹ 2003) Tìm tất cả các bộ ba số nguyên tố (𝑝, 𝑞, 𝑟) thỏa mãn

                               𝑝 | 𝑞 𝑟 + 1,   𝑞 | 𝑟 𝑝 + 1,   𝑟 | 𝑝 𝑞 + 1.

                                                  12
www.VNMATH.com



                                      Lời giải.

     Giả sử 𝑝, 𝑞, 𝑟 thỏa mãn đầu bài suy ra chúng là ba số nguyên tố phân biệt.
     Trường hợp 1 : Cả ba số 𝑝, 𝑞, 𝑟 đều lẻ. Ta có 𝑝 | 𝑞 𝑟 + 1 nên theo ví dụ 10 ta có
2𝑟 | 𝑝 − 1 hoặc 𝑝 | 𝑞 2 − 1.
     +, Nếu 2𝑟 | 𝑝 − 1 ⇒ 𝑝 ≡ 1 (mod 𝑟) ⇒ 0 ≡ 𝑝 𝑞 + 1 ≡ 2 (mod 𝑟) mà 𝑟 > 2 nên
mâu thuẫn với đầu bài.
     +, Nếu 𝑝 | 𝑞 2 − 1 = (𝑞 − 1) (𝑞 + 1). Hơn nữa do 𝑝 lẻ và 𝑝 − 1, 𝑝 + 1 là các số
chẵn nên ta có
                                   𝑞−1              𝑞+1
                               𝑝|        hoặc 𝑝 |       .
                                     2               2
                           𝑞+1
Khi đó ta luôn có 𝑝 ≤            < 𝑞. Bằng cách chứng minh tương tự ta cũng có
                             2
 𝑞 < 𝑟, 𝑟 < 𝑝. Ta cũng có mâu thuẫn nên trường hợp này không xảy ra được.
     Trường hợp 2 : Một trong ba số 𝑝, 𝑞, 𝑟 phải bằng 2, do tính xoay vòng nên ta
có thể giả sử 𝑝 = 2. Khi đó 𝑟 | 2 𝑞 + 1, lại theo ví dụ 10 ta có 2𝑞 | 𝑟 − 1 hoặc
 𝑟 | 22 − 1.
     Rõ ràng nếu 2𝑞 | 𝑟 − 1 như trường hợp trên ta cũng có 𝑟 ≡ 1 (mod 𝑞) ⇒ 0 ≡
 𝑟 + 1 ≡ 2 (mod 𝑞) ⇒ 𝑞 = 2 ta có mâu thuẫn vì 𝑝, 𝑞, 𝑟 phân biệt. Do đó 𝑟 | 22 − 1
   𝑝

suy ra 𝑟 = 3 và 𝑞 | 𝑟2 + 1 = 10 ⇒ 𝑞 = 5. (thỏa mãn).
     Vậy các bộ cần tìm là các hoán vị xoay vòng của (2, 5, 3).
Bài 9. (Iran 2007) Cho 𝑛 là số nguyên dương và 𝑛 = 22007 𝑘 + 1 với 𝑘 là số nguyên
lẻ. Chứng minh rằng
                                       𝑛 2 𝑛−1 + 1.

                                      Lời giải.
                           (︀ )︀22007
   Giả sử 𝑛 | 2 𝑛−1 + 1 = 2 𝑘         + 1. Gọi 𝑝 là một số ước nguyên tố tùy ý của 𝑛
                                        2007                    2008
và đặt 𝑡 = 2 , 𝑑 = ord 𝑝 (𝑡). Ta có 𝑡2
             𝑘
                                             ≡ −1 (mod 𝑝) ⇒ 𝑡2       ≡ 1 (mod 𝑝). Suy
        2008     2007                2008
ra 𝑑 | 2 , 𝑑 2 , tức là 𝑑 = 2             mà 𝑑 | 𝑝 − 1 ⇒ 𝑝 ≡ 1 (mod 22008 ). Do đó

               𝑛 ≡ 1 mod 22008 ⇒ 22007 𝑘 ≡ 0 mod 22008 ⇒ 2 | 𝑘.
                      (︀          )︀                (︀       )︀


Điều này mâu thuẫn với giả thiết, ta có kết luận của bài toán.
Bài 10. (Việt Nam 2001) Cho 𝑛 là số nguyên dương và 𝑎, 𝑏 là các số nguyên tố
                                                               𝑛    𝑛
cùng nhau lớn hơn 1. Giả sử 𝑝, 𝑞 là hai ước lẻ lớn hơn 1 của 𝑎6 + 𝑏6 . Tìm số dư
                      𝑛    𝑛
trong phép chia của 𝑎6 + 𝑏6 cho 6.12 𝑛 .

                                      Lời giải.

    Trước hết ta chứng minh 𝑝 ≡ 1 ≡ 𝑞 (mod 2 𝑛+1 ). Vì (𝑎, 𝑏) = 1 suy ra (𝑎, 𝑝) =
(𝑏, 𝑝) = 1. Gọi 𝑏′ là nghịch đảo của 𝑏 mô-đun 𝑝 và đặt 𝐴 = 𝑎𝑏′ . Khi đó theo giả
thiết
                                𝑛    𝑛    𝑛          𝑛
                          𝑝 | 𝑎6 + 𝑏6 | 𝐴6 + 1 | 𝐴2.6 − 1.


                                         13
www.VNMATH.com



Suy ra ord 𝑝 (𝐴) | 2.6 𝑛 , ord 𝑝 (𝐴) | 6 𝑛 . Do đó ta suy ra ord 𝑝 (𝐴) = 2 𝑛+1 3 𝑛 mà
ord 𝑝 (𝐴) | 𝑝 − 1 ⇒ 𝑝 ≡ 1 (mod 2 𝑛+1 ). Chứng minh tương tự cho 𝑞. Vậy 𝑝 ≡
                                                     (︁            )︁
                           𝑛         𝑛                                2
𝑞 ≡ 1 (mod 2 𝑛+1 ) ⇒ 𝑝6 ≡ 𝑞 6 ≡ 6 𝑛 .2 𝑛+1 + 1         mod (2 𝑛+1 )            (theo khai triển
                     𝑛         𝑛
Newton). Từ đó 𝑝6 ≡ 𝑞 6 ≡ 1 (mod 22𝑛+1 ). (*)
                                            𝑛                                 𝑛
     Hơn nữa ta có 𝜙 (6 𝑛+1 ) = 2.6 𝑛 ⇒ 𝑝6 ≡ ±1 (mod 6 𝑛+1 ). Giả sử 𝑝6 ≡
                         𝑛                                    𝑛
−1 (mod 6 𝑛+1 ) suy ra 𝑝6 ≡ −1 (mod 2 𝑛+1 ) mà từ (*) ta có 𝑝6 ≡ 1 (mod 2 𝑛+1 ) ⇒
     𝑛                                                           𝑛
2𝑝6 ≡ 0 (mod 2 𝑛+1 ) mâu thuẫn vì 𝑝 lẻ. Do đó ta thu được 𝑝6 ≡ 1 (mod 22𝑛+1 )
       𝑛                       𝑛                                𝑛
và 𝑝6 ≡ 1 (mod 6 𝑛+1 ) từ đó 𝑝6 ≡ 1 (mod 6.12 𝑛 ). Tương tự 𝑞 6 ≡ 1 (mod 6.12 𝑛 ).
Suy ra số dư cần tìm là 2.
Bài 11. (Trung Quốc 2009) Tìm tất cả các cặp số nguyên tố (𝑝, 𝑞) sao cho
 𝑝𝑞 | 5 𝑝 + 5 𝑞 .

                                         Lời giải.

     Rõ ràng cặp (5, 5) thỏa mãn. Xét trường hợp 𝑝 = 5, 𝑞 ̸= 5 ta có 5𝑞 | 55 + 5 𝑞 ⇒
 𝑞 | 625 + 5 𝑞−1 . Theo định lý Fermat suy ra 𝑞 là ước của 626, trong trường hợp
này ta có nghiệm (5, 313) và (5, 2). Tương tự trường hợp 𝑞 = 5, 𝑝 ̸= 5 ta có hai
nghiệm (313, 5) và (2, 5).
     Xét 𝑝 = 2, 𝑞 ̸= 5 ta có 2𝑞 | 25 + 5 𝑞 ⇒ 25 + 5 𝑞 ≡ 0 (mod 2𝑞) và theo định
lý Fermat ta thu được 30 ≡ 0 (mod 2𝑞) từ đó dễ thấy 𝑞 = 3 thỏa mãn. Do đó
trong trường hợp trong hai số 𝑝, 𝑞 có một số bằng 2 và số còn lại khác 5 ta có hai
nghiệm (2, 3) và (3, 2).
     Xét 𝑝, 𝑞 là hai số khác 2 và 5. Ta có 5 𝑝 + 5 𝑞 ≡ 0 (mod 𝑝𝑞) ⇒ 5 + 5 𝑞 ≡
0 (mod 𝑝) ⇒ 5 𝑞−1 ≡ −1 (mod 𝑝) ⇒ 52(𝑞−1) ≡ 1 (mod 𝑝). Tương tự ta có
5 𝑝−1 ≡ −1 (mod 𝑞). Chọn 𝑘 sao cho 2 𝑘 ‖ ord 𝑝 5 mà ord 𝑝 5 | 𝑝 − 1, và ord 𝑝 5 |
2 (𝑞 − 1) ; ord 𝑝 5 (𝑞 − 1). Suy ra 2 𝑘−1 ‖ 𝑞 − 1, tức là số mũ lớn nhất của 2 mà
là ước của 𝑝 − 1 lớn hơn số mũ lớn nhất của 2 mà là ước của 𝑞 − 1. Do tính đối
xứng nên ta cũng có số mũ lớn nhất của 2 mà là ước của 𝑞 − 1 lớn hơn số mũ lớn
nhất của 2 mà là ước của 𝑝 − 1. Đây là điều mâu thuẫn, trong trường hợp này
bài toán vô nghiệm.
     Vậy các nghiệm của bài toán là (2, 3) ; (2, 5) ; (5, 5) ; (5, 313) và các nghiệm đối
xứng của nó là (3, 2) ; (5, 2) ; (313, 5).
Bài 12. (CĐT Romanian 2000) Cho 𝑎 > 1 là số nguyên dương. Tìm số nguyên
dương 𝑛 nhỏ nhất sao cho 22000 | 𝑎 𝑛 − 1.

                                         Lời giải.

    Ta có 𝑎 𝑛 ≡ 1 (mod 22000 ). Theo định lý Euler suy ra 𝑛 | 𝜙 (22000 ) = 21999 ⇒ 𝑛
là lũy thừa của 2. Đặt 𝑛 = 2 𝑘 với 𝑘 ≥ 0. Khi đó ta có
                                                           (︁ 𝑘−1    )︁
              𝑎 𝑛 − 1 = (𝑎 − 1) (𝑎 + 1) 𝑎2 + 1 𝑎4 + 1 · · · 𝑎2 + 1 .
                                       (︀     )︀ (︀  )︀

                                                                   (︁                   )︁
                                                                               𝑘−1
Mà 𝑎2𝑚 + 1 ≡ 2 (mod 4) nên 2 𝑘 − 1 ‖ (𝑎2 + 1) (𝑎4 + 1) · · ·              𝑎2         + 1 . Do đó
ta cần tìm 𝑘 nhỏ nhất để 22001−𝑘 | (𝑎 − 1) (𝑎 + 1) = 𝑎2 − 1. Vì thế nếu đặt

                                            14
www.VNMATH.com



 𝑚 = 𝑣2 (𝑎2 − 1) (tức là số mũ của 2 lớn nhất của 𝑎2 − 1) ta có
                             {︂
                                    0 nếu 𝑚 ≥ 2001
                          𝑘=
                                2001 − 𝑚 nếu 𝑚 < 2001.

Vậy 𝑛 = 2 𝑘 với 𝑘 xác định theo công thức trên.
Bài 13. (CĐT Romanian 2005) Giải phương trình trong tập số nguyên dương
3 𝑥 = 2 𝑥 𝑦 + 1.

                                       Lời giải.

   Bằng quy nạp ta dễ dàng chứng minh được hai nhận xét sau:
                                                  𝑛
   ∘ Với mọi số nguyên dương 𝑛 ta có 2 𝑛+2 ‖ 32 − 1,
   ∘ Với mọi số nguyên dương 𝑛 ≥ 3 ta có 2 𝑛 > 𝑛 + 2.
   Viết lại phương trình đã cho dưới dạng

                 2 𝑥 𝑦 = (3 − 1) 3 𝑥−1 + 3 𝑥−2 + · · · + 3 + 1
                                 (︀                           )︀

                       = 2 3 𝑥−1 + 3 𝑥−2 + · · · + 3 + 1 .
                            (︀                          )︀

Đặt 𝑥 = 2 𝑛 𝛼 với 𝛼, 𝑛 ∈ N và 𝛼 lẻ.
    +, Nếu 𝑛 = 0 tức là 𝑥 là số lẻ, mà 3 𝑥−1 + 3 𝑥−2 + ... + 3 + 1 là tổng của 𝑥 số
lẻ nên là số lẻ, suy ra 2 ‖ 2 𝑥 𝑦 ⇒ 𝑥 = 1 ⇒ 𝑦 = 1. Từ đó (𝑥, 𝑦) = (1, 1) là một
nghiệm của phương trình. )︀
                        (︀ 𝑛
    +, Nếu 𝑛 ≥ 1, ta có 32
            𝑛        (︀ 𝑛 )︀ 𝛼
          22 𝛼 𝑦 = 32          −1𝛼
                     (︀ 𝑛       )︀ (︁(︀ 2 𝑛 )︀ 𝛼−1 (︀ 2 𝑛 )︀ 𝛼−2          (︀ 𝑛 )︀ )︁
                  = 32 − 1             3          + 3            + · · · + 32 + 1 .
    (︀ 𝑛 )︀ 𝛼−1 (︀ 2 𝑛 )︀ 𝛼−2          (︀ 𝑛 )︀
Mà 32          + 3            + · · · + 32 + 1 là tổng của 𝑛 số lẻ nên là số lẻ. Do đó
                                                   𝑛
theo nhận xét trên ta có 2 𝑛+2 ‖ 3 𝑥 − 1 = 22 𝛼 𝑦. Suy ra 𝑛 + 2 ≥ 2 𝑛 𝛼 ⇒ 𝑛 ≤ 2.
Kiểm tra trực tiếp dễ thấy với 𝑛 = 1 ⇒ 𝛼 = 1 ⇒ 𝑥 = 2, 𝑦 = 2; với 𝑛 = 2 tương
tự ta có 𝛼 = 1 và từ đó 𝑥 = 4, 𝑦 = 5.
    Vậy phương trình có các nghiệm (1, 1) ; (2, 2) và (4, 5).
Bài 14. (Dự tuyển IMO 1991) Tìm số nguyên dương 𝑘 lơn nhất sao cho
                                          1992            1990
                         1991 𝑘 | 19901991       + 19921991      .

                                       Lời giải.

   Trước hết ta chứng minh bằng quy nạp rằng với mọi số lẻ 𝑎 ≥ 3 và số nguyên
𝑛 ≥ 0 thì ta có
                                   𝑛                                 𝑛
                    𝑎 𝑛+1 ‖ (𝑎 + 1) 𝑎 − 1 và 𝑎 𝑛+1 ‖ (𝑎 − 1) 𝑎 − 1.
                                                      𝑛
Rõ ràng 𝑛 = 0 hiển nhiên đúng. Giả sử (𝑎 + 1) 𝑎 = 1 + 𝑁 𝑎 𝑛+1 , 𝑎 𝑁 . Khi đó
                𝑛+1              )︀ 𝑎
      (𝑎 + 1) 𝑎     = 1 + 𝑁 𝑎 𝑛+1 = 1 + 𝑎.𝑁 𝑎 𝑛+1 + 𝐶 2 𝑁 2 𝑎2𝑛+2 + 𝑀 𝑎3𝑛+3 .
                     (︀
                                                      𝑎


                                          15
www.VNMATH.com



                                                       𝑛+1
Mà 𝑎 lẻ nên 𝐶 2 chia hết cho 𝑎. Suy ra (𝑎 + 1) 𝑎
              𝑎                                              = 1 + 𝑁 ′ 𝑎 𝑛+2 với 𝑎     𝑁 ′.
  Do đó ta có ngay
                                    1992                              1990
               19911993 ‖ 19901991         + 1, 19911991 ‖ 19921991          − 1.

Vậy 𝑘 = 1991.
                                                                                    2𝑛+1
Bài 15. (IMO 1990) Tìm tất cả các số nguyên dương 𝑛 sao cho                              là số
                                                                                      𝑛2
nguyên.

                                           Lời giải

    Với 𝑛 = 1 thỏa mãn. Xét 𝑛 > 1 ⇒ 𝑛 lẻ. Giả sử 𝑝 ≥ 3 là ước nguyên tố
nhỏ nhất của 𝑛. Ta có gcd (𝑝 − 1, 𝑛) = 1 và 𝑝 | 2 𝑛 + 1 | 22𝑛 − 1. Theo định
lý Fermat 𝑝 | 2 𝑝−1 − 1 suy ra 𝑝 | gcd (2 𝑝−1 − 1, 22𝑛 − 1) = 2gcd(2𝑛,𝑝−1) − 1. Mà
gcd (2𝑛, 𝑝 − 1) ≤ 2 suy ra 𝑝 | 3 ⇒ 𝑝 = 3. Đặt 𝑛 = 33 𝑑 với 2, 3 𝑑.
    Ta có nhận xét: Nếu 2 𝑚 − 1 chia hết cho 3 𝑟 thì 𝑚 chia hết cho 3 𝑟−1 (xem ví
dụ 3). Hơn nữa do 32𝑘 | 𝑛2 | 22𝑛 − 1 ⇒ 32𝑘−1 | 𝑛 = 3 𝑘 𝑑 ⇒ 𝑘 = 1.
    Giả sử 𝑑 > 1 và 𝑞 là ước nguyên tố nhỏ nhất của 𝑑. Rõ ràng 𝑞 lẻ và 𝑞 ≥
5, gcd (𝑛, 𝑞 − 1) | 3. Khi đó ta có 𝑞 | 22𝑛 − 1, 𝑞 | 2 𝑞−1 − 1 ⇒ 𝑞 | 2gcd(2𝑛,𝑞−1) − 1 |
26 − 1 = 32 .7. Do đó 𝑞 = 7 ⇒ 7 | 𝑛 | 2 𝑛 + 1 mà 2 𝑛 + 1 ≡ 2, 3, 5 (mod 7) mâu
thuẫn. Vậy 𝑑 = 1 ⇒ 𝑛 = 3 thỏa mãn. Đáp số 𝑛 = 1, 𝑛 = 3.
Bài 16. (CĐT Trung Quốc 2005) Cho 𝑏, 𝑚, 𝑛 là các số nguyên dương 𝑏 > 1, 𝑚 ̸=
 𝑛. Chứng minh rằng nếu 𝑏 𝑚 − 1 và 𝑏 𝑛 − 1 có cùng các ước nguyên tố thì 𝑏 + 1 là
lũy thừa của 2.

                                           Lời giải.

     Trước hết ta chứng minh bài toán cho trường hợp 𝑛 = 1, tức là nếu 𝑎 > 1, 𝑘 >
1 và 𝑎 𝑘 − 1 và 𝑎 − 1 có cùng các nhân tử nguyên tố thì 𝑘 và 𝑎 + 1 là lũy thừa
của 2. Phản chứng, giả sử 𝑝 là ước nguyên tố lẻ của 𝑘. Khi đó mọi ước nguyên
tố 𝑞 của 1 + 𝑎 + 𝑎2 + · · · + 𝑎 𝑝−1 là ước của 1 + 𝑎 + 𝑎2 + · · · + 𝑎 𝑘−1 | 𝑎 𝑘 − 1 | 𝑎 − 1.
Suy ra 1 + 𝑎 + 𝑎2 + · · · + 𝑎 𝑝−1 ≡ 𝑝 (mod 𝑞) ≡ 0 (mod 𝑞), nên 𝑝 = 𝑞. Do đó
1 + 𝑎 + 𝑎2 + · · · + 𝑎 𝑝−1 = 𝑝 𝑡 , 𝑡 > 1. Đặt 𝑎 = 𝑢𝑝 + 1 ⇒ 1 + 𝑎 + 𝑎2 + · · · + 𝑎 𝑝−1 ≡
 𝑝 (mod 𝑝2 ) mâu thuẫn. Vậy 𝑘 là lũy thừa của 2. Gọi 𝑟 là ước nguyên tố bất kỳ
của 𝑎 + 1 ⇒ 𝑟 | 𝑎 𝑘 − 1 ⇒ 𝑟 | 𝑎 − 1 ⇒ 𝑟 = 2.
     Trong trường hợp 𝑛 bất kỳ, đặt 𝑑 = gcd (𝑚, 𝑛) , 𝑘 = 𝑚/𝑑. Ta thu được 𝑏 𝑘𝑏 − 1
và 𝑏 𝑑 − 1 có cùng các ước nguyên tố. Do đó theo chứng minh trên ta có 𝑘 là lũy
thừa của 2 và 𝑏 𝑑 + 1 = 2 𝑡 , 𝑡 ≥ 2. Xét mô-đun 4 suy ra 𝑑 lẻ. Giả sử 𝑟 là ước nguyên
tố của 𝑏 + 1 suy ra 𝑟 | 𝑏 + 1 | 𝑏2 − 1 | 𝑏 𝑘𝑑 − 1 ⇒ 𝑟 | 𝑏 𝑑 − 1 ≡ −2 (mod 𝑟). Từ đó
 𝑟 = 2, tức là 𝑏 + 1 là lũy thừa của 2.

III. Một số bài tập
Bài 1. (Dự tuyển IMO 2007) Cho 𝑏, 𝑛 là các số nguyên. Giả sử với mỗi 𝑘 > 1
luôn tồn tại số nguyên 𝑎 𝑘 sao cho 𝑏 − 𝑎 𝑘𝑛 chia hết cho 𝑘. Chứng minh rằng 𝑏 = 𝐴 𝑛
với 𝐴 là số nguyên nào đó.

                                              16
www.VNMATH.com



     Hướng dẫn: Biểu diễn 𝑏 = 𝑝1𝛼1 . . . 𝑝 𝑙𝛼 𝑙 . Do 𝑏2 | 𝑏 − 𝑎 𝑏𝑛2 ⇒ 𝑝 𝑖𝛼 𝑖 ‖ 𝑎 𝑏𝑛2 ∀𝑖. Suy ra
 𝑛 | 𝛼 𝑖 ∀𝑖 tức là 𝑏 là lũy thừa đúng bậc 𝑛.
Bài 2. (Dự tuyển IMO 2002) Cho 𝑝1 , 𝑝2 , ..., 𝑝 𝑛 là các số nguyên tố phân biệt lớn
hơn 3. Chứng minh rằng 2 𝑝1 𝑝2 ...𝑝 𝑛 + 1 có ít nhất 4 𝑛 ước số.
     Hướng dẫn: Chú ý nếu 𝑢, 𝑣 lẻ và (𝑢, 𝑣) = 1 thì ước chung lớn nhất của 2 𝑢 + 1
và 2 𝑣 + 1 là 3. Từ đó 2 𝑢𝑣 + 1 chia hết cho (2 𝑢 + 1) (2 𝑣 + 1) /3. Sau đó chứng minh
bài toán bằng quy nạp theo 𝑛.
Bài 3. (IMO 2006) Xác định tất cả các cặp số nguyên (𝑥, 𝑦) thỏa mãn phương
trình
                                  1 + 2 𝑥 + 22𝑥+1 = 𝑦 2 .
     Hướng dẫn: Xét 𝑥 > 0, 𝑦 > 0 và 𝑦 lẻ. Chú ý 2 𝑥 (1 + 2 𝑥+1 ) = (𝑦 − 1) (𝑦 + 1) từ
đó ta có thể biểu diễn 𝑦 theo 𝑥. Suy ra 𝑥 = 4, 𝑦 = 23. Đáp số (0, ±2) ; (4, ±23) .
Bài 4. (Dự tuyển IMO 2001) Cho 𝑝 là số nguyên tố lớn hơn 3. Chứng minh rằng
tồn tại số nguyên 𝑎 với 1 ≤ 𝑎 ≤ 𝑝 − 2 sao cho hoặc 𝑎 𝑝−1 − 1 hoặc (𝑎 + 1) 𝑝−1 − 1
chia hết cho 𝑝2 .
     Hướng dẫn: Đặt 𝐴 = {1 ≤ 𝑎 ≤ 𝑝 − 1 : 𝑎 𝑝−1 ̸≡ 1 (mod 𝑝2 )}. Vì ít nhất một
trong hai số 𝑎, 𝑝 − 𝑎 thuộc 𝐴 và 1 ∈ 𝐴 và |𝐴| ≥ (𝑝 − 1) /2 = 𝑘 và giả sử có
                                          /
đúng một trong hai số 2𝑖, 2𝑖 + 1 (1 ≤ 𝑖 ≤ 𝑘 − 1) thuộc 𝐴. Xét 𝑝 ≥ 7 ⇒ 2𝑘 − 1 =
 𝑝 − 2 ∈ 𝐴 ⇒ 𝑝 − 3 = 2𝑘 − 2 ∈ 𝐴. Bằng cách xét mô-đun 𝑝2 ⇒ 2𝑘 − 3 ∈ 𝐴, tức là
       /
 𝑝 − 3, 𝑝 − 4 thỏa mãn.
Bài 5. (CĐT Việt Nam 2008) Cho 𝑚, 𝑛 là các số nguyên dương. Chứng minh
rằng (2𝑚 + 3) 𝑛 + 1 chia hết cho 6𝑚 khi và chỉ khi 3 𝑛 + 1 chia hết cho 4𝑚.
     Hướng dẫn: Khai triển Newton, bài toán trở thành chứng minh 2𝑚 | 3 𝑛 + 1
và 3 | (2𝑚) 𝑛 + 1 khi và chỉ khi 4𝑚 | 3 𝑛 + 1.
Bài 6. (Dự tuyển IMO 2000) Xác định tất cả các bộ ba số nguyên dương (𝑎, 𝑚, 𝑛)
sao cho 𝑎 𝑚 + 1 là ước của (𝑎 + 1) 𝑛 .
     Hướng dẫn: Chú ý nếu 𝑢 | 𝑣 𝑙 ⇒ 𝑢 | (gcd (𝑢, 𝑣)) 𝑙 . Xét 𝑎 > 1 và 𝑚 > 1,
khi đó 𝑚 lẻ. Giả sử 𝑝 là ước nguyên tố của 𝑚. Bằng cách xét mô-đun, suy ra
(𝑏 𝑝 + 1) / (𝑏 + 1) = 𝑝 ⇒ 𝑝 = 3. Đáp số: {(1, 𝑚, 𝑛) ; (𝑎, 1, 𝑛) ; (2, 3, 𝑛 ≥ 2)} .
Bài 7. (Nga 2000) Hỏi có tồn tại ba số 𝑎, 𝑏, 𝑐 > 1 đôi một nguyên tố cùng nhau
và thỏa mãn
                           𝑏 | 2 𝑎 + 1, 𝑐 | 2 𝑏 + 1, 𝑎 | 2 𝑐 + 1.
    Hướng dẫn: Ta có 𝑎, 𝑏, 𝑐 là lẻ. Giả sử 𝑎 = min {𝑎, 𝑏, 𝑐}. Nếu 𝑎, 𝑏, 𝑐 nguyên tố
suy ra 𝑎 = 3, không tồn tại 𝑏, 𝑐. Trường hợp bất kỳ ta ký hiệu 𝜋 (𝑛) là ước nguyên
tố nhỏ nhất của 𝑛. Khi đó nếu 𝑝 là số nguyên tố sao cho 𝑝 | 2 𝑦 + 1 và 𝑝 < 𝜋 (3)
thì 𝑝 = 3. Sử dụng tính chất này suy ra mâu thuẫn.
Bài 8. (IMO 2000) Hỏi có tồn tại hay không số nguyên dương 𝑛 có đúng 2000
ước nguyên tố và 2 𝑛 + 1 chia hết cho 𝑛.
    Hướng dẫn: Sử dụng quy nạp theo số các ước số, 𝑘. Chỉ ra rằng ta có thể tìm
 𝑛 với đúng 𝑘 ước nguyên tố sao cho 𝑛 | (2 𝑛 + 1) và có một ước nguyên tố của
2 𝑛 + 1 nhưng không là ước của 𝑛.


                                              17
www.VNMATH.com



Bài 9. (CĐT Iran 2006) Cho 𝑛 là số nguyên dương. Tìm tất cả các bộ 𝑛 số
 𝑎1 , 𝑎2 , ..., 𝑎 𝑛 đôi một phân biệt và đôi một nguyên tố cùng nhau sao cho với mọi
1 ≤ 𝑖 ≤ 𝑛 ta có
                             𝑎1 + 𝑎2 + · · · + 𝑎 𝑛 | 𝑎1 + 𝑎2 + · · · + 𝑎 𝑖𝑛 .
                                                      𝑖    𝑖


   Hướng dẫn: Chứng minh 𝑎1 +𝑎2 +· · ·+𝑎 𝑛 | 𝑛 (𝑛 − 1) và hai số này bằng nhau.
Nếu 𝑛 ≥ 8 thì 𝑎1 + 𝑎2 + · · · + 𝑎 𝑛 ≥ 1 + 2 + 3 + 5 + · · · + 2𝑛 + 1 = 𝑛2 − 7 > 𝑛2 − 𝑛.
Đáp số 𝑛 = 1, 𝑎1 tùy ý.
Bài 10. (Iran 2004) Cho 𝑎1 , 𝑎2 , ..., 𝑎 𝑛 là các số nguyên không đồng thời bằng
nhau. Chứng minh rằng tồn tại vô số số nguyên tố 𝑝 sao cho có tồn tại số nguyên
dương 𝑘 sao cho
                                𝑝 | 𝑎1𝑘 + 𝑎2𝑘 + · · · + 𝑎 𝑘𝑛 .
   Hướng dẫn: Ta có thể giả sử gcd (𝑎1 , 𝑎2 , ..., 𝑎 𝑛 ) = 1 và 𝑎 𝑛 ≥ 2. Giả sử chỉ có
hữu hạn các số nguyên tố 𝑝1 , ..., 𝑝 𝑚 . Chọn 𝑘 thích hợp để suy ra 𝑎1𝑘 + 𝑎2𝑘 + · · · + 𝑎 𝑘𝑛
có ước nguyên tố lớn hơn 𝑝1 𝑝2 ...𝑝 𝑚 .
Bài 11. (Dự tuyển IMO 2002) Tìm tất cả các cặp số nguyên 𝑚, 𝑛 ≥ 3 sao cho
tồn tại vô hạn các số nguyên 𝑎 thỏa mãn
                                        𝑎 𝑚+ 𝑎−1
                                        𝑎 𝑛 + 𝑎2 − 1

là một số nguyên.
    Hướng dẫn: Rõ ràng 𝑛 < 𝑚. Sử dụng tính chất đa thức suy ra biểu thức 𝐴 đã
cho nhận giá trị nguyên với mọi 𝑎. Đa thức ở tử số chia hết đa thức ở mẫu số nên
chúng có nghiệm chung 𝛼 ∈ (0, 1) từ đó đánh giá theo 𝛼 suy ra 𝑚 < 2𝑛. Cuối
cùng chọn 𝑎 = 2 và xét mô-đun 𝑑 = 2 𝑛 + 3 (mẫu số) ta được (𝑚, 𝑛) = (5, 3) .
Bài 12. (Trung Quốc 2008) Tìm tất cả các bộ ba số (𝑝, 𝑞, 𝑛) sao cho 𝑞 𝑛+2 ≡
3 𝑛+2 (mod 𝑝 𝑛 ) , 𝑝 𝑛+2 ≡ 3 𝑛+2 (mod 𝑞 𝑛 ) trong đó 𝑝, 𝑞 là các số nguyên tố lẻ và 𝑛 là
số nguyên dương.
    Hướng dẫn: Nếu hai trong ba số 𝑝, 𝑞, 3 bằng nhau ta có nghiệm (3, 3, 𝑛). Xét
trường hợp chúng phân biệt và 𝑝 > 𝑞. Ta có 𝑝 𝑛 𝑞 𝑛 | 𝑝 𝑛+2 + 𝑞 𝑛+2 − 3 𝑛+2 < 2𝑝 𝑛+2 ⇒
 𝑝2 > 𝑞 𝑛−1 . Mà 𝑞 𝑛+2 − 3 𝑛+2 ≥ 𝑝 𝑛 . Suy ra 𝑛 ≤ 3.




                                            18

Weitere ähnliche Inhalte

Was ist angesagt?

Bộ sưu tập bất đẳng thức của võ quốc bá cẩn
Bộ sưu tập bất đẳng thức của võ quốc bá cẩnBộ sưu tập bất đẳng thức của võ quốc bá cẩn
Bộ sưu tập bất đẳng thức của võ quốc bá cẩnThế Giới Tinh Hoa
 
72 hệ phương trình
72 hệ phương trình72 hệ phương trình
72 hệ phương trìnhHades0510
 
Topo daicuong1[1]
Topo daicuong1[1]Topo daicuong1[1]
Topo daicuong1[1]Bui Loi
 
19 phương phap chứng minh bất đẳng thức
19 phương phap chứng minh bất đẳng thức19 phương phap chứng minh bất đẳng thức
19 phương phap chứng minh bất đẳng thứcThế Giới Tinh Hoa
 
CHUYÊN ĐỀ SỐ HỌC ÔN THI VÀO LỚP 10 CÁC TRƯỜNG CHUYÊN
CHUYÊN ĐỀ SỐ HỌC ÔN THI VÀO LỚP 10 CÁC TRƯỜNG CHUYÊNCHUYÊN ĐỀ SỐ HỌC ÔN THI VÀO LỚP 10 CÁC TRƯỜNG CHUYÊN
CHUYÊN ĐỀ SỐ HỌC ÔN THI VÀO LỚP 10 CÁC TRƯỜNG CHUYÊNBOIDUONGTOAN.COM
 
Tuyển tập 500 bài Bất Đẳng Thức cổ điển
Tuyển tập 500 bài Bất Đẳng Thức cổ điểnTuyển tập 500 bài Bất Đẳng Thức cổ điển
Tuyển tập 500 bài Bất Đẳng Thức cổ điểnNguyễn Việt Long
 
Phương Tích - Trục Đẳng Phương
Phương Tích - Trục Đẳng PhươngPhương Tích - Trục Đẳng Phương
Phương Tích - Trục Đẳng PhươngNhập Vân Long
 
Một số chuyên đề nâng cao đại số lớp 7
Một số chuyên đề nâng cao đại số lớp 7Một số chuyên đề nâng cao đại số lớp 7
Một số chuyên đề nâng cao đại số lớp 7vukimhoanc2vinhhoa
 
chuyên đề cực trị GTLN và GTNN , rất chi tiết và đầy đủ
chuyên đề cực trị GTLN và GTNN , rất chi tiết và đầy đủ chuyên đề cực trị GTLN và GTNN , rất chi tiết và đầy đủ
chuyên đề cực trị GTLN và GTNN , rất chi tiết và đầy đủ Jackson Linh
 
Ứng dụng đồng dư vào giải toán chia hết lớp 9
Ứng dụng đồng dư vào giải toán chia hết lớp 9Ứng dụng đồng dư vào giải toán chia hết lớp 9
Ứng dụng đồng dư vào giải toán chia hết lớp 9youngunoistalented1995
 
Một số dạng toán về đa thức qua các kỳ thi Olympic
Một số dạng toán về đa thức qua các kỳ thi OlympicMột số dạng toán về đa thức qua các kỳ thi Olympic
Một số dạng toán về đa thức qua các kỳ thi OlympicMan_Ebook
 
Đặt ẩn phụ giải phương trình chứa căn
Đặt ẩn phụ giải phương trình chứa cănĐặt ẩn phụ giải phương trình chứa căn
Đặt ẩn phụ giải phương trình chứa căntuituhoc
 
Cđ giải hpt không mẫu mực
Cđ giải hpt không mẫu mựcCđ giải hpt không mẫu mực
Cđ giải hpt không mẫu mựcCảnh
 

Was ist angesagt? (20)

Bộ sưu tập bất đẳng thức của võ quốc bá cẩn
Bộ sưu tập bất đẳng thức của võ quốc bá cẩnBộ sưu tập bất đẳng thức của võ quốc bá cẩn
Bộ sưu tập bất đẳng thức của võ quốc bá cẩn
 
72 hệ phương trình
72 hệ phương trình72 hệ phương trình
72 hệ phương trình
 
Topo daicuong1[1]
Topo daicuong1[1]Topo daicuong1[1]
Topo daicuong1[1]
 
19 phương phap chứng minh bất đẳng thức
19 phương phap chứng minh bất đẳng thức19 phương phap chứng minh bất đẳng thức
19 phương phap chứng minh bất đẳng thức
 
CHUYÊN ĐỀ SỐ HỌC ÔN THI VÀO LỚP 10 CÁC TRƯỜNG CHUYÊN
CHUYÊN ĐỀ SỐ HỌC ÔN THI VÀO LỚP 10 CÁC TRƯỜNG CHUYÊNCHUYÊN ĐỀ SỐ HỌC ÔN THI VÀO LỚP 10 CÁC TRƯỜNG CHUYÊN
CHUYÊN ĐỀ SỐ HỌC ÔN THI VÀO LỚP 10 CÁC TRƯỜNG CHUYÊN
 
Tuyển tập 500 bài Bất Đẳng Thức cổ điển
Tuyển tập 500 bài Bất Đẳng Thức cổ điểnTuyển tập 500 bài Bất Đẳng Thức cổ điển
Tuyển tập 500 bài Bất Đẳng Thức cổ điển
 
Phương Tích - Trục Đẳng Phương
Phương Tích - Trục Đẳng PhươngPhương Tích - Trục Đẳng Phương
Phương Tích - Trục Đẳng Phương
 
Dãy số và giới hạn
Dãy số và giới hạnDãy số và giới hạn
Dãy số và giới hạn
 
Hinh hoc-affine
Hinh hoc-affineHinh hoc-affine
Hinh hoc-affine
 
Phương trình hàm đa thức
Phương trình hàm đa thứcPhương trình hàm đa thức
Phương trình hàm đa thức
 
Một số chuyên đề nâng cao đại số lớp 7
Một số chuyên đề nâng cao đại số lớp 7Một số chuyên đề nâng cao đại số lớp 7
Một số chuyên đề nâng cao đại số lớp 7
 
chuyên đề cực trị GTLN và GTNN , rất chi tiết và đầy đủ
chuyên đề cực trị GTLN và GTNN , rất chi tiết và đầy đủ chuyên đề cực trị GTLN và GTNN , rất chi tiết và đầy đủ
chuyên đề cực trị GTLN và GTNN , rất chi tiết và đầy đủ
 
Ứng dụng đồng dư vào giải toán chia hết lớp 9
Ứng dụng đồng dư vào giải toán chia hết lớp 9Ứng dụng đồng dư vào giải toán chia hết lớp 9
Ứng dụng đồng dư vào giải toán chia hết lớp 9
 
Một số dạng toán về đa thức qua các kỳ thi Olympic
Một số dạng toán về đa thức qua các kỳ thi OlympicMột số dạng toán về đa thức qua các kỳ thi Olympic
Một số dạng toán về đa thức qua các kỳ thi Olympic
 
9 drichle
9 drichle9 drichle
9 drichle
 
Bài tập hàm biến phức
Bài tập hàm biến phứcBài tập hàm biến phức
Bài tập hàm biến phức
 
Luận văn: Một số lớp bài toán về phương trình hàm, HAY, 9đ
Luận văn: Một số lớp bài toán về phương trình hàm, HAY, 9đLuận văn: Một số lớp bài toán về phương trình hàm, HAY, 9đ
Luận văn: Một số lớp bài toán về phương trình hàm, HAY, 9đ
 
Đặt ẩn phụ giải phương trình chứa căn
Đặt ẩn phụ giải phương trình chứa cănĐặt ẩn phụ giải phương trình chứa căn
Đặt ẩn phụ giải phương trình chứa căn
 
Cđ giải hpt không mẫu mực
Cđ giải hpt không mẫu mựcCđ giải hpt không mẫu mực
Cđ giải hpt không mẫu mực
 
Dãy số vmo2009
Dãy số vmo2009Dãy số vmo2009
Dãy số vmo2009
 

Andere mochten auch

9 phương pháp giải phương trình nghiệm nguyên
9 phương pháp giải phương trình nghiệm nguyên9 phương pháp giải phương trình nghiệm nguyên
9 phương pháp giải phương trình nghiệm nguyênThấy Tên Tao Không
 
Cđ đồng dư thức trong toán 7
Cđ đồng dư thức trong toán 7Cđ đồng dư thức trong toán 7
Cđ đồng dư thức trong toán 7Cảnh
 
Toán lớp 6 - Một số phương pháp giải Toán chia hết
Toán lớp 6 - Một số phương pháp giải Toán chia hếtToán lớp 6 - Một số phương pháp giải Toán chia hết
Toán lớp 6 - Một số phương pháp giải Toán chia hếtBồi dưỡng Toán lớp 6
 
Tuyển tập 28 đề thi và đáp án HSG Toán lớp 6
Tuyển tập 28 đề thi và đáp án HSG Toán lớp 6Tuyển tập 28 đề thi và đáp án HSG Toán lớp 6
Tuyển tập 28 đề thi và đáp án HSG Toán lớp 6Bồi dưỡng Toán lớp 6
 
Thuật toán Nhân Bình Phương - demo
Thuật toán Nhân Bình Phương - demoThuật toán Nhân Bình Phương - demo
Thuật toán Nhân Bình Phương - demoCông Thắng Trương
 
MATMA - Chuong3 l tso
MATMA - Chuong3 l tsoMATMA - Chuong3 l tso
MATMA - Chuong3 l tsoSai Lemovom
 
07 chương 5. lý thuyết số (2)
07  chương 5. lý thuyết số (2)07  chương 5. lý thuyết số (2)
07 chương 5. lý thuyết số (2)Andy Nhân
 
Chương 1 dai so truu tuong
Chương 1 dai so truu tuongChương 1 dai so truu tuong
Chương 1 dai so truu tuongvpmity
 
CHUYÊN ĐỀ LŨY THỪA & TỶ LỆ THỨC DÃY TỶ SỐ BẰNG NHAU TOÁN 7 CỰC HAY -HOÀNG THÁ...
CHUYÊN ĐỀ LŨY THỪA & TỶ LỆ THỨC DÃY TỶ SỐ BẰNG NHAU TOÁN 7 CỰC HAY -HOÀNG THÁ...CHUYÊN ĐỀ LŨY THỪA & TỶ LỆ THỨC DÃY TỶ SỐ BẰNG NHAU TOÁN 7 CỰC HAY -HOÀNG THÁ...
CHUYÊN ĐỀ LŨY THỪA & TỶ LỆ THỨC DÃY TỶ SỐ BẰNG NHAU TOÁN 7 CỰC HAY -HOÀNG THÁ...Hoàng Thái Việt
 
Chuyen de mon toan nguyen tac dirichlet
Chuyen de mon toan nguyen tac dirichletChuyen de mon toan nguyen tac dirichlet
Chuyen de mon toan nguyen tac dirichletCảnh
 
Cđ nguyên lí đi rich lê
Cđ nguyên lí đi rich lêCđ nguyên lí đi rich lê
Cđ nguyên lí đi rich lêCảnh
 
PHÉP QUAY & PHÉP VỊ TỰ
PHÉP QUAY & PHÉP VỊ TỰPHÉP QUAY & PHÉP VỊ TỰ
PHÉP QUAY & PHÉP VỊ TỰDANAMATH
 
Toán lớp 6 nâng cao - Chuyên đề dãy Số tự nhiên theo quy luật
Toán lớp 6 nâng cao - Chuyên đề dãy Số tự nhiên theo quy luậtToán lớp 6 nâng cao - Chuyên đề dãy Số tự nhiên theo quy luật
Toán lớp 6 nâng cao - Chuyên đề dãy Số tự nhiên theo quy luậtBồi dưỡng Toán lớp 6
 
Ứng dụng tam thức bậc 2 để chứng minh bất đẳng thức
Ứng dụng tam thức bậc 2 để chứng minh bất đẳng thứcỨng dụng tam thức bậc 2 để chứng minh bất đẳng thức
Ứng dụng tam thức bậc 2 để chứng minh bất đẳng thứcNhập Vân Long
 
NÂNG CAO TƯ DUY PHÁT TRIỂN VÀ BỔI DƯỠNG HSG MÔN TOÁN LỚP 6 QUA 18 CHUYÊN ĐỀ
NÂNG CAO TƯ DUY PHÁT TRIỂN VÀ BỔI DƯỠNG HSG MÔN TOÁN LỚP 6 QUA 18 CHUYÊN ĐỀNÂNG CAO TƯ DUY PHÁT TRIỂN VÀ BỔI DƯỠNG HSG MÔN TOÁN LỚP 6 QUA 18 CHUYÊN ĐỀ
NÂNG CAO TƯ DUY PHÁT TRIỂN VÀ BỔI DƯỠNG HSG MÔN TOÁN LỚP 6 QUA 18 CHUYÊN ĐỀBồi dưỡng Toán lớp 6
 

Andere mochten auch (18)

9 phương pháp giải phương trình nghiệm nguyên
9 phương pháp giải phương trình nghiệm nguyên9 phương pháp giải phương trình nghiệm nguyên
9 phương pháp giải phương trình nghiệm nguyên
 
Cđ đồng dư thức trong toán 7
Cđ đồng dư thức trong toán 7Cđ đồng dư thức trong toán 7
Cđ đồng dư thức trong toán 7
 
Toán lớp 6 - Một số phương pháp giải Toán chia hết
Toán lớp 6 - Một số phương pháp giải Toán chia hếtToán lớp 6 - Một số phương pháp giải Toán chia hết
Toán lớp 6 - Một số phương pháp giải Toán chia hết
 
Tuyển tập 28 đề thi và đáp án HSG Toán lớp 6
Tuyển tập 28 đề thi và đáp án HSG Toán lớp 6Tuyển tập 28 đề thi và đáp án HSG Toán lớp 6
Tuyển tập 28 đề thi và đáp án HSG Toán lớp 6
 
Thuật toán Nhân Bình Phương - demo
Thuật toán Nhân Bình Phương - demoThuật toán Nhân Bình Phương - demo
Thuật toán Nhân Bình Phương - demo
 
MATMA - Chuong3 l tso
MATMA - Chuong3 l tsoMATMA - Chuong3 l tso
MATMA - Chuong3 l tso
 
07 chương 5. lý thuyết số (2)
07  chương 5. lý thuyết số (2)07  chương 5. lý thuyết số (2)
07 chương 5. lý thuyết số (2)
 
Chương 1 dai so truu tuong
Chương 1 dai so truu tuongChương 1 dai so truu tuong
Chương 1 dai so truu tuong
 
Bt dai so hoang
Bt dai so hoangBt dai so hoang
Bt dai so hoang
 
CHUYÊN ĐỀ LŨY THỪA & TỶ LỆ THỨC DÃY TỶ SỐ BẰNG NHAU TOÁN 7 CỰC HAY -HOÀNG THÁ...
CHUYÊN ĐỀ LŨY THỪA & TỶ LỆ THỨC DÃY TỶ SỐ BẰNG NHAU TOÁN 7 CỰC HAY -HOÀNG THÁ...CHUYÊN ĐỀ LŨY THỪA & TỶ LỆ THỨC DÃY TỶ SỐ BẰNG NHAU TOÁN 7 CỰC HAY -HOÀNG THÁ...
CHUYÊN ĐỀ LŨY THỪA & TỶ LỆ THỨC DÃY TỶ SỐ BẰNG NHAU TOÁN 7 CỰC HAY -HOÀNG THÁ...
 
Chuyen de mon toan nguyen tac dirichlet
Chuyen de mon toan nguyen tac dirichletChuyen de mon toan nguyen tac dirichlet
Chuyen de mon toan nguyen tac dirichlet
 
Cđ nguyên lí đi rich lê
Cđ nguyên lí đi rich lêCđ nguyên lí đi rich lê
Cđ nguyên lí đi rich lê
 
Tai lieu-on-thi-lop-10-mon-toan
Tai lieu-on-thi-lop-10-mon-toanTai lieu-on-thi-lop-10-mon-toan
Tai lieu-on-thi-lop-10-mon-toan
 
PHÉP QUAY & PHÉP VỊ TỰ
PHÉP QUAY & PHÉP VỊ TỰPHÉP QUAY & PHÉP VỊ TỰ
PHÉP QUAY & PHÉP VỊ TỰ
 
Toán lớp 6 nâng cao - Chuyên đề dãy Số tự nhiên theo quy luật
Toán lớp 6 nâng cao - Chuyên đề dãy Số tự nhiên theo quy luậtToán lớp 6 nâng cao - Chuyên đề dãy Số tự nhiên theo quy luật
Toán lớp 6 nâng cao - Chuyên đề dãy Số tự nhiên theo quy luật
 
Ứng dụng tam thức bậc 2 để chứng minh bất đẳng thức
Ứng dụng tam thức bậc 2 để chứng minh bất đẳng thứcỨng dụng tam thức bậc 2 để chứng minh bất đẳng thức
Ứng dụng tam thức bậc 2 để chứng minh bất đẳng thức
 
NÂNG CAO TƯ DUY PHÁT TRIỂN VÀ BỔI DƯỠNG HSG MÔN TOÁN LỚP 6 QUA 18 CHUYÊN ĐỀ
NÂNG CAO TƯ DUY PHÁT TRIỂN VÀ BỔI DƯỠNG HSG MÔN TOÁN LỚP 6 QUA 18 CHUYÊN ĐỀNÂNG CAO TƯ DUY PHÁT TRIỂN VÀ BỔI DƯỠNG HSG MÔN TOÁN LỚP 6 QUA 18 CHUYÊN ĐỀ
NÂNG CAO TƯ DUY PHÁT TRIỂN VÀ BỔI DƯỠNG HSG MÔN TOÁN LỚP 6 QUA 18 CHUYÊN ĐỀ
 
Tuyển tập 300 bài bdt
Tuyển tập 300 bài bdtTuyển tập 300 bài bdt
Tuyển tập 300 bài bdt
 

Ähnlich wie Bài toán số học liên quan tới lũy thữa

Hướng dẫn giải bài tập chuỗi - Toán cao cấp
Hướng dẫn giải bài tập chuỗi - Toán cao cấpHướng dẫn giải bài tập chuỗi - Toán cao cấp
Hướng dẫn giải bài tập chuỗi - Toán cao cấpVan-Duyet Le
 
B1 tinh don dieu cua ham so
B1 tinh don dieu cua ham soB1 tinh don dieu cua ham so
B1 tinh don dieu cua ham sokhoilien24
 
Chuyen%20de%20phuong%20trinh%20nghiem%20nguyen
Chuyen%20de%20phuong%20trinh%20nghiem%20nguyenChuyen%20de%20phuong%20trinh%20nghiem%20nguyen
Chuyen%20de%20phuong%20trinh%20nghiem%20nguyenTam Vu Minh
 
Tai lieu danh cho hsg toan lop 8
Tai lieu danh cho hsg toan lop 8Tai lieu danh cho hsg toan lop 8
Tai lieu danh cho hsg toan lop 8Học Tập Long An
 
Tai lieu danh cho hsg toan lop 8
Tai lieu danh cho hsg toan lop 8Tai lieu danh cho hsg toan lop 8
Tai lieu danh cho hsg toan lop 8Học Tập Long An
 
Tai lieu danh cho hsg toan lop 8
Tai lieu danh cho hsg toan lop 8Tai lieu danh cho hsg toan lop 8
Tai lieu danh cho hsg toan lop 8Học Tập Long An
 
Bài tập phương trình nghiệm nguyên
Bài tập phương trình nghiệm nguyênBài tập phương trình nghiệm nguyên
Bài tập phương trình nghiệm nguyênDuong BUn
 
Kỹ thuật nhân liên hợp
Kỹ thuật nhân liên hợpKỹ thuật nhân liên hợp
Kỹ thuật nhân liên hợptuituhoc
 
kenhgiaovien_6_bai17_ham_so_lien_tuc.pptx
kenhgiaovien_6_bai17_ham_so_lien_tuc.pptxkenhgiaovien_6_bai17_ham_so_lien_tuc.pptx
kenhgiaovien_6_bai17_ham_so_lien_tuc.pptxHoangThong17
 
Bdhsg toan 9 cuc ha ydoc
Bdhsg toan 9  cuc ha ydocBdhsg toan 9  cuc ha ydoc
Bdhsg toan 9 cuc ha ydocTam Vu Minh
 
Bai 2 cong tru so huu ti
Bai 2 cong tru so huu tiBai 2 cong tru so huu ti
Bai 2 cong tru so huu timanggiaoduc
 
Tai lieu danh cho hsg toan lop 8
Tai lieu danh cho hsg toan lop 8Tai lieu danh cho hsg toan lop 8
Tai lieu danh cho hsg toan lop 8Học Tập Long An
 
Hệ phương trình hữu tỉ
Hệ phương trình hữu tỉHệ phương trình hữu tỉ
Hệ phương trình hữu tỉNhập Vân Long
 
Dùng dãy số chứng minh bất đẳng thức
Dùng dãy số chứng minh bất đẳng thứcDùng dãy số chứng minh bất đẳng thức
Dùng dãy số chứng minh bất đẳng thứcThế Giới Tinh Hoa
 
giải tích tuần 1.pptx
giải tích tuần 1.pptxgiải tích tuần 1.pptx
giải tích tuần 1.pptxTinTng26
 
Một số dạng toán về đa thức qua các kỳ thi Olympic 6732069.pdf
Một số dạng toán về đa thức qua các kỳ thi Olympic 6732069.pdfMột số dạng toán về đa thức qua các kỳ thi Olympic 6732069.pdf
Một số dạng toán về đa thức qua các kỳ thi Olympic 6732069.pdfTieuNgocLy
 
Toan pt.de054.2012
Toan pt.de054.2012Toan pt.de054.2012
Toan pt.de054.2012BẢO Hí
 
Đề thi tóan cao cấp k15
Đề thi tóan cao cấp k15Đề thi tóan cao cấp k15
Đề thi tóan cao cấp k15dethinhh
 

Ähnlich wie Bài toán số học liên quan tới lũy thữa (20)

Hướng dẫn giải bài tập chuỗi - Toán cao cấp
Hướng dẫn giải bài tập chuỗi - Toán cao cấpHướng dẫn giải bài tập chuỗi - Toán cao cấp
Hướng dẫn giải bài tập chuỗi - Toán cao cấp
 
Huongdangiai bt chuoi
Huongdangiai bt chuoiHuongdangiai bt chuoi
Huongdangiai bt chuoi
 
B1 tinh don dieu cua ham so
B1 tinh don dieu cua ham soB1 tinh don dieu cua ham so
B1 tinh don dieu cua ham so
 
Chuyen%20de%20phuong%20trinh%20nghiem%20nguyen
Chuyen%20de%20phuong%20trinh%20nghiem%20nguyenChuyen%20de%20phuong%20trinh%20nghiem%20nguyen
Chuyen%20de%20phuong%20trinh%20nghiem%20nguyen
 
slide bài giảng giải tích 3
slide bài giảng giải tích 3slide bài giảng giải tích 3
slide bài giảng giải tích 3
 
Tai lieu danh cho hsg toan lop 8
Tai lieu danh cho hsg toan lop 8Tai lieu danh cho hsg toan lop 8
Tai lieu danh cho hsg toan lop 8
 
Tai lieu danh cho hsg toan lop 8
Tai lieu danh cho hsg toan lop 8Tai lieu danh cho hsg toan lop 8
Tai lieu danh cho hsg toan lop 8
 
Tai lieu danh cho hsg toan lop 8
Tai lieu danh cho hsg toan lop 8Tai lieu danh cho hsg toan lop 8
Tai lieu danh cho hsg toan lop 8
 
Bài tập phương trình nghiệm nguyên
Bài tập phương trình nghiệm nguyênBài tập phương trình nghiệm nguyên
Bài tập phương trình nghiệm nguyên
 
Kỹ thuật nhân liên hợp
Kỹ thuật nhân liên hợpKỹ thuật nhân liên hợp
Kỹ thuật nhân liên hợp
 
kenhgiaovien_6_bai17_ham_so_lien_tuc.pptx
kenhgiaovien_6_bai17_ham_so_lien_tuc.pptxkenhgiaovien_6_bai17_ham_so_lien_tuc.pptx
kenhgiaovien_6_bai17_ham_so_lien_tuc.pptx
 
Bdhsg toan 9 cuc ha ydoc
Bdhsg toan 9  cuc ha ydocBdhsg toan 9  cuc ha ydoc
Bdhsg toan 9 cuc ha ydoc
 
Bai 2 cong tru so huu ti
Bai 2 cong tru so huu tiBai 2 cong tru so huu ti
Bai 2 cong tru so huu ti
 
Tai lieu danh cho hsg toan lop 8
Tai lieu danh cho hsg toan lop 8Tai lieu danh cho hsg toan lop 8
Tai lieu danh cho hsg toan lop 8
 
Hệ phương trình hữu tỉ
Hệ phương trình hữu tỉHệ phương trình hữu tỉ
Hệ phương trình hữu tỉ
 
Dùng dãy số chứng minh bất đẳng thức
Dùng dãy số chứng minh bất đẳng thứcDùng dãy số chứng minh bất đẳng thức
Dùng dãy số chứng minh bất đẳng thức
 
giải tích tuần 1.pptx
giải tích tuần 1.pptxgiải tích tuần 1.pptx
giải tích tuần 1.pptx
 
Một số dạng toán về đa thức qua các kỳ thi Olympic 6732069.pdf
Một số dạng toán về đa thức qua các kỳ thi Olympic 6732069.pdfMột số dạng toán về đa thức qua các kỳ thi Olympic 6732069.pdf
Một số dạng toán về đa thức qua các kỳ thi Olympic 6732069.pdf
 
Toan pt.de054.2012
Toan pt.de054.2012Toan pt.de054.2012
Toan pt.de054.2012
 
Đề thi tóan cao cấp k15
Đề thi tóan cao cấp k15Đề thi tóan cao cấp k15
Đề thi tóan cao cấp k15
 

Mehr von Thế Giới Tinh Hoa

Cách chụp ảnh công ty đẹp 2019
Cách chụp ảnh công ty đẹp 2019Cách chụp ảnh công ty đẹp 2019
Cách chụp ảnh công ty đẹp 2019Thế Giới Tinh Hoa
 
Bảng báo giá sản phẩm rèm bạch dương
Bảng báo giá sản phẩm rèm bạch dươngBảng báo giá sản phẩm rèm bạch dương
Bảng báo giá sản phẩm rèm bạch dươngThế Giới Tinh Hoa
 
Album sổ mẫu Rèm cửa Bạch Dương
Album sổ mẫu Rèm cửa Bạch DươngAlbum sổ mẫu Rèm cửa Bạch Dương
Album sổ mẫu Rèm cửa Bạch DươngThế Giới Tinh Hoa
 
Cách tắm cho bé vào mùa đông
Cách tắm cho bé vào mùa đôngCách tắm cho bé vào mùa đông
Cách tắm cho bé vào mùa đôngThế Giới Tinh Hoa
 
Giáo trình tự học illustrator cs6
Giáo trình tự học illustrator cs6  Giáo trình tự học illustrator cs6
Giáo trình tự học illustrator cs6 Thế Giới Tinh Hoa
 
Nữ quái sân trườngtruonghocso.com
Nữ quái sân trườngtruonghocso.comNữ quái sân trườngtruonghocso.com
Nữ quái sân trườngtruonghocso.comThế Giới Tinh Hoa
 
Những chàng trai xấu tính nguyễn nhật ánhtruonghocso.com
Những chàng trai xấu tính  nguyễn nhật ánhtruonghocso.comNhững chàng trai xấu tính  nguyễn nhật ánhtruonghocso.com
Những chàng trai xấu tính nguyễn nhật ánhtruonghocso.comThế Giới Tinh Hoa
 
Những bài văn mẫu dành cho học sinh lớp 10truonghocso.com
Những bài văn mẫu dành cho học sinh lớp 10truonghocso.comNhững bài văn mẫu dành cho học sinh lớp 10truonghocso.com
Những bài văn mẫu dành cho học sinh lớp 10truonghocso.comThế Giới Tinh Hoa
 

Mehr von Thế Giới Tinh Hoa (20)

Cách chụp ảnh công ty đẹp 2019
Cách chụp ảnh công ty đẹp 2019Cách chụp ảnh công ty đẹp 2019
Cách chụp ảnh công ty đẹp 2019
 
Lỗi web bachawater
Lỗi web bachawaterLỗi web bachawater
Lỗi web bachawater
 
Bảng báo giá sản phẩm rèm bạch dương
Bảng báo giá sản phẩm rèm bạch dươngBảng báo giá sản phẩm rèm bạch dương
Bảng báo giá sản phẩm rèm bạch dương
 
Album sổ mẫu Rèm cửa Bạch Dương
Album sổ mẫu Rèm cửa Bạch DươngAlbum sổ mẫu Rèm cửa Bạch Dương
Album sổ mẫu Rèm cửa Bạch Dương
 
thong tin lam viec tren lamchame
thong tin lam viec tren lamchamethong tin lam viec tren lamchame
thong tin lam viec tren lamchame
 
Cách tắm cho bé vào mùa đông
Cách tắm cho bé vào mùa đôngCách tắm cho bé vào mùa đông
Cách tắm cho bé vào mùa đông
 
Giáo trình tự học illustrator cs6
Giáo trình tự học illustrator cs6  Giáo trình tự học illustrator cs6
Giáo trình tự học illustrator cs6
 
Nang luc truyen thong
Nang luc truyen thongNang luc truyen thong
Nang luc truyen thong
 
Huongdansudung izishop
Huongdansudung izishopHuongdansudung izishop
Huongdansudung izishop
 
Ho so nang luc cong ty
Ho so nang luc cong tyHo so nang luc cong ty
Ho so nang luc cong ty
 
seo contract
seo contractseo contract
seo contract
 
di google cong
di google congdi google cong
di google cong
 
E1 f4 bộ binh
E1 f4 bộ binhE1 f4 bộ binh
E1 f4 bộ binh
 
E2 f2 bộ binh
E2 f2 bộ binhE2 f2 bộ binh
E2 f2 bộ binh
 
E3 f1 bộ binh
E3 f1 bộ binhE3 f1 bộ binh
E3 f1 bộ binh
 
E2 f1 bộ binh
E2 f1 bộ binhE2 f1 bộ binh
E2 f1 bộ binh
 
E1 f1 bộ binh
E1 f1 bộ binhE1 f1 bộ binh
E1 f1 bộ binh
 
Nữ quái sân trườngtruonghocso.com
Nữ quái sân trườngtruonghocso.comNữ quái sân trườngtruonghocso.com
Nữ quái sân trườngtruonghocso.com
 
Những chàng trai xấu tính nguyễn nhật ánhtruonghocso.com
Những chàng trai xấu tính  nguyễn nhật ánhtruonghocso.comNhững chàng trai xấu tính  nguyễn nhật ánhtruonghocso.com
Những chàng trai xấu tính nguyễn nhật ánhtruonghocso.com
 
Những bài văn mẫu dành cho học sinh lớp 10truonghocso.com
Những bài văn mẫu dành cho học sinh lớp 10truonghocso.comNhững bài văn mẫu dành cho học sinh lớp 10truonghocso.com
Những bài văn mẫu dành cho học sinh lớp 10truonghocso.com
 

Bài toán số học liên quan tới lũy thữa

  • 1. www.VNMATH.com MỘT SỐ BÀI TOÁN SỐ HỌC LIÊN QUAN ĐẾN LŨY THỪA Phạm Văn Quốc (Trường THPT chuyên Khoa Học Tự Nhiên) Trong các kỳ thi học sinh giỏi chúng ta hay gặp các bài toán số học liên quan đến lũy thừa như chứng minh sự chia hết, chứng minh sự tồn tại hoặc tìm các số nguyên thỏa mãn điều kiện,... Trong những năm gần đây, dạng toán này cũng xuất hiện nhiều trong các đề thi quốc gia, đề thi chọn đội tuyển thi quốc tế (CĐT) của các nước, các đề dự tuyển và các đề thi Toán quốc tế (IMO). Đây là những bài toán hay và tất nhiên không dễ nếu không nắm được một số kỹ thuật cũng như nhận dạng được kiểu bài toán. Các lời giải thường sử dụng công cụ không khó nhưng chứa đựng nhiều sự tinh tế và sự linh hoạt trong vận dụng kiến thức. Bài viết dưới đây đề cập đến một số kiến thức cơ bản và kỹ năng liên quan đến các bài toán dạng này. I. Kiến thức cơ bản Trong phần này là một số kiến thức cơ bản nhưng chúng hay được dùng trong các dạng toán mà ta đang xét: công thức lũy thừa, số mũ "đúng", định lý Fermat, Định lý Euler, cấp của số nguyên và một số tính chất liên quan hay dùng . 1. Một số khai triển liên quan đến lũy thừa Định lý 1. Cho 𝑛 là số nguyên dương, khi đó với 𝑥, 𝑦 bất kỳ ta có ∘ 𝑥 𝑛 − 𝑦 𝑛 = (𝑥 − 𝑦) (𝑥 𝑛−1 + 𝑥 𝑛−2 𝑦 + 𝑥 𝑛−3 𝑦 2 + · · · + 𝑥𝑦 𝑛−2 + 𝑦 𝑛−1 ) , ∘ 𝑥 𝑛 + 𝑦 𝑛 = (𝑥 + 𝑦) (𝑥 𝑛−1 − 𝑥 𝑛−2 𝑦 + 𝑥 𝑛−3 𝑦 2 − · · · − 𝑥𝑦 𝑛−2 + 𝑦 𝑛−1 ) nếu 𝑛 lẻ, ∘ (𝑥 + 𝑦) 𝑛 = 𝑥 𝑛 + 𝐶 1 𝑥 𝑛−1 𝑦 + 𝐶 2 𝑥 𝑛−2 𝑦 2 + · · · + 𝐶 𝑛𝑛−1 𝑥𝑦 𝑛−1 + 𝑦 𝑛 . 𝑛 𝑛 Ta hãy bắt đầu bằng ví dụ sau. Ví dụ 1. (Romania 2002) Cho 𝑘, 𝑛 là các số nguyên dương với 𝑛 > 2. Chứng minh rằng phương trình 𝑥𝑛 − 𝑦 𝑛 = 2𝑘 không có nghiệm nguyên dương. Lời giải. Giả sử phương trình có nghiệm nguyên dương (𝑥, 𝑦). Nếu gcd (𝑥, 𝑦) = 𝑑 > 1 ⇒ 𝑑 | 2 𝑘 nên 𝑑 là lũy thừa của 2. Bằng cách chia hai vế cho 𝑑 𝑛 , ta có thể giả sử gcd (𝑥, 𝑦) = 1 và suy ra 𝑥, 𝑦 lẻ. 1
  • 2. www.VNMATH.com Nếu 𝑛 chẵn, 𝑛 = 2𝑚 ta có 𝑥 𝑛 − 𝑦 𝑛 = (𝑥 𝑚 − 𝑦 𝑚 ) (𝑥 𝑚 + 𝑦 𝑚 ) nên 𝑥 𝑚 − )︀ 𝑚 = (︀ 𝑦 𝑎 𝑚 𝑚 𝑘−𝑎 𝑚 𝑎−1 𝑘−2𝑎 2 , 𝑥 + 𝑦 =2 với 𝑎 là số nguyên dương. Khi đó 𝑥 = 2 1+2 mà 𝑥 lẻ nên 𝑎 = 1. Hơn nữa vì 𝑚 ≥ 2 nên 𝑥 𝑚 − 𝑦 𝑚 = (𝑥 − 𝑦) 𝑥 𝑚−1 + 𝑥 𝑚−2 𝑦 + · · · + 𝑦 𝑚−1 > 2 (︀ )︀ mâu thuẫn. Do đó 𝑛 là số lẻ. Ta có 𝑥 𝑛 − 𝑦 𝑛 = (𝑥 − 𝑦) 𝑥 𝑛−1 + 𝑥 𝑛−2 𝑦 + · · · + 𝑦 𝑛−1 . (︀ )︀ Nhưng do 𝑥, 𝑦 lẻ nên 𝑥 𝑛−1 + 𝑥 𝑛−2 𝑦 + · · · + 𝑦 𝑛−1 ≡ 𝑛 ≡ 1 (mod 2). Suy ra 𝑥 𝑛−1 + 𝑥 𝑛−2 𝑦 + · · · + 𝑦 𝑛−1 = 1, điều này là không thể vì 𝑥, 𝑦 nguyên dương và 𝑛 > 2. Vậy phương trình đã cho vô nghiệm. Lời giải của ví dụ này chủ yếu dùng công thức của hiệu hai lũy thừa và tính chất tích của hai số nguyên dương là lũy thừa của 2 thì mỗi số là lũy thừa của 2. Ví dụ tiếp theo cũng có ý giải cũng gần giống nhưng cần một chút khéo léo hơn. Ví dụ 2. (Dự tuyển IMO 2008) Cho 𝑛 là số nguyên dương và 𝑝 là số nguyên tố. Chứng minh rằng nếu 𝑎, 𝑏, 𝑐 là các số nguyên (không nhất thiết dương) thỏa mãn đẳng thức 𝑎 𝑛 + 𝑝𝑏 = 𝑏 𝑛 + 𝑝𝑐 = 𝑐 𝑛 + 𝑝𝑎 thì 𝑎 = 𝑏 = 𝑐. Lời giải. Ta chứng minh bài toán bằng phản chứng. Rõ ràng nếu hai trong ba số 𝑎, 𝑏, 𝑐 bằng nhau thì tất cả chúng bằng nhau. Giả sử cả ba số phân biệt đôi một, khi đó theo giả thiết ta có 𝑎𝑛− 𝑏𝑛 𝑏𝑛− 𝑐𝑛 𝑐𝑛− 𝑎𝑛 . . = −𝑝3 . 𝑎− 𝑏 𝑏− 𝑐 𝑐− 𝑎 Vì vế phải âm nên có ít nhất một trong ba số 𝑎, 𝑏, 𝑐 là số âm. Hơn nữa 𝑛 phải là số chẵn (nếu trái lại suy ra mỗi thừa số ở vế trái là số dương). +, Nếu 𝑝 là số lẻ, mà 𝑎𝑛− 𝑏𝑛 2 = 𝑎 𝑛−1 + 𝑎 𝑛−2 𝑏 + · · · + 𝑏 𝑛−1 . 𝑎− 𝑏 Suy ra 𝑎, 𝑏 khác tính chẵn lẻ, tức là 2 𝑎 − 𝑏. Tương tự 2 𝑏 − 𝑐, 2 𝑐 − 𝑎 đây là điều mâu thuẫn. +, Nếu 𝑝 = 2. Rõ ràng nếu một trong ba số 𝑎, 𝑏, 𝑐 bằng 0 thì cả ba số bằng 0. Xét |𝑎| , |𝑏| , |𝑐| ≥ 1. Đặt 𝑛 = 2𝑚 ta có 𝑎𝑛− 𝑏𝑛 𝑎2𝑚 − 𝑏2𝑚 𝐴 = = (𝑎 + 𝑏) 2 𝑎− 𝑏 𝑎 − 𝑏2 2
  • 3. www.VNMATH.com = (𝑎 + 𝑏) 𝑎2𝑚−2 + 𝑎2𝑚−4 𝑏2 + · · · + 𝑏2𝑚−2 . (︀ )︀ Dễ thấy nếu 𝑚 > 1 ta có ngay |𝐴| ≥ 4 (do 𝐴 ̸= 0) và tương tự suy ra mâu thuẫn vì tích của chúng là −8. Do đó 𝑚 = 1 ⇒ 𝑛 = 2 và ta thu được (𝑎 + 𝑏) (𝑏 + 𝑐) (𝑐 + 𝑎) = −8. Chú ý là do 𝑝 = 2 nên từ giả thiết ta có ngay 𝑎, 𝑏, 𝑐 cùng tính chẵn lẻ, suy ra 𝑎 + 𝑏, 𝑏 + 𝑐, 𝑐 + 𝑎 chẵn. Mà −8 = 2.2. (−2) nên dễ thấy hai trong ba số 𝑎, 𝑏, 𝑐 phải bằng nhau, suy ra 𝑎 = 𝑏 = 𝑐 (mâu thuẫn). Từ đó ta có điều phải chứng minh. 2. Số mũ "đúng" (exact exponent) Định nghĩa 2. Cho 𝑝 là số nguyên tố, 𝑎 là số nguyên và 𝛼 là số tự nhiên. Ta nói 𝑝 𝛼 là ước đúng (exact power) của 𝑎, và 𝛼 được gọi là số mũ đúng của 𝑝 trong khai triển của 𝑎, nếu 𝑝 𝛼 | 𝑎 và 𝑝 𝛼+1 𝑎. Khi đó ta viết 𝑝 𝛼 ‖ 𝑎 và ký hiệu 𝛼 = 𝑣 𝑝 (𝑎). Tính chất 3. Cho 𝑎, 𝑏 là các số nguyên, khi đó ta có ∘ Nếu 𝑝 𝛼 ‖ 𝑎 và 𝑝 𝛽 ‖ 𝑏 thì 𝑝 𝛼+𝛽 ‖ 𝑎𝑏, ∘ Nếu 𝑝 𝛼 ‖ 𝑎 thì 𝑝 𝑘𝛼 ‖ 𝑎 𝑘 , ∘ Nếu 𝑝 𝛼 ‖ 𝑎 và 𝑝 𝛽 ‖ 𝑏 với 𝑎 ̸= 𝑏 thì 𝑝min(𝛼,𝛽) ‖ 𝑎 + 𝑏. Ví dụ 3. Cho 𝑎, 𝑘 là các số nguyên dương và 𝑝 là số nguyên tố lẻ, 𝛼 ∈ N+ sao cho 𝑝 𝛼 ‖ 𝑎 − 1, khi đó với mọi số nguyên 𝛽 ≥ 0 thì 𝑝 𝛼+𝛽 ‖ 𝑎 𝑘 − 1 ⇔ 𝑝 𝛽 ‖ 𝑘. Lời giải. Ta chứng minh bằng quy nạp theo 𝛽. Nếu 𝛽 = 0, thì 𝑎𝑘 −1 = 𝑎 𝑘−1 + · · · + 𝑎 + 1 ≡ 𝑘 (mod 𝑝) do 𝑎 ≡ 1 (mod 𝑝) 𝑎−1 và suy ra nó không chia hết cho 𝑝. Bài toán đúng. Giả sử bài toán đúng với 𝛽 ≥ 0 nào đó và 𝑘 = 𝑝 𝛽+1 𝑡 với 𝑝 𝑡. Theo giả thiết 𝛽 quy nạp ta có 𝑎 𝑘/𝑝 = 𝑎 𝑝 𝑡 = 𝑚𝑝 𝛼+𝛽 + 1 với 𝑚 không chia hết cho 𝑝. Hơn nữa ta có )︀ 𝑝 𝑎𝑘 −1 = 𝑚𝑝 𝛼+𝛽 + 1 − 1 (︀ )︀ 𝑝 )︀2 𝑚𝑝 𝛼+𝛽 + · · · + 𝐶 2 𝑚𝑝 𝛼+𝛽 + 𝑚𝑝 𝛼+𝛽+1 . (︀ (︀ = 𝑝 𝑝 (𝑝 − 1) Vì 𝑝 | 𝐶 2 = 𝑝 , nên tất cả các số hạng trong khai triển trên, ngoại trừ số 2 hạng cuối cùng, đều chia hết cho 𝑝 𝛼+𝛽+2 . Từ đó ta có ngay 𝑝 𝛼+𝛽+1 ‖ 𝑎 𝑘 − 1. Theo nguyên lý quy nạp ta có điều phải chứng minh. Tương tự như ví dụ trên, ta có bài toán với 𝑝 = 2. Trong trường hợp này do 2 𝐶 𝑝 = 1 nên bài toán có sự thay đổi một chút. Ví dụ 4. Cho 𝑎, 𝑘 là các số nguyên dương, 𝛼 ∈ N+ sao cho 2 𝛼 ‖ 𝑎2 − 1, khi đó với mọi số nguyên 𝛽 ≥ 0 thì 2 𝛼+𝛽 ‖ 𝑎 𝑘 − 1 ⇔ 2 𝛽+1 ‖ 𝑘. 3
  • 4. www.VNMATH.com Lời giải. Rõ ràng ta chỉ cần xét với 𝑘 là số nguyên dương chẵn. Ta chứng minh bài toán bằng quy nạp theo 𝛽 ≥ 0. Với 𝛽 = 0 ⇔ 𝑘 = 2𝑛 (𝑛 là số lẻ). Khi đó 𝑛−1 𝑎 𝑘 − 1 ∑︁ 2(𝑛−1−𝑖) = 𝑎 ≡ 𝑛 ̸≡ 0 (mod 2 𝛼 ) . 𝑎2 − 1 𝑖=0 Bài toán đúng với 𝛽 = 0. Giả sử bài toán đúng với 𝛽 ≥ 0, đặt 𝑘 = 2𝑛 với 2 𝛽+1 ‖ 𝑛, khi đó 𝑎𝑘 −1 = 𝑎 𝑛 + 1 ≡ 2 (mod 4) 𝑎𝑛−1 (do 𝑎 lẻ và 𝑛 chẵn). Mà 2 𝛼+𝛽 ‖ 𝑎 𝑛 − 1. Suy ra bài toán đúng đến 𝛽 + 1. Ta có điều phải chứng minh. Chú ý: Hai ví dụ trên có thể tổng quát hơn, đó là định lý về số mũ đúng như dưới đây, cách chứng minh hoàn toàn tương tự bằng quy nạp. Định lý 4. (Lifting the Exponent Lemma) i. Với 𝑥, 𝑦 là các số nguyên (không nhất thiết dương), 𝑛 là số nguyên dương và 𝑝 là số nguyên tố lẻ sao cho 𝑝 | 𝑥 − 𝑦 và 𝑥, 𝑦 không chia hết cho 𝑝. Khi đó 𝑣 𝑝 (𝑥 𝑛 − 𝑦 𝑛 ) = 𝑣 𝑝 (𝑥 − 𝑦) + 𝑣 𝑝 (𝑛) . ii. Với 𝑥, 𝑦 là hai số nguyên lẻ và 𝑛 là số nguyên dương chẵn. Khi đó 𝑣2 (𝑥 𝑛 − 𝑦 𝑛 ) = 𝑣2 (𝑥 − 𝑦) + 𝑣2 (𝑥 + 𝑦) + 𝑣2 (𝑛) − 1. Nhận xét: +, Nếu 𝑛 lẻ bằng cách thay 𝑦 bởi −𝑦 thì ta cũng có đẳng thức tương tự trong phần i, 𝑣 𝑝 (𝑥 𝑛 + 𝑦 𝑛 ) = 𝑣 𝑝 (𝑥 + 𝑦) + 𝑣 𝑝 (𝑛) ; +, Trong phần ii, do 𝑥, 𝑦 lẻ nên một trong hai số 𝑣2 (𝑥 − 𝑦) , 𝑣2 (𝑥 + 𝑦) bằng 1. Ví dụ 5. (Nga 1996) Các số nguyên dương 𝑎, 𝑏, 𝑝, 𝑛, 𝑘 thỏa mãn 𝑎 𝑛 + 𝑏 𝑛 = 𝑝 𝑘 . Chứng minh rằng nếu 𝑛 > 1 là số lẻ và 𝑝 là số nguyên tố lẻ thì 𝑛 là lũy thừa của 𝑝. Lời giải. Ta có 𝑝 𝑘 = (𝑎 + 𝑏) (𝑎 𝑛−1 − 𝑎 𝑛−2 𝑏 + · · · + 𝑏 𝑛−1 ) suy ra 𝑎 + 𝑏 = 𝑝 𝑗 , 𝑗 ≥ 1. Giả sử 𝑝 𝑙 ‖ 𝑛 mà 𝑎 ≡ −𝑏 (mod 𝑝). Nên theo định lý 4 ta có 𝑣 𝑝 (𝑎 𝑛 + 𝑏 𝑛 ) = 𝑣 𝑝 (𝑎 + 𝑏) + 𝑣 𝑝 (𝑛) = 𝑗 + 𝑙, tức là 𝑝 𝑙+𝑗 ‖ (𝑎 𝑛 + 𝑏 𝑛 ) = 𝑝 𝑘 ⇒ 𝑙 = 𝑘 − 𝑗. Hơn nữa theo định lý 4 ta lại có (︁ 𝑘−𝑗 )︁ 𝑝 𝑝 𝑘−𝑗 = 𝑣 𝑝 (𝑎 + 𝑏) + 𝑣 𝑝 𝑝 𝑘−𝑗 = 𝑗 + 𝑘 − 𝑗 = 𝑘, (︀ )︀ 𝑣𝑝 𝑎 + 𝑏 4
  • 5. www.VNMATH.com 𝑘−𝑗 𝑘−𝑗 𝑘−𝑗 𝑘−𝑗 suy ra 𝑝 𝑘 ‖ 𝑎 𝑝 + 𝑏 𝑝 và 𝑎 𝑝 + 𝑏 𝑝 ‖ 𝑎 𝑛 + 𝑏 𝑛 (vì 𝑛 lẻ và 𝑝 𝑘−𝑗 ‖ 𝑛). Mà 𝑘−𝑗 𝑘−𝑗 𝑎 𝑛 + 𝑏 𝑛 = 𝑝 𝑘 nên ta có 𝑝 𝑘 = 𝑎 𝑝 + 𝑏 𝑝 = 𝑎 𝑛 + 𝑏 𝑛 và từ đó 𝑛 = 𝑝 𝑘−𝑗 . 3. Định lý Fermat (nhỏ) Định lý 5. (Định lý Fermat) Cho 𝑝 là một số nguyên tố thỏa mãn 𝑝 𝑎. Khi đó 𝑎 𝑝−1 ≡ 1 (mod 𝑝) . Đối với số nguyên 𝑎 bất kỳ, ta có 𝑎 𝑝 ≡ 𝑎 (mod 𝑝) . Ví dụ 6. Giả sử số nguyên tố 𝑝 có dạng 4𝑘 + 3 và là ước của (𝑥2 + 𝑦 2 ) thì 𝑝 | 𝑥, 𝑝 | 𝑦. Lời giải. Giả sử trái lại 𝑝 𝑥, 𝑝 𝑦 ⇒ gcd (𝑥, 𝑝) = gcd (𝑦, 𝑝) = 1. Do đó theo định lý Fermat ta có 𝑥 𝑝−1 ≡ 𝑦 𝑝−1 ≡ 1 (mod 𝑝). Trong khi đó từ giả thiết ta có 𝑝−1 𝑝−1 𝑥2 ≡ −𝑦 2 (mod 𝑝) ⇒ (𝑥2 ) 2 ≡ (−𝑦 2 ) 2 (mod 𝑝). Hay 𝑥 𝑝−1 ≡ −𝑦 𝑝−1 (mod 𝑝) ⇔ 1 ≡ −1 (mod 𝑝). Ta có mâu thuẫn vì 𝑝 là số nguyên tố lẻ. Nhận xét: Từ ví dụ này ta suy ra ngay với 𝑛 là số nguyên dương mọi ước nguyên tố lẻ của 𝑛2 + 1 đều có dạng 4𝑘 + 1. Và từ đó mọi ước dương của 𝑛2 + 1 đều có dạng 2 𝑖 (4𝑘 + 1) , 𝑖 = 0, 1. Ví dụ 7. (CĐT Mỹ 2008) Tìm tất cả các số nguyên dương 𝑛 sao cho 𝑛7 + 7 là số chính phương. Lời giải. Giả sử tồn tại 𝑚 ∈ N sao cho 𝑛7 + 7 = 𝑚2 . Khi đó ta có 𝑚2 + 112 = 𝑛7 + 27 = (𝑛 + 2) 𝑛6 − 2𝑛5 + · · · − 25 𝑛 + 26 . (︀ )︀ Rõ ràng 𝑚2 + 112 ≡ 1, 2 (mod 4) ⇒ 𝑛7 + 27 ≡ 1, 2 (mod 4). Từ đó dễ dàng thấy 𝑛 ≡ 1 (mod 4). Mà theo đẳng thức trên ta có (𝑛 + 2) | 𝑚2 + 112 suy ra 𝑚2 + 112 phải có một ước nguyên tố 𝑝 có dạng 4𝑖 + 3. Từ đó 112 ≡ −𝑚2 (mod 𝑝). Nếu 𝑝 ̸= 11 thì theo định lý Fermat ta có 11 𝑝−1 ≡ 𝑚 𝑝−1 ≡ 1 (mod 𝑝) nhưng trong khi đó )︀ 𝑝−1 11 𝑝−1 ≡ −𝑚2 2 = −𝑚 𝑝−1 (mod 𝑝) (︀ mâu thuẫn. Suy ra 𝑝 = 11. Do đó 𝑚 = 11𝑘, 𝑘 ∈ N và 11 | (𝑛 + 2) . Hơn nữa từ đó ta lại có 𝑛6 − 2𝑛5 + · · · − 25 𝑛 + 26 ≡ 7.26 ≡ 8 (mod 11) . Suy ra 112 | (𝑛 + 2) ⇒ 𝑛 = 112 ℎ − 2 và ℎ là ước dương của 𝑘 2 + 1. Theo ví dụ 6 các ước nguyên tố lẻ của 𝑘 2 + 1 chỉ có dạng 4𝑖 + 1, tức là ℎ chỉ có dạng 2 𝑗 (4𝑖 + 1) , 𝑗 = 0, 1. Khi đó 𝑛 = 112 ℎ − 2 ≡ 0; 3 (mod 4). Ta có mâu thuẫn vì 𝑛 ≡ 1 (mod 4). Do đó không tồn tại 𝑛 thỏa mãn đầu bài. 5
  • 6. www.VNMATH.com 4. Hàm Euler Định nghĩa 6. Cho 𝑛 là số nguyên dương. Hàm Euler 𝜙 xác định trên tập các số nguyên dương như sau: 𝜙 (𝑛) là số các số nguyên dương nhỏ hơn 𝑛 và nguyên tố cùng nhau với 𝑛. Quy ước 𝜙 (1) = 1. Tính chất 7. Hàm 𝜙 (𝑛) có tính chất nhân theo nghĩa: Nếu 𝑎, 𝑏 là hai số nguyên tố cùng nhau thì 𝜙 (𝑎𝑏) = 𝜙 (𝑎) 𝜙 (𝑏) . Tính chất 8. Giả sử 𝑛 = 𝑝1𝛼1 𝑝2𝛼2 . . . 𝑝 𝑘𝛼 𝑘 là phân tích tiêu chuẩn của 𝑛 > 1. Khi đó (︂ )︂ (︂ )︂ (︂ )︂ 1 1 1 𝜙 (𝑛) = 𝑛 1 − 1− ··· 1 − . 𝑝1 𝑝2 𝑝𝑘 Định lý 9. (Định lý Euler ) Cho 𝑎, 𝑛 là các số nguyên, 𝑛 > 1, (𝑎, 𝑛) = 1. Khi đó 𝑎 𝜙(𝑛) ≡ 1 (mod 𝑛) . Với 𝑎, 𝑛 là hai số nguyên dương bất kỳ ta có 𝑎 𝑛 ≡ 𝑎 𝑛−𝜙(𝑛) (mod 𝑛) . Nhận xét: Định lý Fermat là trường hợp riêng của định lý Euler trong trường hợp 𝑛 là số nguyên tố. Bổ đề 10. Với 𝑎, 𝑏 là các số nguyên dương 𝑎, 𝑏 thì với 𝑛 đủ lớn ta có 𝑏 𝑛+𝜙(𝑎) ≡ 𝑏 𝑛 (mod 𝑎) {︀ }︀ (cụ thể hơn 𝑛 ≥ max 𝑣 𝑝 𝑖 (𝑎) + 1, 𝑝 𝑖 là ước nguyên tố của 𝑎 ). Chứng minh của bổ đề này có thể xem trong bài 7 dưới đây. Ví dụ 8. (IMO 1971) Chứng minh rằng dãy số 2 𝑛 − 3 (𝑛 > 1) chứa một dãy con vô hạn gồm các số đôi một nguyên tố cùng nhau. Lời giải. Ta xây dựng dãy con bằng quy nạp. Giả sử ta đã xây dựng được dãy các số 𝑎1 = 2 𝑛1 − 1, 𝑎2 = 2 𝑛2 − 1, ..., 𝑎 𝑘 = 2 𝑛 𝑘 − 1, ở đó 2 = 𝑛1 < 𝑛2 < · · · < 𝑛 𝑘 , mà các phần tử đôi một nguyên tố cùng nhau. Ta sẽ xây dựng 𝑎 𝑘+1 = 2 𝑛 𝑘+1 − 3 như sau: Đặt 𝑠 = 𝑎1 𝑎2 ...𝑎 𝑘 ta có 𝑠 là số lẻ nên theo định lý Euler ta có 𝑠 | 2 𝜙(𝑠) − 1 ⇒ 2 𝜙(𝑠) − 1 = 𝑞𝑠, 𝑞 ∈ N. Khi đó ta có 2 𝜙(𝑠)+2 − 3 = 4𝑞𝑠 − 1 là nguyên tố cùng nhau với 𝑠, nên ta có thể chọn 𝑛 𝑘+1 = 𝜙 (𝑠) + 2. Rõ ràng 𝑛 𝑘+1 > 𝑛 𝑘 và dãy số 𝑎1 , 𝑎2 , ..., 𝑎 𝑘 , 𝑎 𝑘+1 đôi một nguyên tố cùng nhau, từ đó ta có điều phải chứng minh. Ví dụ 9. (Mỹ 1991) Chứng minh rằng với mọi số nguyên 𝑛 ≥ 1, dãy số sau 2 22 2, 22 , 22 , 22 , . . . (mod 𝑛) là dãy hằng số kể từ một lúc nào đó. 6
  • 7. www.VNMATH.com Lời giải. Ta chứng minh bằng quy nạp theo 𝑛. Với 𝑛 = 1 bài toán hiển nhiên đúng. Giả sử bài toán đúng đến 𝑘 − 1, (𝑘 ≥ 2). +, Nếu 𝑘 chẵn, 𝑘 = 2 𝑎 𝑏 với 𝑏 lẻ. Khi đó theo giả thiết quy nạp thì dãy đã cho là hằng số từ lúc nào đó mô-đun 𝑏. Mà rõ ràng dãy này là dãy 0 mô-đun 2 𝑎 kể từ lúc nào đó. Vì thế theo mô-đun 𝑘 nó cũng là dãy hằng số kể từ một lúc nào đó. +, Nếu 𝑘 lẻ, theo định lý Euler ta có 2 𝜙(𝑘) ≡ 1 (mod 𝑘). Mà theo giả thiết quy 2 nạp dãy 1, 2, 22 , 22 , . . . (dãy các số mũ) là hằng số kể từ một lúc nào đó mô-đun 2 22 𝜙 (𝑘). Vì thế 2, 22 , 22 , 22 , . . . (mod 𝑘) là dãy hằng số kể từ một lúc nào đó. Theo nguyên lý quy nạp ta có điều phải chứng minh. 5. Cấp (order) của một số nguyên Định nghĩa 11. Cho 𝑎, 𝑛 là hai số nguyên dương và 𝑎 là số nguyên bất kỳ thỏa mãn (𝑎, 𝑛) = 1. Số nguyên dương ℎ nhỏ nhất sao cho 𝑎ℎ ≡ 1 (mod 𝑛) được gọi là cấp của 𝑎 mô-đun 𝑛, ký hiệu ord 𝑛 (𝑎) . Bổ đề 12. Cho 𝑚, 𝑛 là các số nguyên dương và 𝑎 là số nguyên khác 1. Khi đó ⃒ ⃒ gcd (𝑎 𝑚 − 1, 𝑎 𝑛 − 1) = ⃒ 𝑎gcd(𝑚,𝑛) − 1⃒ . Định lý 13. Cho 𝑎, 𝑛 là các số nguyên nguyên tố cùng nhau với 𝑛 > 0 khi đó ord 𝑛 (𝑎) | 𝜙 (𝑛). Định nghĩa 14. Nếu 𝑟, 𝑛 là các số nguyên tố cùng nhau với 𝑛 > 0 và nếu ord 𝑛 𝑟 = 𝜙 (𝑛) thì 𝑟 được gọi là căn nguyên thủy mô-đun 𝑛. Định lý 15. Nếu 𝑟, 𝑛 là các số nguyên dương nguyên tố cùng nhau và nếu 𝑟 là {︀ }︀ căn nguyên thủy mô-đun 𝑛. Khi đó các số nguyên 𝑟, 𝑟2 , ..., 𝑟 𝜙(𝑛) tạo thành một hệ thặng dư thu gọn mô-đun 𝑛. Định lý 16. (Sự tồn tại của căn nguyên thủy) Mọi số nguyên tố đều có căn nguyên thủy. Tổng quát hơn: số nguyên dương 𝑛 > 1 có căn nguyên thủy khi và chỉ khi 𝑛 = 2, 4, 𝑝 𝑘 hoặc 2𝑝 𝑘 trong đó 𝑝 là số nguyên tố lẻ và 𝑘 là số nguyên dương. Ví dụ 10. Cho 𝑛 là hai số nguyên dương và 𝑝 là số nguyên tố thỏa. Chứng minh rằng nếu 𝑚 là ước nguyên tố lẻ của 𝑛 𝑝 + 1 thì 2𝑝 | 𝑚 − 1 hoặc 𝑚 | 𝑛2 − 1. Lời giải. Từ giả thiết 𝑚 | 𝑛 𝑝 + 1 ⇒ 𝑛 𝑝 ≡ −1 ̸≡ 1 (mod 𝑚) vì 𝑚 > 2. Suy ra 𝑛2𝑝 ≡ 1 (mod 𝑚). Đặt 𝑑 = ord 𝑚 𝑛 khi đó 𝑑 là ước của 2𝑝 nhưng không là ước của 𝑝. Mà 𝑝 là số nguyên tố suy ra 𝑑 = 2 hoặc 𝑑 = 2𝑝. +, Nếu 𝑑 = 2 suy ra 𝑛2 ≡ 1 (mod 𝑚) hay 𝑚 | 𝑛2 − 1. +, Nếu 𝑑 = 2𝑝 ⇒ 2𝑝 | 𝑚 − 1 vì theo định lý Fermat ta có 𝑑 | 𝑚 − 1 (đpcm). Ví dụ 11. (Dự tuyển IMO 2006) Tìm tất cả các nghiệm nguyên của phương trình 𝑥7 − 1 = 𝑦 5 − 1. 𝑥−1 7
  • 8. www.VNMATH.com Lời giải. 𝑥7 − 1 Gọi 𝑝 là một ước nguyên tố bất kỳ của = 𝑥6 + 𝑥5 + · · · + 𝑥 + 1. Ta có 𝑥−1 hai trường hợp: 𝑥7 − 1 +, Nếu 𝑝 | (𝑥 − 1), suy ra ≡ 1 + 1 + · · · + 1 + 1 ≡ 7 (mod 𝑝) ⇒ 𝑝 = 7; 𝑥−1 +, Nếu 𝑝 (𝑥 − 1), ta có ngay ord 𝑝 𝑥 = 7. Từ đó suy ra 𝑝 ≡ 1 (mod 7) . 𝑥7 − 1 Vậy ta có mọi ước nguyên dương 𝑑 của thỏa mãn 𝑑 ≡ 0; 1 (mod 7). 𝑥−1 Bây giờ giả sử (𝑥, 𝑦) là một nghiệm của phương trình đã cho. Mà 𝑦 5 − 1 = (𝑦 − 1) (𝑦 4 + 𝑦 3 + 𝑦 2 + 𝑦 + 1) suy ra 𝑦 − 1 ≡ 0; 1 (mod 7) 4 3 2 𝑦 + 𝑦 + 𝑦 + 𝑦 + 1 ≡ 0; 1 (mod 7) . Ta có mâu thuẫn vì khi đó 𝑦 ≡ 1; 2 (mod 7) ⇒ 𝑦 4 + 𝑦 3 + 𝑦 2 + 𝑦 + 1 ≡ 5; 3 (mod 7). Vậy bài toán vô nghiệm. Ví dụ 12. (CĐT Trung Quốc 2006) Tìm tất cả các cặp số nguyên (𝑎, 𝑛) sao cho (𝑎 + 1) 𝑛 − 𝑎 𝑛 là số nguyên. 𝑛 Lời giải. Xét 𝑛 ≥ 2, giả sử 𝑝 là ước nguyên tố nhỏ nhất của 𝑛. Khi đó 𝑝 | (𝑎 + 1) 𝑛 −𝑎 𝑛 ⇒ (𝑝, 𝑛) = 1 nên tồn tại 𝑏 sao cho 𝑎𝑏 ≡ 1 (mod 𝑝). Ta có (𝑎 + 1) 𝑛 ≡ 𝑎 𝑛 (mod 𝑝) ⇒ ((𝑎 + 1) 𝑏) 𝑛 ≡ 1 (mod 𝑝). Do đó 𝑑 | 𝑛 với 𝑑 = ord 𝑝 (𝑎𝑏 + 𝑏). Hơn nữa theo định lý Fermat ta có ((𝑎 + 1) 𝑏) 𝑝−1 ≡ 1 (mod 𝑝) ⇒ 𝑑 | 𝑝 − 1 ⇒ gcd (𝑑, 𝑛) = 1 (theo định nghĩa của 𝑝). Tức là 𝑑 = 1, suy ra 𝑎 + 1 ≡ 𝑎 (mod 𝑝). Mâu thuẫn, do đó 𝑛 = 1. Chú ý : Ta có thể trình bày lời giải bằng căn nguyên thủy như sau: Gọi 𝑔 là căn nguyên thủy mô-đun 𝑝. Theo giả thiết ta dễ thấy rằng 𝑎, 𝑎 + 1 không chia hết cho 𝑝, suy ra (𝑎 + 1, 𝑝) = (𝑎, 𝑝) = 1 ⇒ 𝑎 + 1 ≡ 𝑔 𝑘 , 𝑎 ≡ 𝑔 ℎ (mod 𝑝) với 𝑘 ̸= ℎ. Thay vào phương trình đã cho suy ra 𝑔 𝑘𝑛 ≡ 𝑔 ℎ𝑛 (mod 𝑝) ⇒ 𝑝 − 1 | 𝑛 (𝑘 − ℎ). Mà 𝑝 là ước nguyên tố nhỏ nhất của 𝑛 nên (𝑝 − 1, 𝑛) = 1 ⇒ 𝑝 − 1 | 𝑘 − ℎ. Theo định lý Fermat suy ra 𝑝 | 𝑔 𝑝−1 − 1 | 𝑔 𝑘 − 𝑔 ℎ ⇒ 𝑝 = 1 mâu thuẫn. 6. Một số hệ quả hay dùng khác i. Nếu 𝑝 | 𝑥 thì (𝑥 + 𝑦) 𝑛 ≡ 𝑛𝑥𝑦 𝑛−1 + 𝑦 𝑛 (mod 𝑝2 ) và nếu 𝑎 ≡ 𝑏 (mod 𝑝) thì 𝑎 = 𝑘𝑝 + 𝑏 ta có 𝑎 𝑛 ≡ 𝑛𝑘𝑝𝑏 𝑛−1 + 𝑏 𝑛 (mod 𝑝2 ) . ii . Cho 𝑚 là số nguyên dương và 𝑎, 𝑏 là hai số nguyên tố cùng nhau với 𝑚. Nếu 𝑥, 𝑦 là hai số nguyên thỏa mãn 𝑎 𝑥 ≡ 𝑏 𝑥 (mod 𝑚) và 𝑎 𝑦 ≡ 𝑏 𝑦 (mod 𝑚). Khi đó 𝑎gcd(𝑥,𝑦) ≡ 𝑏gcd(𝑥,𝑦) (mod 𝑚). iii . Cho 𝑝 là một số nguyên tố lẻ. Khi đó a) nếu 𝑎 ≥ 2 thì 𝑎 𝑝 − 1 có một ước nguyên tố mà không là ước của 𝑎 − 1; b) nếu 𝑎 ≥ 2, 𝑝 ̸= 3 hoặc 𝑎 > 2 thì 𝑎 𝑝 + 1 có một ước nguyên tố mà không là ước của 𝑎 + 1. 8
  • 9. www.VNMATH.com Hướng dẫn. Giả sử trái lại suy ra mọi ước của 𝐴 = 𝑎 𝑝−1 + 𝑎 𝑝−2 + · · · + 𝑎 + 1 đều là ước của 𝑎 − 1 mà 𝐴 = (𝑎 − 1) 𝐵 + 𝑝 ⇒ gcd (𝑎 − 1, 𝑎 𝑝−1 + 𝑎 𝑝−2 + · · · + 𝑎 + 1) | 𝑝 ⇒ 𝐴 là lũy thừa của 𝑝. Do đó 𝑝2 ‖ 𝑎 𝑝 − 1 ⇒ 𝐴 = 𝑝 − 1 mâu thuẫn. iv. Nếu 𝑎 là số nguyên không chia hết cho số nguyên tố 𝑝 và có một số nguyên dương 𝑘 thỏa mãn 𝑎 𝑘 ≡ −1 (mod 𝑝) khi đó nếu 𝑑 = ord 𝑝 𝑎 thì ℎ = 𝑑/2 là số nguyên dương nhỏ nhất thỏa mãn 𝑎ℎ ≡ −1 (mod 𝑝) . 𝑎2𝑘 𝑘 (︀ ℎ Hướng dẫn. )︀ ≡ 1 (mod 𝑝) ⇒ 𝑑 | 2𝑘 mà 𝑎 ≡ −1 (mod 𝑝) nên 𝑑 = 2ℎ ⇒ )︀ (︀ ℎ 𝑎 − 1 𝑎 + 1 ≡ 0 (mod 𝑝) . 𝑥𝑝 −1 v. Nếu 𝑥 là số nguyên dương và 𝑝, 𝑞 là hai số nguyên tố sao cho 𝑞 | thì 𝑥−1 𝑞 = 𝑝 hoặc 𝑞 ≡ 1 (mod 𝑝) . (︀ )︀ (︀ )︀ vi. Số nguyên dương 𝑑 nhỏ nhất thỏa mãn 2 𝑑 ≡ 1 mod 3 𝑘 là 𝑑 = 𝜙 3 𝑘 = 2.3 𝑘−1 . Nói cách khác 2 là căn nguyên thủy mô-đun 3 𝑛 . Hướng dẫn. Nếu 𝑝 là số nguyên tố lẻ và 𝑟 là căn nguyên thủy mô-đun 𝑝2 thì 𝑟 là căn nguyên thủy mô-đun 𝑝 𝑘 với mọi số nguyên dương 𝑘. II. Bài tập áp dụng Bài 1. (IMO 2005) Xét dãy số 𝑎1 , 𝑎2 , ... xác định bởi công thức 𝑎 𝑛 = 2 𝑛 + 3 𝑛 + 6 𝑛 − 1 (𝑛 = 1, 2, ...) . Xác định tất cả các số nguyên dương mà chúng nguyên tố cùng nhau với mọi số hạng của dãy trên. Lời giải. Ta sẽ chứng minh với mọi số nguyên tố 𝑝 luôn tồn tại một số hạng 𝑎 𝑚 của dãy sao cho 𝑝 | 𝑎 𝑚 , từ đó suy ra tất cả các số cần tìm là 1. +, Với 𝑝 = 2, 3 rõ ràng 𝑎2 = 48 từ đó 𝑝 | 𝑎2 . +, Xét 𝑝 > 3. Áp dụng định lý Fermat ta có 6𝑎 𝑝−2 = 3.2 𝑝−1 + 2.3 𝑝−1 + 6 𝑝−1 − 6 ≡ 3 + 2 + 1 − 6 ≡ 0 (mod 𝑝) . Từ đó 𝑝 | 𝑎 𝑝−2 ⇒ gcd (𝑝, 𝑎 𝑝−2 ) = 𝑝 > 1. Vậy chỉ có số 1 thỏa mãn bài toán. Bài 2. (Dự tuyển IMO 2005) Giả sử 𝑎, 𝑏 là hai số nguyên dương sao cho 𝑎 𝑛 + 𝑛 là ước của 𝑏 𝑛 + 𝑛 với mọi số nguyên dương 𝑛. Chứng minh rằng 𝑎 = 𝑏. Lời giải. Giả sử 𝑎 ̸= 𝑏, khi đó từ giả thiết dễ thấy 𝑏 > 𝑎. Chọn 𝑝 là số nguyên tố lớn hơn 𝑏 và lấy 𝑛 = (𝑎 + 1) (𝑝 − 1) + 1. Theo cách chọn này ta có 𝑛 ≡ 1 (mod 𝑝 − 1) và 𝑛 ≡ −𝑎 (mod 𝑝). Khi đó theo định lý Fermat ta có )︀ 𝑎+1 𝑟 𝑛 ≡ 𝑟 𝑟 𝑝−1 (︀ ≡ 𝑟 (mod 𝑝) ∀𝑟 ∈ Z. Ta lại có 𝑎 𝑛 + 𝑛 ≡ 𝑎 − 𝑎 ≡ 0 (mod 𝑝) ⇒ 𝑝 | 𝑎 𝑛 + 𝑛. Hơn nữa 𝑏 𝑛 + 𝑛 ≡ 𝑏 − 𝑎 ≡ 0 (mod 𝑝) ⇒ 𝑝 | 𝑏 − 𝑎. Điều này mâu thuẫn vì 𝑝 > 𝑏. Do đó 𝑎 = 𝑏 thỏa mãn. Bài 3. (Bulgaria 1995) Tìm tất cả các số nguyên tố 𝑝, 𝑞 sao cho 𝑝𝑞 là ước của (5 𝑝 − 2 𝑝 ) (5 𝑞 − 2 𝑞 ). 9
  • 10. www.VNMATH.com Lời giải. Do tính đối xứng nên ta có thể giả sử 𝑝 ≤ 𝑞 mà (5 𝑝 − 2 𝑝 ) (5 𝑞 − 2 𝑞 ) là số lẻ nên ta có 5 ≤ 𝑝 ≤ 𝑞. Để ý rằng nếu số nguyên tố 𝑘 là ước của 5 𝑘 − 2 𝑘 thì theo định lý Fermat ta có ngay 3 ≡ 5 − 2 ≡ 5 𝑘 − 2 𝑘 (mod 𝑘) ⇒ 𝑘 = 3. Giả sử 𝑝 > 3, theo nhận xét trên ta có 𝑝 là ước của 5 𝑞 −2 𝑞 hay 5 𝑞 ≡ 2 𝑞 (mod 𝑝). Lại theo định lý Fermat thì 5 𝑝−1 ≡ 2 𝑝−1 (mod 𝑝). Do đó 5gcd(𝑝−1,𝑞) ≡ 2gcd(𝑝−1,𝑞) (mod 𝑝) . Mà 𝑞 ≥ 𝑝 ⇒ gcd (𝑝 − 1, 𝑞) = 1 do đó ta có 5 ≡ 2 (mod 𝑝) ⇒ 𝑝 = 3 mâu thuẫn. Suy ra 𝑝 = 3. Nếu 𝑞 > 3 suy ra 𝑞 là ước của 5 𝑝 − 2 𝑝 = 53 − 23 = 9.13 ⇒ 𝑞 = 13 thỏa mãn. Vậy tất cả các cặp (𝑝, 𝑞) cần tìm là: (3, 3) ; (3, 13) ; (13, 3). Bài 4. (Dự tuyển IMO 2003) Cho 𝑝 là một số nguyên tố. Chứng minh rằng tồn tại một số nguyên tố 𝑞 sao cho với mọi số nguyên 𝑛, số 𝑛 𝑝 − 𝑝 không chia hết cho 𝑞. Lời giải. Ta có 𝑝𝑝 −1 = 1 + 𝑝 + 𝑝2 + · · · 𝑝 𝑝−1 ≡ 𝑝 + 1 mod 𝑝2 , (︀ )︀ 𝑝−1 𝑝𝑝 −1 suy ra có ít nhất một ước nguyên tố của không đồng dư 1 mô-đun 𝑝2 . Gọi 𝑝−1 số nguyên tố này là 𝑞 và ta sẽ chỉ ra đây là số 𝑞 cần tìm. Thật vậy, giả sử tồn tại số nguyên 𝑛 sao cho 𝑛 𝑝 ≡ 𝑝 (mod 𝑞). Khi đó, theo 2 cách chọn số 𝑞 ta có 𝑛 𝑝 ≡ 𝑝 𝑝 ≡ 1 (mod 𝑞). Mặt khác, theo định lý Fermat, 𝑛 𝑞−1 ≡ 1 (mod 𝑞), vì 𝑞 là số nguyên tố. Hơn nữa ta có 𝑝2 𝑞 − 1 nên (𝑝2 , 𝑞 − 1) | 𝑝 ⇒ 𝑛 𝑝 ≡ 1 (mod 𝑞). Suy ra 𝑝 ≡ 1 (mod 𝑞). Khi đó ta thu được 1 + 𝑝 + · · · + 𝑝 𝑝−1 ≡ 𝑝 (mod 𝑞) . Cùng với định nghĩa của 𝑞 ta có ngay 𝑝 ≡ 0 (mod 𝑞), đây là điều mâu thuẫn. Ta có điều phải chứng minh. Nhận xét: Để dùng định lý Fermat, ta sẽ tìm cách chọn số 𝑞 có dạng 𝑝𝑘 + 1. Khi đó ∃𝑛 𝑛 𝑝 ≡ 𝑝 (mod 𝑞) ⇔ 𝑝 𝑘 ≡ 1 (mod 𝑞) , tức là ∀𝑛 𝑛 𝑝 ̸≡ 𝑝 (mod 𝑞) ⇔ 𝑝 𝑘 ̸≡ 1 (mod 𝑞) . Do đó để tìm 𝑞 ta sẽ chọn 𝑞 là một ước nguyên tố của 𝑝 𝑝 − 1 và ta có cách chọn như trên. Bài 5. (Dự tuyển IMO 2005) Tìm tất cả các số nguyên dương 𝑛 > 1 sao cho tồn tại duy nhất số nguyên 𝑎 với 0 < 𝑎 ≤ 𝑛! thỏa mãn 𝑎 𝑛 + 1 chia hết cho 𝑛!. 10
  • 11. www.VNMATH.com Lời giải. Ta sẽ chỉ ra rằng tất cả các số 𝑛 cần tìm là các số nguyên tố. Thật vậy: +, Với 𝑛 = 2 khi đó rõ ràng chỉ có duy nhất 𝑎 = 1 thỏa mãn yêu cầu. +, Với 𝑛 > 2 và 𝑛 chẵn ta có 4 | 𝑛! nhưng 𝑎 𝑛 + 1 ≡ 1, 2 (mod 4) , khi đó không tồn tại 𝑎 thỏa mãn. +, Xét 𝑛 lẻ. Ta có (𝑛! − 1) 𝑛 + 1 ≡ (−1) 𝑛 + 1 ≡ 0 (mod 𝑛!) . (*) Suy ra nếu 𝑛 là hợp số và 𝑑 là một ước nguyên tố của 𝑛 thì ta có (︂ )︂ 𝑛 𝑛 ∑︁ (𝑛!) 𝑘 𝑛! −1 +1= 𝐶 𝑛𝑘 𝑘 ≡ 0 (mod 𝑛!) 𝑑 𝑘=1 𝑑 bởi vì rõ ràng 𝑑2 | 𝑛! nên mỗi số hạng của tổng trên đều chia hết cho 𝑛!. Do vậy trường hợp này cũng không thỏa mãn. +, Xét 𝑛 là số nguyên tố lẻ và giả sử 𝑎 là số nguyên thỏa mãn 0 < 𝑎 ≤ 𝑛!, 𝑎 𝑛 +1 chia hết cho 𝑛!. Do 𝑛 lẻ nên ta có khai triển 𝑎 𝑛 + 1 = (𝑎 + 1) 𝑎 𝑛−1 − 𝑎 𝑛−2 + · · · + 𝑎2 − 𝑎 + 1 . (︀ )︀ Ta sẽ chứng minh 𝑎 = 𝑛! − 1 là giá trị duy nhất thỏa mãn. Xét 𝑝 là số nguyên tố, 𝑝 ≤ 𝑛. Ta có 𝑝 | 𝑎 𝑛 + 1 ⇒ 𝑝 | (−𝑎) 𝑛 − 1. Theo định lý Fermat ta lại có 𝑝 | (−𝑎) 𝑝−1 − 1 nên 𝑝 | (−𝑎)(𝑛,𝑝−1) − 1 = −𝑎 − 1. Suy ra 𝑛 | 𝑎 + 1 (𝑝 = 𝑛) và nếu 𝑝 < 𝑛 thì 𝑎 𝑛−1 − 𝑎 𝑛−2 + · · · + 𝑎2 − 𝑎 + 1 ≡ 𝑛 (mod 𝑝) ̸≡ 0 (mod 𝑝) do (𝑛, 𝑝) = 1. Từ đó ta thu được 𝑎 𝑛−1 − 𝑎 𝑛−2 + · · · + 𝑎2 − 𝑎 + 1 và (𝑛 − 1)! nguyên tố cùng nhau, suy ra (𝑛 − 1)! | 𝑎 + 1. Mà 𝑛 | 𝑎 + 1 nên 𝑛! | 𝑎 + 1. Do điều kiện 0 < 𝑎 ≤ 𝑛! ⇒ 𝑛! = 𝑎 + 1 ⇒ 𝑎 = 𝑛! − 1. Cùng với (*) ta có 𝑎 = 𝑛! − 1 là giá trị duy nhất thỏa mãn. Vậy tất các số nguyên dương 𝑛 cần tìm là tất cả các số nguyên tố. Bài 6. (Dự tuyển IMO 2006) Chứng minh rằng với mọi số nguyên dương 𝑛 luôn tồn tại số nguyên 𝑚 sao cho 2 𝑚 + 𝑚 chia hết cho 𝑛. Lời giải. Ta sẽ chứng minh bằng quy nạp theo 𝑛 rằng luôn tồn tại số nguyên dương 𝑚 đủ lớn để 2 𝑚 ≡ −𝑚 (mod 𝑛) . +, Với 𝑛 = 1 hiển nhiên đúng. +, Xét 𝑛 > 1. Theo tính chất của hàm Euler ta có dãy các số mũ của 2 theo mô-đun 𝑛 là tuần hoàn với chu kỳ là bội của 𝜙 (𝑛). Do đó với 𝑥, 𝑦 đủ lớn và 𝑥 ≡ 𝑦 (mod 𝜙 (𝑛)) thì 2 𝑥 ≡ 2 𝑦 (mod 𝑛). Chọn số 𝑚 có dạng 𝑚 ≡ −2 𝑘 (mod 𝑛𝜙 (𝑛)). Khi đó 2 𝑚 ≡ −𝑚 (mod 𝑛) ⇔ 2 𝑚 ≡ 2 𝑘 (mod 𝑛). Theo giả thiết quy nạp ta có thể chọn được số 𝑘 đủ lớn để 2 𝑘 ≡ −𝑘 (mod 𝜙 (𝑛)) ⇒ −2 𝑘 ≡ 𝑚 ≡ 𝑘 (mod 𝜙 (𝑛)). Từ đó ta thu được 2 𝑚 ≡ −𝑚 (mod 𝑛). Theo nguyên lý quy nạp ta có điều phải chứng minh. Bài 7. (CĐT Mỹ 2007) Hỏi có tồn tại hay không hai số nguyên dương 𝑎, 𝑏 sao cho 𝑎 không là ước của 𝑏 𝑛 − 𝑛 với mọi số nguyên dương 𝑛. 11
  • 12. www.VNMATH.com Lời giải. Trước hết ta chứng minh rằng với mọi số nguyên dương 𝑎, 𝑏 thì với 𝑛 đủ lớn ta có 𝑏 𝑛+𝜙(𝑎) ≡ 𝑏 𝑛 (mod 𝑎) . (*) Thật vậy, giả sử 𝑎 = 𝑝1𝛼1 𝑝2𝛼2 . . . 𝑝 𝑘𝛼 𝑘 , trong đó 𝑝1 , 𝑝2 , ..., 𝑝 𝑘 là các số nguyên tố phân biệt. Vì 𝜙 là hàm nhân tính nên ta có 𝜙 (𝑎) = 𝜙 (𝑝1𝛼1 ) 𝜙 (𝑝2𝛼2 ) . . . 𝜙 (𝑝 𝑘𝛼 𝑘 ) = 𝑝1𝛼1 − 𝑝1𝛼1 −1 𝑝2𝛼2 − 𝑝2𝛼2 −1 . . . 𝑝 𝑘𝛼 𝑘 − 𝑝 𝑘𝛼 𝑘 −1 (︀ )︀ (︀ )︀ (︀ )︀ (︂ )︂ (︂ )︂ (︂ )︂ 1 1 1 = 𝑎 1− 1− ··· 1 − . 𝑝1 𝑝2 𝑝𝑘 Khi đó ta có 𝜙 (𝑝 𝑖𝛼 𝑖 ) | 𝜙 (𝑎) và 𝜙 (𝑎) < 𝑎. Với mỗi 𝑝 𝑖 , ta có hai trường hợp: +, Nếu 𝑝 𝑖 là ước của 𝑏 ⇒ 𝑏 𝑛 ≡ 0 (mod 𝑝 𝑖𝛼 𝑖 ) với 𝑛 ≥ 𝛼 𝑖 + 1. Suy ra 𝑏 𝑛+𝜙(𝑎) ≡ 𝑏 𝑛 𝑏 𝜙(𝑎) ≡ 𝑏 𝑛 ≡ 0 (mod 𝑝 𝑖𝛼 𝑖 ) với 𝑛 ≥ 𝛼 𝑖 + 1. +, Nếu 𝑝 𝑖 không là ước của 𝑏 ⇒ gcd (𝑝 𝑖𝛼 𝑖 , 𝑏) = 1. Theo định lý Euler thì 𝛼𝑖 𝑏 𝜙( 𝑝 𝑖 ) ≡ 1 (mod 𝑝 𝑖𝛼 𝑖 ), mà 𝜙 (𝑝 𝑖𝛼 𝑖 ) | 𝜙 (𝑎) ⇒ 𝑏 𝑛+𝜙(𝑎) ≡ 𝑏 𝑛 (mod 𝑝 𝑖𝛼 𝑖 ) . Tức là với mỗi 𝑝 𝑖 ta luôn chọn được 𝑛 𝑖 sao cho với mọi 𝑛 > 𝑛 𝑖 thì 𝑏 𝑛+𝜙(𝑎) ≡ 𝑏 𝑛 (mod 𝑝 𝑖𝛼 𝑖 ). Do đó nếu ta chọn 𝑁 = max {𝑛 𝑖 } khi đó với mọi 𝑛 > 𝑁 , ta có 𝑏 𝑛+𝜙(𝑎) ≡ 𝑏 𝑛 (mod 𝑝 𝑖𝛼 𝑖 ) với mọi 1 ≤ 𝑖 ≤ 𝑘. Bởi vì các 𝑝 𝑖 là các số nguyên tố phân biệt nên 𝑏 𝑛+𝜙(𝑎) ≡ 𝑏 𝑛 (mod 𝑎) với mọi 𝑛 > 𝑁 (đpcm). Trở lại bài toán, ta sẽ chỉ ra bằng quy nạp theo 𝑎 rằng với mọi số nguyên dương 𝑎, 𝑏 thì luôn tồn tại vô hạn số nguyên dương 𝑛 sao cho 𝑎 là ước của 𝑏 𝑛 − 𝑛 (**). +, Với 𝑎 = 1 rõ ràng khẳng định (**) đúng. +, Giả sử (*) đúng với mọi số nguyên 1 ≤ 𝑎 < 𝑎0 (𝑎0 ≥ 2). Do 𝜙 (𝑎) < 𝑎 nên theo giả thiết quy nạp và theo (*) suy ra tồn tại vô số số nguyên dương 𝑛 sao cho 𝜙 (𝑎) | (𝑏 𝑛 − 𝑛) và 𝑏 𝑛+𝜙(𝑎) ≡ 𝑏 𝑛 (mod 𝑎) . Với mỗi 𝑛 như vậy, đặt 𝑏𝑛− 𝑛 𝑡= và 𝑛1 = 𝑏 𝑛 = 𝑛 + 𝑡𝜙 (𝑎) . 𝜙 (𝑎) Thế thì theo định lý Euler 𝑏 𝑛1 − 𝑛1 ≡ 𝑏 𝑛+𝑡𝜙(𝑎) − 𝑛 − 𝑡𝜙 (𝑎) ≡ 𝑏 𝑛 − 𝑛 − 𝑡𝜙 (𝑎) ≡ 0 (mod 𝑎) , tức là 𝑛1 = 𝑏 𝑛 thỏa mãn yêu cầu. Theo giả thiết quy nạp, có vô hạn số 𝑛1 = 𝑏 𝑛 thỏa mãn điều kiện trong khẳng định (**), từ đó (**) đúng. Và do đó không có các số nguyên dương 𝑎, 𝑏 nào thỏa mãn đầu bài. Bài 8. (CĐT Mỹ 2003) Tìm tất cả các bộ ba số nguyên tố (𝑝, 𝑞, 𝑟) thỏa mãn 𝑝 | 𝑞 𝑟 + 1, 𝑞 | 𝑟 𝑝 + 1, 𝑟 | 𝑝 𝑞 + 1. 12
  • 13. www.VNMATH.com Lời giải. Giả sử 𝑝, 𝑞, 𝑟 thỏa mãn đầu bài suy ra chúng là ba số nguyên tố phân biệt. Trường hợp 1 : Cả ba số 𝑝, 𝑞, 𝑟 đều lẻ. Ta có 𝑝 | 𝑞 𝑟 + 1 nên theo ví dụ 10 ta có 2𝑟 | 𝑝 − 1 hoặc 𝑝 | 𝑞 2 − 1. +, Nếu 2𝑟 | 𝑝 − 1 ⇒ 𝑝 ≡ 1 (mod 𝑟) ⇒ 0 ≡ 𝑝 𝑞 + 1 ≡ 2 (mod 𝑟) mà 𝑟 > 2 nên mâu thuẫn với đầu bài. +, Nếu 𝑝 | 𝑞 2 − 1 = (𝑞 − 1) (𝑞 + 1). Hơn nữa do 𝑝 lẻ và 𝑝 − 1, 𝑝 + 1 là các số chẵn nên ta có 𝑞−1 𝑞+1 𝑝| hoặc 𝑝 | . 2 2 𝑞+1 Khi đó ta luôn có 𝑝 ≤ < 𝑞. Bằng cách chứng minh tương tự ta cũng có 2 𝑞 < 𝑟, 𝑟 < 𝑝. Ta cũng có mâu thuẫn nên trường hợp này không xảy ra được. Trường hợp 2 : Một trong ba số 𝑝, 𝑞, 𝑟 phải bằng 2, do tính xoay vòng nên ta có thể giả sử 𝑝 = 2. Khi đó 𝑟 | 2 𝑞 + 1, lại theo ví dụ 10 ta có 2𝑞 | 𝑟 − 1 hoặc 𝑟 | 22 − 1. Rõ ràng nếu 2𝑞 | 𝑟 − 1 như trường hợp trên ta cũng có 𝑟 ≡ 1 (mod 𝑞) ⇒ 0 ≡ 𝑟 + 1 ≡ 2 (mod 𝑞) ⇒ 𝑞 = 2 ta có mâu thuẫn vì 𝑝, 𝑞, 𝑟 phân biệt. Do đó 𝑟 | 22 − 1 𝑝 suy ra 𝑟 = 3 và 𝑞 | 𝑟2 + 1 = 10 ⇒ 𝑞 = 5. (thỏa mãn). Vậy các bộ cần tìm là các hoán vị xoay vòng của (2, 5, 3). Bài 9. (Iran 2007) Cho 𝑛 là số nguyên dương và 𝑛 = 22007 𝑘 + 1 với 𝑘 là số nguyên lẻ. Chứng minh rằng 𝑛 2 𝑛−1 + 1. Lời giải. (︀ )︀22007 Giả sử 𝑛 | 2 𝑛−1 + 1 = 2 𝑘 + 1. Gọi 𝑝 là một số ước nguyên tố tùy ý của 𝑛 2007 2008 và đặt 𝑡 = 2 , 𝑑 = ord 𝑝 (𝑡). Ta có 𝑡2 𝑘 ≡ −1 (mod 𝑝) ⇒ 𝑡2 ≡ 1 (mod 𝑝). Suy 2008 2007 2008 ra 𝑑 | 2 , 𝑑 2 , tức là 𝑑 = 2 mà 𝑑 | 𝑝 − 1 ⇒ 𝑝 ≡ 1 (mod 22008 ). Do đó 𝑛 ≡ 1 mod 22008 ⇒ 22007 𝑘 ≡ 0 mod 22008 ⇒ 2 | 𝑘. (︀ )︀ (︀ )︀ Điều này mâu thuẫn với giả thiết, ta có kết luận của bài toán. Bài 10. (Việt Nam 2001) Cho 𝑛 là số nguyên dương và 𝑎, 𝑏 là các số nguyên tố 𝑛 𝑛 cùng nhau lớn hơn 1. Giả sử 𝑝, 𝑞 là hai ước lẻ lớn hơn 1 của 𝑎6 + 𝑏6 . Tìm số dư 𝑛 𝑛 trong phép chia của 𝑎6 + 𝑏6 cho 6.12 𝑛 . Lời giải. Trước hết ta chứng minh 𝑝 ≡ 1 ≡ 𝑞 (mod 2 𝑛+1 ). Vì (𝑎, 𝑏) = 1 suy ra (𝑎, 𝑝) = (𝑏, 𝑝) = 1. Gọi 𝑏′ là nghịch đảo của 𝑏 mô-đun 𝑝 và đặt 𝐴 = 𝑎𝑏′ . Khi đó theo giả thiết 𝑛 𝑛 𝑛 𝑛 𝑝 | 𝑎6 + 𝑏6 | 𝐴6 + 1 | 𝐴2.6 − 1. 13
  • 14. www.VNMATH.com Suy ra ord 𝑝 (𝐴) | 2.6 𝑛 , ord 𝑝 (𝐴) | 6 𝑛 . Do đó ta suy ra ord 𝑝 (𝐴) = 2 𝑛+1 3 𝑛 mà ord 𝑝 (𝐴) | 𝑝 − 1 ⇒ 𝑝 ≡ 1 (mod 2 𝑛+1 ). Chứng minh tương tự cho 𝑞. Vậy 𝑝 ≡ (︁ )︁ 𝑛 𝑛 2 𝑞 ≡ 1 (mod 2 𝑛+1 ) ⇒ 𝑝6 ≡ 𝑞 6 ≡ 6 𝑛 .2 𝑛+1 + 1 mod (2 𝑛+1 ) (theo khai triển 𝑛 𝑛 Newton). Từ đó 𝑝6 ≡ 𝑞 6 ≡ 1 (mod 22𝑛+1 ). (*) 𝑛 𝑛 Hơn nữa ta có 𝜙 (6 𝑛+1 ) = 2.6 𝑛 ⇒ 𝑝6 ≡ ±1 (mod 6 𝑛+1 ). Giả sử 𝑝6 ≡ 𝑛 𝑛 −1 (mod 6 𝑛+1 ) suy ra 𝑝6 ≡ −1 (mod 2 𝑛+1 ) mà từ (*) ta có 𝑝6 ≡ 1 (mod 2 𝑛+1 ) ⇒ 𝑛 𝑛 2𝑝6 ≡ 0 (mod 2 𝑛+1 ) mâu thuẫn vì 𝑝 lẻ. Do đó ta thu được 𝑝6 ≡ 1 (mod 22𝑛+1 ) 𝑛 𝑛 𝑛 và 𝑝6 ≡ 1 (mod 6 𝑛+1 ) từ đó 𝑝6 ≡ 1 (mod 6.12 𝑛 ). Tương tự 𝑞 6 ≡ 1 (mod 6.12 𝑛 ). Suy ra số dư cần tìm là 2. Bài 11. (Trung Quốc 2009) Tìm tất cả các cặp số nguyên tố (𝑝, 𝑞) sao cho 𝑝𝑞 | 5 𝑝 + 5 𝑞 . Lời giải. Rõ ràng cặp (5, 5) thỏa mãn. Xét trường hợp 𝑝 = 5, 𝑞 ̸= 5 ta có 5𝑞 | 55 + 5 𝑞 ⇒ 𝑞 | 625 + 5 𝑞−1 . Theo định lý Fermat suy ra 𝑞 là ước của 626, trong trường hợp này ta có nghiệm (5, 313) và (5, 2). Tương tự trường hợp 𝑞 = 5, 𝑝 ̸= 5 ta có hai nghiệm (313, 5) và (2, 5). Xét 𝑝 = 2, 𝑞 ̸= 5 ta có 2𝑞 | 25 + 5 𝑞 ⇒ 25 + 5 𝑞 ≡ 0 (mod 2𝑞) và theo định lý Fermat ta thu được 30 ≡ 0 (mod 2𝑞) từ đó dễ thấy 𝑞 = 3 thỏa mãn. Do đó trong trường hợp trong hai số 𝑝, 𝑞 có một số bằng 2 và số còn lại khác 5 ta có hai nghiệm (2, 3) và (3, 2). Xét 𝑝, 𝑞 là hai số khác 2 và 5. Ta có 5 𝑝 + 5 𝑞 ≡ 0 (mod 𝑝𝑞) ⇒ 5 + 5 𝑞 ≡ 0 (mod 𝑝) ⇒ 5 𝑞−1 ≡ −1 (mod 𝑝) ⇒ 52(𝑞−1) ≡ 1 (mod 𝑝). Tương tự ta có 5 𝑝−1 ≡ −1 (mod 𝑞). Chọn 𝑘 sao cho 2 𝑘 ‖ ord 𝑝 5 mà ord 𝑝 5 | 𝑝 − 1, và ord 𝑝 5 | 2 (𝑞 − 1) ; ord 𝑝 5 (𝑞 − 1). Suy ra 2 𝑘−1 ‖ 𝑞 − 1, tức là số mũ lớn nhất của 2 mà là ước của 𝑝 − 1 lớn hơn số mũ lớn nhất của 2 mà là ước của 𝑞 − 1. Do tính đối xứng nên ta cũng có số mũ lớn nhất của 2 mà là ước của 𝑞 − 1 lớn hơn số mũ lớn nhất của 2 mà là ước của 𝑝 − 1. Đây là điều mâu thuẫn, trong trường hợp này bài toán vô nghiệm. Vậy các nghiệm của bài toán là (2, 3) ; (2, 5) ; (5, 5) ; (5, 313) và các nghiệm đối xứng của nó là (3, 2) ; (5, 2) ; (313, 5). Bài 12. (CĐT Romanian 2000) Cho 𝑎 > 1 là số nguyên dương. Tìm số nguyên dương 𝑛 nhỏ nhất sao cho 22000 | 𝑎 𝑛 − 1. Lời giải. Ta có 𝑎 𝑛 ≡ 1 (mod 22000 ). Theo định lý Euler suy ra 𝑛 | 𝜙 (22000 ) = 21999 ⇒ 𝑛 là lũy thừa của 2. Đặt 𝑛 = 2 𝑘 với 𝑘 ≥ 0. Khi đó ta có (︁ 𝑘−1 )︁ 𝑎 𝑛 − 1 = (𝑎 − 1) (𝑎 + 1) 𝑎2 + 1 𝑎4 + 1 · · · 𝑎2 + 1 . (︀ )︀ (︀ )︀ (︁ )︁ 𝑘−1 Mà 𝑎2𝑚 + 1 ≡ 2 (mod 4) nên 2 𝑘 − 1 ‖ (𝑎2 + 1) (𝑎4 + 1) · · · 𝑎2 + 1 . Do đó ta cần tìm 𝑘 nhỏ nhất để 22001−𝑘 | (𝑎 − 1) (𝑎 + 1) = 𝑎2 − 1. Vì thế nếu đặt 14
  • 15. www.VNMATH.com 𝑚 = 𝑣2 (𝑎2 − 1) (tức là số mũ của 2 lớn nhất của 𝑎2 − 1) ta có {︂ 0 nếu 𝑚 ≥ 2001 𝑘= 2001 − 𝑚 nếu 𝑚 < 2001. Vậy 𝑛 = 2 𝑘 với 𝑘 xác định theo công thức trên. Bài 13. (CĐT Romanian 2005) Giải phương trình trong tập số nguyên dương 3 𝑥 = 2 𝑥 𝑦 + 1. Lời giải. Bằng quy nạp ta dễ dàng chứng minh được hai nhận xét sau: 𝑛 ∘ Với mọi số nguyên dương 𝑛 ta có 2 𝑛+2 ‖ 32 − 1, ∘ Với mọi số nguyên dương 𝑛 ≥ 3 ta có 2 𝑛 > 𝑛 + 2. Viết lại phương trình đã cho dưới dạng 2 𝑥 𝑦 = (3 − 1) 3 𝑥−1 + 3 𝑥−2 + · · · + 3 + 1 (︀ )︀ = 2 3 𝑥−1 + 3 𝑥−2 + · · · + 3 + 1 . (︀ )︀ Đặt 𝑥 = 2 𝑛 𝛼 với 𝛼, 𝑛 ∈ N và 𝛼 lẻ. +, Nếu 𝑛 = 0 tức là 𝑥 là số lẻ, mà 3 𝑥−1 + 3 𝑥−2 + ... + 3 + 1 là tổng của 𝑥 số lẻ nên là số lẻ, suy ra 2 ‖ 2 𝑥 𝑦 ⇒ 𝑥 = 1 ⇒ 𝑦 = 1. Từ đó (𝑥, 𝑦) = (1, 1) là một nghiệm của phương trình. )︀ (︀ 𝑛 +, Nếu 𝑛 ≥ 1, ta có 32 𝑛 (︀ 𝑛 )︀ 𝛼 22 𝛼 𝑦 = 32 −1𝛼 (︀ 𝑛 )︀ (︁(︀ 2 𝑛 )︀ 𝛼−1 (︀ 2 𝑛 )︀ 𝛼−2 (︀ 𝑛 )︀ )︁ = 32 − 1 3 + 3 + · · · + 32 + 1 . (︀ 𝑛 )︀ 𝛼−1 (︀ 2 𝑛 )︀ 𝛼−2 (︀ 𝑛 )︀ Mà 32 + 3 + · · · + 32 + 1 là tổng của 𝑛 số lẻ nên là số lẻ. Do đó 𝑛 theo nhận xét trên ta có 2 𝑛+2 ‖ 3 𝑥 − 1 = 22 𝛼 𝑦. Suy ra 𝑛 + 2 ≥ 2 𝑛 𝛼 ⇒ 𝑛 ≤ 2. Kiểm tra trực tiếp dễ thấy với 𝑛 = 1 ⇒ 𝛼 = 1 ⇒ 𝑥 = 2, 𝑦 = 2; với 𝑛 = 2 tương tự ta có 𝛼 = 1 và từ đó 𝑥 = 4, 𝑦 = 5. Vậy phương trình có các nghiệm (1, 1) ; (2, 2) và (4, 5). Bài 14. (Dự tuyển IMO 1991) Tìm số nguyên dương 𝑘 lơn nhất sao cho 1992 1990 1991 𝑘 | 19901991 + 19921991 . Lời giải. Trước hết ta chứng minh bằng quy nạp rằng với mọi số lẻ 𝑎 ≥ 3 và số nguyên 𝑛 ≥ 0 thì ta có 𝑛 𝑛 𝑎 𝑛+1 ‖ (𝑎 + 1) 𝑎 − 1 và 𝑎 𝑛+1 ‖ (𝑎 − 1) 𝑎 − 1. 𝑛 Rõ ràng 𝑛 = 0 hiển nhiên đúng. Giả sử (𝑎 + 1) 𝑎 = 1 + 𝑁 𝑎 𝑛+1 , 𝑎 𝑁 . Khi đó 𝑛+1 )︀ 𝑎 (𝑎 + 1) 𝑎 = 1 + 𝑁 𝑎 𝑛+1 = 1 + 𝑎.𝑁 𝑎 𝑛+1 + 𝐶 2 𝑁 2 𝑎2𝑛+2 + 𝑀 𝑎3𝑛+3 . (︀ 𝑎 15
  • 16. www.VNMATH.com 𝑛+1 Mà 𝑎 lẻ nên 𝐶 2 chia hết cho 𝑎. Suy ra (𝑎 + 1) 𝑎 𝑎 = 1 + 𝑁 ′ 𝑎 𝑛+2 với 𝑎 𝑁 ′. Do đó ta có ngay 1992 1990 19911993 ‖ 19901991 + 1, 19911991 ‖ 19921991 − 1. Vậy 𝑘 = 1991. 2𝑛+1 Bài 15. (IMO 1990) Tìm tất cả các số nguyên dương 𝑛 sao cho là số 𝑛2 nguyên. Lời giải Với 𝑛 = 1 thỏa mãn. Xét 𝑛 > 1 ⇒ 𝑛 lẻ. Giả sử 𝑝 ≥ 3 là ước nguyên tố nhỏ nhất của 𝑛. Ta có gcd (𝑝 − 1, 𝑛) = 1 và 𝑝 | 2 𝑛 + 1 | 22𝑛 − 1. Theo định lý Fermat 𝑝 | 2 𝑝−1 − 1 suy ra 𝑝 | gcd (2 𝑝−1 − 1, 22𝑛 − 1) = 2gcd(2𝑛,𝑝−1) − 1. Mà gcd (2𝑛, 𝑝 − 1) ≤ 2 suy ra 𝑝 | 3 ⇒ 𝑝 = 3. Đặt 𝑛 = 33 𝑑 với 2, 3 𝑑. Ta có nhận xét: Nếu 2 𝑚 − 1 chia hết cho 3 𝑟 thì 𝑚 chia hết cho 3 𝑟−1 (xem ví dụ 3). Hơn nữa do 32𝑘 | 𝑛2 | 22𝑛 − 1 ⇒ 32𝑘−1 | 𝑛 = 3 𝑘 𝑑 ⇒ 𝑘 = 1. Giả sử 𝑑 > 1 và 𝑞 là ước nguyên tố nhỏ nhất của 𝑑. Rõ ràng 𝑞 lẻ và 𝑞 ≥ 5, gcd (𝑛, 𝑞 − 1) | 3. Khi đó ta có 𝑞 | 22𝑛 − 1, 𝑞 | 2 𝑞−1 − 1 ⇒ 𝑞 | 2gcd(2𝑛,𝑞−1) − 1 | 26 − 1 = 32 .7. Do đó 𝑞 = 7 ⇒ 7 | 𝑛 | 2 𝑛 + 1 mà 2 𝑛 + 1 ≡ 2, 3, 5 (mod 7) mâu thuẫn. Vậy 𝑑 = 1 ⇒ 𝑛 = 3 thỏa mãn. Đáp số 𝑛 = 1, 𝑛 = 3. Bài 16. (CĐT Trung Quốc 2005) Cho 𝑏, 𝑚, 𝑛 là các số nguyên dương 𝑏 > 1, 𝑚 ̸= 𝑛. Chứng minh rằng nếu 𝑏 𝑚 − 1 và 𝑏 𝑛 − 1 có cùng các ước nguyên tố thì 𝑏 + 1 là lũy thừa của 2. Lời giải. Trước hết ta chứng minh bài toán cho trường hợp 𝑛 = 1, tức là nếu 𝑎 > 1, 𝑘 > 1 và 𝑎 𝑘 − 1 và 𝑎 − 1 có cùng các nhân tử nguyên tố thì 𝑘 và 𝑎 + 1 là lũy thừa của 2. Phản chứng, giả sử 𝑝 là ước nguyên tố lẻ của 𝑘. Khi đó mọi ước nguyên tố 𝑞 của 1 + 𝑎 + 𝑎2 + · · · + 𝑎 𝑝−1 là ước của 1 + 𝑎 + 𝑎2 + · · · + 𝑎 𝑘−1 | 𝑎 𝑘 − 1 | 𝑎 − 1. Suy ra 1 + 𝑎 + 𝑎2 + · · · + 𝑎 𝑝−1 ≡ 𝑝 (mod 𝑞) ≡ 0 (mod 𝑞), nên 𝑝 = 𝑞. Do đó 1 + 𝑎 + 𝑎2 + · · · + 𝑎 𝑝−1 = 𝑝 𝑡 , 𝑡 > 1. Đặt 𝑎 = 𝑢𝑝 + 1 ⇒ 1 + 𝑎 + 𝑎2 + · · · + 𝑎 𝑝−1 ≡ 𝑝 (mod 𝑝2 ) mâu thuẫn. Vậy 𝑘 là lũy thừa của 2. Gọi 𝑟 là ước nguyên tố bất kỳ của 𝑎 + 1 ⇒ 𝑟 | 𝑎 𝑘 − 1 ⇒ 𝑟 | 𝑎 − 1 ⇒ 𝑟 = 2. Trong trường hợp 𝑛 bất kỳ, đặt 𝑑 = gcd (𝑚, 𝑛) , 𝑘 = 𝑚/𝑑. Ta thu được 𝑏 𝑘𝑏 − 1 và 𝑏 𝑑 − 1 có cùng các ước nguyên tố. Do đó theo chứng minh trên ta có 𝑘 là lũy thừa của 2 và 𝑏 𝑑 + 1 = 2 𝑡 , 𝑡 ≥ 2. Xét mô-đun 4 suy ra 𝑑 lẻ. Giả sử 𝑟 là ước nguyên tố của 𝑏 + 1 suy ra 𝑟 | 𝑏 + 1 | 𝑏2 − 1 | 𝑏 𝑘𝑑 − 1 ⇒ 𝑟 | 𝑏 𝑑 − 1 ≡ −2 (mod 𝑟). Từ đó 𝑟 = 2, tức là 𝑏 + 1 là lũy thừa của 2. III. Một số bài tập Bài 1. (Dự tuyển IMO 2007) Cho 𝑏, 𝑛 là các số nguyên. Giả sử với mỗi 𝑘 > 1 luôn tồn tại số nguyên 𝑎 𝑘 sao cho 𝑏 − 𝑎 𝑘𝑛 chia hết cho 𝑘. Chứng minh rằng 𝑏 = 𝐴 𝑛 với 𝐴 là số nguyên nào đó. 16
  • 17. www.VNMATH.com Hướng dẫn: Biểu diễn 𝑏 = 𝑝1𝛼1 . . . 𝑝 𝑙𝛼 𝑙 . Do 𝑏2 | 𝑏 − 𝑎 𝑏𝑛2 ⇒ 𝑝 𝑖𝛼 𝑖 ‖ 𝑎 𝑏𝑛2 ∀𝑖. Suy ra 𝑛 | 𝛼 𝑖 ∀𝑖 tức là 𝑏 là lũy thừa đúng bậc 𝑛. Bài 2. (Dự tuyển IMO 2002) Cho 𝑝1 , 𝑝2 , ..., 𝑝 𝑛 là các số nguyên tố phân biệt lớn hơn 3. Chứng minh rằng 2 𝑝1 𝑝2 ...𝑝 𝑛 + 1 có ít nhất 4 𝑛 ước số. Hướng dẫn: Chú ý nếu 𝑢, 𝑣 lẻ và (𝑢, 𝑣) = 1 thì ước chung lớn nhất của 2 𝑢 + 1 và 2 𝑣 + 1 là 3. Từ đó 2 𝑢𝑣 + 1 chia hết cho (2 𝑢 + 1) (2 𝑣 + 1) /3. Sau đó chứng minh bài toán bằng quy nạp theo 𝑛. Bài 3. (IMO 2006) Xác định tất cả các cặp số nguyên (𝑥, 𝑦) thỏa mãn phương trình 1 + 2 𝑥 + 22𝑥+1 = 𝑦 2 . Hướng dẫn: Xét 𝑥 > 0, 𝑦 > 0 và 𝑦 lẻ. Chú ý 2 𝑥 (1 + 2 𝑥+1 ) = (𝑦 − 1) (𝑦 + 1) từ đó ta có thể biểu diễn 𝑦 theo 𝑥. Suy ra 𝑥 = 4, 𝑦 = 23. Đáp số (0, ±2) ; (4, ±23) . Bài 4. (Dự tuyển IMO 2001) Cho 𝑝 là số nguyên tố lớn hơn 3. Chứng minh rằng tồn tại số nguyên 𝑎 với 1 ≤ 𝑎 ≤ 𝑝 − 2 sao cho hoặc 𝑎 𝑝−1 − 1 hoặc (𝑎 + 1) 𝑝−1 − 1 chia hết cho 𝑝2 . Hướng dẫn: Đặt 𝐴 = {1 ≤ 𝑎 ≤ 𝑝 − 1 : 𝑎 𝑝−1 ̸≡ 1 (mod 𝑝2 )}. Vì ít nhất một trong hai số 𝑎, 𝑝 − 𝑎 thuộc 𝐴 và 1 ∈ 𝐴 và |𝐴| ≥ (𝑝 − 1) /2 = 𝑘 và giả sử có / đúng một trong hai số 2𝑖, 2𝑖 + 1 (1 ≤ 𝑖 ≤ 𝑘 − 1) thuộc 𝐴. Xét 𝑝 ≥ 7 ⇒ 2𝑘 − 1 = 𝑝 − 2 ∈ 𝐴 ⇒ 𝑝 − 3 = 2𝑘 − 2 ∈ 𝐴. Bằng cách xét mô-đun 𝑝2 ⇒ 2𝑘 − 3 ∈ 𝐴, tức là / 𝑝 − 3, 𝑝 − 4 thỏa mãn. Bài 5. (CĐT Việt Nam 2008) Cho 𝑚, 𝑛 là các số nguyên dương. Chứng minh rằng (2𝑚 + 3) 𝑛 + 1 chia hết cho 6𝑚 khi và chỉ khi 3 𝑛 + 1 chia hết cho 4𝑚. Hướng dẫn: Khai triển Newton, bài toán trở thành chứng minh 2𝑚 | 3 𝑛 + 1 và 3 | (2𝑚) 𝑛 + 1 khi và chỉ khi 4𝑚 | 3 𝑛 + 1. Bài 6. (Dự tuyển IMO 2000) Xác định tất cả các bộ ba số nguyên dương (𝑎, 𝑚, 𝑛) sao cho 𝑎 𝑚 + 1 là ước của (𝑎 + 1) 𝑛 . Hướng dẫn: Chú ý nếu 𝑢 | 𝑣 𝑙 ⇒ 𝑢 | (gcd (𝑢, 𝑣)) 𝑙 . Xét 𝑎 > 1 và 𝑚 > 1, khi đó 𝑚 lẻ. Giả sử 𝑝 là ước nguyên tố của 𝑚. Bằng cách xét mô-đun, suy ra (𝑏 𝑝 + 1) / (𝑏 + 1) = 𝑝 ⇒ 𝑝 = 3. Đáp số: {(1, 𝑚, 𝑛) ; (𝑎, 1, 𝑛) ; (2, 3, 𝑛 ≥ 2)} . Bài 7. (Nga 2000) Hỏi có tồn tại ba số 𝑎, 𝑏, 𝑐 > 1 đôi một nguyên tố cùng nhau và thỏa mãn 𝑏 | 2 𝑎 + 1, 𝑐 | 2 𝑏 + 1, 𝑎 | 2 𝑐 + 1. Hướng dẫn: Ta có 𝑎, 𝑏, 𝑐 là lẻ. Giả sử 𝑎 = min {𝑎, 𝑏, 𝑐}. Nếu 𝑎, 𝑏, 𝑐 nguyên tố suy ra 𝑎 = 3, không tồn tại 𝑏, 𝑐. Trường hợp bất kỳ ta ký hiệu 𝜋 (𝑛) là ước nguyên tố nhỏ nhất của 𝑛. Khi đó nếu 𝑝 là số nguyên tố sao cho 𝑝 | 2 𝑦 + 1 và 𝑝 < 𝜋 (3) thì 𝑝 = 3. Sử dụng tính chất này suy ra mâu thuẫn. Bài 8. (IMO 2000) Hỏi có tồn tại hay không số nguyên dương 𝑛 có đúng 2000 ước nguyên tố và 2 𝑛 + 1 chia hết cho 𝑛. Hướng dẫn: Sử dụng quy nạp theo số các ước số, 𝑘. Chỉ ra rằng ta có thể tìm 𝑛 với đúng 𝑘 ước nguyên tố sao cho 𝑛 | (2 𝑛 + 1) và có một ước nguyên tố của 2 𝑛 + 1 nhưng không là ước của 𝑛. 17
  • 18. www.VNMATH.com Bài 9. (CĐT Iran 2006) Cho 𝑛 là số nguyên dương. Tìm tất cả các bộ 𝑛 số 𝑎1 , 𝑎2 , ..., 𝑎 𝑛 đôi một phân biệt và đôi một nguyên tố cùng nhau sao cho với mọi 1 ≤ 𝑖 ≤ 𝑛 ta có 𝑎1 + 𝑎2 + · · · + 𝑎 𝑛 | 𝑎1 + 𝑎2 + · · · + 𝑎 𝑖𝑛 . 𝑖 𝑖 Hướng dẫn: Chứng minh 𝑎1 +𝑎2 +· · ·+𝑎 𝑛 | 𝑛 (𝑛 − 1) và hai số này bằng nhau. Nếu 𝑛 ≥ 8 thì 𝑎1 + 𝑎2 + · · · + 𝑎 𝑛 ≥ 1 + 2 + 3 + 5 + · · · + 2𝑛 + 1 = 𝑛2 − 7 > 𝑛2 − 𝑛. Đáp số 𝑛 = 1, 𝑎1 tùy ý. Bài 10. (Iran 2004) Cho 𝑎1 , 𝑎2 , ..., 𝑎 𝑛 là các số nguyên không đồng thời bằng nhau. Chứng minh rằng tồn tại vô số số nguyên tố 𝑝 sao cho có tồn tại số nguyên dương 𝑘 sao cho 𝑝 | 𝑎1𝑘 + 𝑎2𝑘 + · · · + 𝑎 𝑘𝑛 . Hướng dẫn: Ta có thể giả sử gcd (𝑎1 , 𝑎2 , ..., 𝑎 𝑛 ) = 1 và 𝑎 𝑛 ≥ 2. Giả sử chỉ có hữu hạn các số nguyên tố 𝑝1 , ..., 𝑝 𝑚 . Chọn 𝑘 thích hợp để suy ra 𝑎1𝑘 + 𝑎2𝑘 + · · · + 𝑎 𝑘𝑛 có ước nguyên tố lớn hơn 𝑝1 𝑝2 ...𝑝 𝑚 . Bài 11. (Dự tuyển IMO 2002) Tìm tất cả các cặp số nguyên 𝑚, 𝑛 ≥ 3 sao cho tồn tại vô hạn các số nguyên 𝑎 thỏa mãn 𝑎 𝑚+ 𝑎−1 𝑎 𝑛 + 𝑎2 − 1 là một số nguyên. Hướng dẫn: Rõ ràng 𝑛 < 𝑚. Sử dụng tính chất đa thức suy ra biểu thức 𝐴 đã cho nhận giá trị nguyên với mọi 𝑎. Đa thức ở tử số chia hết đa thức ở mẫu số nên chúng có nghiệm chung 𝛼 ∈ (0, 1) từ đó đánh giá theo 𝛼 suy ra 𝑚 < 2𝑛. Cuối cùng chọn 𝑎 = 2 và xét mô-đun 𝑑 = 2 𝑛 + 3 (mẫu số) ta được (𝑚, 𝑛) = (5, 3) . Bài 12. (Trung Quốc 2008) Tìm tất cả các bộ ba số (𝑝, 𝑞, 𝑛) sao cho 𝑞 𝑛+2 ≡ 3 𝑛+2 (mod 𝑝 𝑛 ) , 𝑝 𝑛+2 ≡ 3 𝑛+2 (mod 𝑞 𝑛 ) trong đó 𝑝, 𝑞 là các số nguyên tố lẻ và 𝑛 là số nguyên dương. Hướng dẫn: Nếu hai trong ba số 𝑝, 𝑞, 3 bằng nhau ta có nghiệm (3, 3, 𝑛). Xét trường hợp chúng phân biệt và 𝑝 > 𝑞. Ta có 𝑝 𝑛 𝑞 𝑛 | 𝑝 𝑛+2 + 𝑞 𝑛+2 − 3 𝑛+2 < 2𝑝 𝑛+2 ⇒ 𝑝2 > 𝑞 𝑛−1 . Mà 𝑞 𝑛+2 − 3 𝑛+2 ≥ 𝑝 𝑛 . Suy ra 𝑛 ≤ 3. 18